NR224 Fundamentals 1 Final Review

Lakukan tugas rumah & ujian kamu dengan baik sekarang menggunakan Quizwiz!

Which of the following best describes an iatrogenic infection? 1. It results from a diagnostic or therapeutic procedure. 2. It results from extended infection of the urinary tract. 3. It involves an incubation period of 3 to 4 weeks before it can be detected. 4. It occurs when patients are infected with their own organisms as a result of immunodeficiency.

1

The body alignment of the patient in the tripod position includes the following: (Select all that apply.) 1. An erect head and neck 2. Straight vertebrae 3. Extended hips and knees 4. Axillae resting on the crutch pads 5. Bent knees and hips

1, 2, 3.

The infection control nurse has asked the staff to work on reducing the number of iatrogenic infections on the unit. Which of the following actions on your part would contribute to reducing health care-acquired infections? (Select all that apply.) 1. Teaching correct handwashing to assigned patients 2. Using correct procedures in starting and caring for an intravenous infusion 3. Providing perineal care to a patient with an indwelling urinary catheter 4. Isolating a patient who has just been diagnosed as having tuberculosis 5. Decreasing a patient's environmental stimuli to decrease nausea

1, 2, 3.

Which of the following examples are steps of nursing assessment? (Select all that apply) 1. Collection of information form patient's family members. 2. Recognition that further observations are needed to clarify information. 3. Comparison of data with another source to determine data accuracy. 4. Complete documentation of observational information. 5. Determining which medications to administer based o a patients assessment data.

1, 2, 3.

In which of the following examples is a nurse applying critical thinking skills in practice? (Select all that apply) 1. The nurse thinks back about a personal experience before administering a medication subcutaneously. 2. The nurse uses a pain-rating scale to measure a patients pain. 3. The nurse explains a procedure step by step for giving an enema to a patient care technician. 4. the nurse gathers data on a patient with a mobility limitation to identify a nursing diagnosis. 5. A nurse offers support to a colleague who has whitnessed a stressful event.

1, 2, 4.

The nurse enters the room of a 82-year-old patient for when she has not caerd previously. The nurse notices that the patient wears a hearing aid. The patient looks up as the nurse approaches the bedside. Which of the following approaches are likely to be effective with an older adult? (Select all that apply) 1. Listen attentively to the patients story. 2. Use gestures that reinforce your question of comments. 3. Stand back away from the bedside. 4. Maintain direct eye contact. 5. Ask questions quickly to reduce the patient's fatique.

1, 2, 4.

Which of the following describes a nurse's application of a specific knowledge base during critical thinking? (Select all that apply) 1. Initiative in reading current evidence from the literature. 2. Application of nursing theory. 3. Reviewing policy ad procedure manual. 4. Considering holistic view of patient needs. 5. Previous time caring for a specific group of patients.

1, 2, 4.

Which of these statements are true regarding disinfection and cleaning? (Select all that apply.) 1. Proper cleaning requires mechanical removal of all soil from an object or area. 2. General environmental cleaning is an example of medical asepsis. 3. When cleaning a wound, wipe around the wound edge first and then clean inward toward the center of the wound. 4. Cleaning in a direction from the least to the most contaminated area helps reduce infections. 5. Disinfecting and sterilizing medical devices and equipment involve the same procedures.

1, 2, 4.

The effects of immobility on the cardiac system include which of the following? (Select all that apply.) 1. Thrombus formation 2. Increased cardiac workload 3. Weak peripheral pulses 4. Irregular heartbeat 5. Orthostatic hypotension

1, 2, 5.

Which of the following patients are at most risk for tachypnea? (Select all that apply.) 1. Patient just admitted with four rib fractures 2. Woman who is 9 months' pregnant 3. Adult who has consumed alcoholic beverages 4. Adolescent waking from sleep 5. Three-pack-per-day smoker with pneumonia

1, 2, 5.

Which type of personal protective equipment are staff required to wear when caring for a pediatric patient who is placed into airborne precautions for confirmed chickenpox/herpes zoster? (Select all that apply.) 1. Disposable gown 2. N 95 respirator mask 3. Face shield or goggles 4. Surgical mask 5. Gloves

1, 2, 5.

A patient is receiving 5000 units of heparin subcutaneously every 12 hours while on prolonged bed rest to prevent thrombophlebitis. Because bleeding is a potential side effect of this medication, the nurse should continually assess the patient for the following signs of bleeding: (Select all that apply.) 1. Bruising 2. Pale yellow urine 3. Bleeding gums 4. Coffee ground-like vomitus 5. Light brown stool

1, 3, 4.

When a nurse conducts an assessment, data about a patient often comes from which of the following sources? (Select all that apply.) 1. An observation of how a patient turns and moves in bed. 2. The unit policy and procedure manual. 3. The care recommendations of a physical therapist. 4. The results of a diagnostic x-ray film. 5. Your experiences in caring for other patients with similar problems.

1, 3, 4.

Put the following steps for removal of protective barriers after leaving an isolation room in order. 1. Remove gloves. 2. Perform hand hygiene. 3. Remove eyewear or goggles. 4. Untie top and then bottom mask strings and remove from face. 5. Untie waist and neck strings of gown. Remove gown, rolling it onto itself without touching the contaminated side.

1, 3, 5, 4, 2.

A patient has an indwelling urinary catheter. Why does an indwelling urinary catheter present a risk for urinary tract infection? (Select all that apply.) 1. It allows migration of organisms into the bladder. 2. The insertion procedure is not done under sterile conditions. 3. It obstructs the normal flushing action of urine flow. 4. It keeps an incontinent patient's skin dry. 5. The outer surface of the catheter is not considered sterile.

1, 3.

The nurse is teaching a patient how to sit with crutches. In which order will the nurse present the instructions starting with the first step? 1. Place both crutches in one hand. 2. Grasp arm of chair with free hand. 3. Completely lower self into chair. 4. Transfer weight to crutches and unaffected leg.

1, 4, 2, 3.

A nurse gathers the following assessment data. Which of the following cues together forms a pattern suggesting a problem? (Select all that apply) 1. The skin around the wound is tender to touch. 2. Fluid intake for 8 hours is 800 mL. 3. Patient has a heart rate of 78 b/m and regular. 4. Patient has drainage from surgical wound. 5. Body temperature is 38.3 C. (101 F).

1, 4, 5.

A nurse makes the following statement during a change of shift report to another nurse. "I assessed Mr. Diaz, my 61 year old patient from Chile. He fell at home and hurt his back 3 days ago. He has some difficulty turning in bed and he says that he has pain that radiates down his leg. He rates the pain at a 6, and he moves slowly as he transfers to a chair." What can the nurse who is beginning a shift do to validate the previous nurses assessment findings when she conducts rounds on the patent? (Select all that apply) 1. The nurse ask the patient to rate his pain 0-10. 2. The nurse ask the patient what caused his fall. 3. The nurse ask the patient if he has had pain in his back in the past. 4. The nurse assesses the patients lower-limb strength. 5. The nurse ask the patient what pain medication is most effective in managing his pain.

1, 4.

Place the following options in the order in which elastic stockings should be applied. 1. Identify patient using two identifiers. 2. Smooth any creases or wrinkles. 3. Slide the remainder of the stocking over the patient's heel and up the leg 4. Turn the stocking inside out until heel is reached. 5. Assess the condition of the patient's skin and circulation of the legs. 6. Place toes into foot of the stocking. 7. Use tape measure to measure patient's legs to determine proper stocking size.

1, 5, 7, 4, 6, 3, 2.

A healthy adult patient tells the nurse that he obtained his blood pressure in "one of those quick machines in the mall" and was alarmed that it was 152/72 when his normal value ranges from 114/72 to 118/78. The nurse obtains a blood pressure of 116/76. What would account for the blood pressure of 152/92? (Select all that apply.) 1. Cuff too small 2. Arm positioned above heart level 3. Slow inflation of the cuff by the machine 4. Patient did not remove his long-sleeved shirt 5. Insufficient time between measurements

1, 5.

Pulse range: Infants

120- 160

Normal Blood Pressure

120/80 mmHg

A patient with a right knee replacement is prescribed no weight bearing on the right leg. You reinforce crutch walking knowing that which of the following crutch gaits is most appropriate for this patient? 1. Two-point gait 2. Three-point gait 3. Four-point gait 4. Swing-through gait

2

Respiratory rate: Toddlers

25-32

The nurse is auscultating Mrs. McKinnon's blood pressure. The nurse inflates the cuff to 180 mm Hg. At 156 mm Hg, the nurse hears the onset of a tapping sound. At 130 mm Hg, the sound changes to a murmur or swishing. At 100 mm Hg, the sound momentarily becomes sharper, and at 92 mm Hg, it becomes muffled. At 88mm Hg, the sound disappears. Mrs. McKinnon's blood pressure is: 1. 130/88 mm Hg. 2. 156/88 mm Hg. 3. 180/92 mm Hg. 4. 180/130 mm Hg.

2

The nurse would expect all of the following physiological effects of exercise on the body systems except: 1. Change in metabolic rate. 2. Decrease in cardiac output. 3. Increased repiratory rate and depth. 4. Increased muscle tone, size and strength.

2

Which is the correct gait when a patient is ascending stairs on crutches? 1. A modified two-point gait. (The affected leg is advanced between the crutches to the stairs.) 2. A modified three-point gait. (The unaffected leg is advanced between the crutches to the stairs.) 3. A swing-through gait 4. A modified four-point gait. (Both legs advance between the crutches to the stairs.)

2

Which of the following most motivates a patient to participate in an exercise program? 1. Providing a patient with a pamphlet on exercise 2. Providing information to the patient when he or she is ready to change behavior 3. Explaining the importance of exercise at the time of diagnosis of a chronic disease 4. Providing the patient with a booklet with examples of exercises 5. Providing the patient with a prescribed exercise program

2

Which of the following statements made by an older adult reflects the best understanding of the need to exercise regardless of age? 1. "You are never too old to begin an exercise program." 2. "My granddaughter and I walk together around the high school track 3 times a week." 3. "I purchased a subscription to a runner's magazine for my grandson for Christmas." 4. "When I was a child, I exercised more than I see kids doing today."

2

What does it mean when a patient is diagnosed with a multidrug-resistant organism in his or her surgical wound? (Select all that apply.) 1. There is more than one organism in the wound that is causing the infection. 2. The antibiotics the patient has received are not strong enough to kill the organism. 3. The patient will need more than one type of antibiotic to kill the organism. 4. The organism has developed a resistance to one or more broad-spectrum antibiotics, indicating that the organism will be hard to treat effectively. 5. There are no longer any antibiotic options available to treat the patient's infection.

2, 4.

The nursing assistive personnel (NAP) reports to you that the blood pressure (BP) of the patient in Question 11 is 140/76 on the left arm and 128/72 on the right arm. What actions do you take on the basis of this information? (Select all that apply.) 1. Notify the health care provider immediately 2. Repeat the measurements on both arms using a stethoscope 3. Ask the patient if she has taken her blood pressure medications recently 4. Obtain blood pressure measurements on lower extremities 5. Verify that the correct cuff size was used during the measurements 6. Review the patient's record for her baseline vital signs 7. Compare right and left radial pulses for strength

2, 6.

In which order will the nurse use the nursing process steps during the clinical decision-making process? 1. Evaluating goals 2. Assessing patient needs 3. Planning priorities of care 4. Determining nursing diagnoses 5. Implementing nursing interventions

2,4,3,5,1

A nurse is instructing a patient who has decreased leg strength on the left side how to use a cane. Which action indicates proper cane use by the patient? 1. The patient keeps the cane on the left side of the body. 2. The patient slightly leans to one side while walking. 3. The patient keeps two points of support on the floor at all times. 4. After the patient places the cane forward, he or she then moves the right leg forward to the cane.

3

A nursing assistive personnel asks for help to transfer a patient who is 125 lbs (56.8 kg) from the bed to a wheelchair. The patient is unable to help. What is the nurse's best response? 1. "As long as we use proper body mechanics, no one will get hurt." 2. "The patient only weighs 125 lbs. You don't need my assistance." 3. "Call the lift team for additional assistance." 4. "The two of us can lift the patient easily."

3

A patient has been hospitalized for the past 48 hours with a fever of unknown origin. His medical record indicates tympanic temperatures of 38.7° C (101.6° F) (0400), 36.6° C (97.9° F) (0800), 36.9° C (98.4° F) (1200), 37.6° C (99.6° F) (1600), and 38.3° C (100.9° F) (2000). How would you describe this pattern of temperature measurements? 1. Usual range of circadian rhythm measurements 2. Sustained fever pattern 3. Intermittent fever pattern 4. Resolving fever pattern

3

A patient presents in the clinic with dizziness and fatigue. The nursing assistant reports a slow but regular radial pulse of 44. What is your priority intervention? 1. Request that the nursing assistant repeat the pulse check 2. Call for a stat electrocardiogram (ECG) 3. Assess the patient's apical pulse and evidence of a pulse deficit 4. Prepare to administer cardiac-stimulating medications

3

As you are obtaining the oxygen saturation on a 19-year-old college student with severe asthma, you note that she has black nail polish on her nails. You remove the polish from one nail, and she asks you why her nail polish had to be removed. What is the best response? 1. Nail polish attracts microorganisms and contaminates the finger sensor. 2. Nail polish increases oxygen saturation. 3. Nail polish interferes with sensor function. 4. Nail polish creates excessive heat in sensor probe.

3

Musculoskeletal disorders are the most prevalent and debilitating occupational health hazards for nurses. To reduce the risk for these injuries, the American Nurses Association advocates which of the following? 1. Mandate that physical therapists do all patient transfers 2. Require adequate staffing levels in health care organizations 3. Require the use of assistive equipment and devices 4. Require an adequate number of staff to be involved in all patient transfers

3

The nurse observes a nursing student taking a blood pressure (BP) on a patient. The nurse notes that the student very slowly deflates the cuff in an attempt to hear the sounds. The patient's BP range over the past 24 hours is 132/64 to 126/72 mm Hg. Which of the following BP readings made by the student is most likely caused by an incorrect technique? 1. 96/40 mm Hg 2. 110/66 mm Hg 3. 130/90 mm Hg 4. 156/82 mm Hg

3

Which of the following indicates that additional assistance is needed to transfer the patient from the bed to the stretcher? 1. The patient is 5 feet 6 inches and weighs 120 lbs 2. The patient speaks and understands English. 3. The patient is returning to unit from recovery room after a procedure requiring conscious sedation. 4. The patient received analgesia for pain 30 minutes ago.

3

Which patient is at highest risk for tachycardia? 1. A healthy basketball player during warmup exercises 2. A patient admitted with hypothermia 3. A patient with a fever of 39.4° C (103° F) 4. A 90-year-old male taking beta blockers

3

The nurse needs to move a patient up in bed using a drawsheet. The nurse has another nurse helping. In which order will the nurses perform the steps, beginning with the first one? 1. Grasp the drawsheet firmly near the patient. 2. Move the patient and drawsheet to the desired position. 3. Position one nurse at each side of the bed. 4. Place the drawsheet under the patient from shoulder to thigh. 5. Place your feet apart with a forward-backward stance. 6. Flex knees and hips and on count of three shift weight from the front to back leg.

3, 4, 1, 5, 6, 2.

A nurse changed a patient's surgical wound dressing the day before and now prepares for another dressing change. The nurse had difficulty removing the gauze from the wound bed yesterday, causing the patient discomfort. Today he gives the patient an analgesic 30 minutes before the dressing change. Then he adds some sterile saline to loosen the gauze for a few minutes before removing it. The patient reports that the procedure was much more comfortable. Which of the following describes the nurse's approach to the dressing change? (Select all that apply) 1. Clinical inference. 2. Basic critical thinking. 3. Complex critical thinking. 4. Experience. 5. Reflection.

3, 4.

Which of the following is a principle of proper body mechanics when lifting or carrying objects? (Select all that apply.) 1. Keep the knees in a locked position. 2. Bend at the waist to maintain a center of gravity. 3. Maintain a wide base of support. 4. Hold objects away from the body for improved leverage. 5. Encourage patient to help as much as possible.

3, 5.

The nurse and a new nurse in orientation are caring for a patient with pneumonia. Which statement by the new nurse will indicate a correct understanding of this condition? a. "An infectious disease like pneumonia may not pose a risk to others." b. "We need to isolate the patient in a private negative-pressure room." c. "Clinical signs and symptoms are not present in pneumonia." d. "The patient will not be able to return home."

"An infectious disease like pneumonia may not pose a risk to others."

The patient and the nurse are discussing Rickettsia rickettsii—Rocky Mountain spotted fever. Which patient statement to the nurse indicates understanding regarding the mode of transmission for this disease? a. "When camping, I will use sunscreen." b. "When camping, I will drink bottled water." c. "When camping, I will wear insect repellent." d. "When camping, I will wash my hands with hand gel."

"When camping, I will wear insect repellent."

Develoment of an infection occurs in a cycle that depends on:

-An infectious agent or pathogen. -A reservoir or source. -A portal of exit. -A mode of transmission. -A portal of entry to a host. -A susceptible host.

Complex Critical Thinking

-Analyzes and examines choices independently. -Students learn to think beyond and synthesize knowledge.

Basic Critical Thinking

-Concrete thinking based off a set of rules. -Results from limited nursing knowledge and experience, as well as inadequate critical thinking experience.

Immobility disrupts normal metabolic functioning by:

-Decreasing metabolic rate. -Altering metabolism of carbs, fats and proteins causing fluid electrolyte and calcium imbalances. -Causing GI disturbances such as decreased appetite and slowing of peristalsis.

Psychosocial effects that occur with immobolization

-Emotional and behavioral responses. -Sensory alterations. -Changes in coping.

Professional Standards

-Ethical criteria for nursing judgments. -Evidence-based criteria used for evaluation. -Criteria for professional responsibility.

Localized infection

-Ex. a wound infection. -Patient experiences pain, tenderness, warmth, redness at the wound site.

Intellecutal Standards

-Guidelines for rational thought. Ensures you obtain a complete database of information. Clear, precise, specific, accurate, relevant, plausible, consistent, logical, deep, broad, complete, significant, adequate, and fair.

Variation in Blood Pressure Older Adult

-Increased Systolic blood pressure. -Diastolic stays the same.

Airborne precautions require

-Negative air flow room "airborne infection isolation room." -N95 mask

Prodromal period

-Short period after incubation; early, mild symptoms..

Droplet precautions require

-Surgical mask when within 3 feet of the patient. -Proper hand hygiene. -Dedicated care equipment.

Inflammation

-The body's response to injury, infection or irritation. -A protective vascular reaction that delivers fluid, blood products, and nutrients to area of injury.

Concepts of a Critical Thinker

-Truth seeking. -Open mindedness. -Analyze potentially problematic situations. -Systematically be organized and focused. -Self confidence: trust in your own reasoning process. -Be eager to learn. -Maturity: reflect on your own judgements.

Complications of immobility in relation to the urinary system

-Urinary stasis: renal pelvis fills before urine enters the ureters. -Renal calculi: kidney stones

Sites of Health Care-Associated Infections

-Urinary tract. -Surgical or traumatic wounds. -Respiratory tract. -Bloodstream.

A patient is isolated for pulmonary tuberculosis. The nurse notes that the patient seems to be angry, but he knows that this is a normal response to isolation. Which is the best intervention? 1. Provide a dark, quiet room to calm the patient. 2. Reduce the level of precautions to keep the patient from becoming angry. 3. Explain the reasons for isolation procedures and provide meaningful stimulation. 4. Limit family and other caregiver visits to reduce the risk of spreading the infection.

3.

What is the most effective way to control transmission of infection? 1. Isolation precautions 2. Identifying the infectious agent 3. Hand hygiene practices 4. Vaccinations

3.

The nursing assistive personnel (NAP) informs you that the electronic blood pressure machine on the patient who has recently returned from surgery following removal of her gallbladder is flashing a blood pressure of 65/46 and alarming. Place your care activities in priority order. 1. Press the start button of the electronic blood pressure machine to obtain a new reading. 2. Obtain a manual blood pressure with a stethoscope. 3. Check the patient's pulse distal to the blood pressure cuff. 4. Assess the patient's mental status. 5. Remind the patient not to bend her arm with the blood pressure cuff.

4, 1, 3, 2, 5.

A patient's surgical wound has become swollen, red, and tender. The nurse notes that the patient has a new fever, purulent wound drainage, and leukocytosis. Which interventions would be appropriate and in what order? 1. Notify the health care provider of the patient's status. 2. Reassure the patient and recheck the wound later. 3. Support the patient's fluid and nutritional needs. 4. Use aseptic technique to change the dressing.

4, 2, 1, 3.

Place the steps of the scientific method in their correct order with number 1 being the first step of the process. 1. Formulate a question or hypothesis. 2. Evaluate results of the study. 3. Collect data. 4. Identify the problem. 5. Test the question or hypothesis.

4, 3, 1, 5, 2.

White, shiny, flexible bands of fibrous tissue binding joints together and connecting various bones and cartilage types are known as: 1. Joints 2. Muscles 3. Tendons 4. Ligaments

4.

Your assigned patient has a leg ulcer that has a dressing on it. During your assessment you find that the dressing is saturated with purulent drainage. Which action would be best on your part? 1. Reinforce dressing with a clean, dry dressing and call the health care provider. 2. Remove wet dressing and apply new dressing using sterile procedure. 3. Put on gloves before removing the old dressing; then obtain a wound culture. 4. Remove saturated dressing with gloves, remove gloves, then perform hand hygiene and apply new gloves before putting on a clean dressing.

4.

WBC Normal Values

5000-10,000. -Increased in acte infection. -Decreased in certain viral or overwhelming infections.

Pulse range: School-age children

75-100

Pulse range: Preschoolers

80-110

Pulse range: Toddler

90-140

When taking the pulse of an infant, the nurse notices that the rate is 145 beats/min and the rhythm is regular. How should the nurse interpret this finding? a. This is normal for an infant. b. This is too fast for an infant. c. This is too slow for an infant. d. This is not a rate for an infant but for a toddler.

A

The nurse is caring for a group of medical-surgical patients. Which patient is most at risk for developing an infection? a. A patient who is in observation for chest pain b. A patient who has been admitted with dehydration c. A patient who is recovering from a right total hip surgery d. A patient who has been admitted for stabilization of heart problems

A patient who is recovering from a right total hip surgery

. The nurse is caring for a group of patients. Which patient will the nurse see first? a. A patient with Clostridium difficile in droplet precautions b. A patient with tuberculosis in airborne precautions c. A patient with MRSA infection in contact precautions d. A patient with a lung transplant in protective environment precautions

A patient with Clostridium difficile in droplet precautions

A nurse is providing care to a group of patients. Which patient will the nurse see first? a. A patient with a hip replacement on prolonged bed rest reporting chest pain and dyspnea b. A bedridden patient who has a reddened area on the buttocks who needs to be turned c. A patient on bed rest who has renal calculi and needs to go to the bathroom d. A patient after knee surgery who needs range of motion exercises

A patient with a hip replacement on prolonged bed rest reporting chest pain and dyspnea

A nurse is supervising the logrolling of a patient. To which patient is the nurse most likely providing care? a. A patient with neck surgery b. A patient with hypostatic pneumonia c. A patient with a total knee replacement d. A patient with a Stage IV pressure ulcer

A patient with neck surgery

Mobility

A person's ability to move about freely

Reservior

A place wher a pathogen survives.

A 72-year-old male patient comes to the health clinic for an annual follow-up. The nurse enters the patient's room and notices him to be diaphoretic, holding his chest and breathing with difficulty. The nurse immediately checks the patient's heart rate and blood pressureand ask him, "Tell me where your pain is." Which of the following assessment approaches does this scenario describe? 1. Review of systems approach. 2. Use of a structured database format. 3. Back channeling. 4. A problem-solving approach.

A problem-solving approach.

Review of Systems

A systematic approach for collecting the patient's self-reported data on all body systems.

Bacteriocidal

A temperature or chemical that destroys bacteria.

The nurse is caring for a patient with a spinal cord injury and notices that the patient's hips have a tendency to rotate externally when the patient is supine. Which device will the nurse use to help prevent injury secondary to this rotation? a. Hand rolls b. A trapeze bar c. A trochanter roll d. Hand-wrist splints

A trochanter roll

Portal of Exit

A way for the infectious agent to escape from the reservoir in which it has been growing

The nurse is caring for a patient who has had a recent stroke and is paralyzed on the left side. The patient has no respiratory or cardiac issues but cannot walk. The patient cannot button a shirt and cannot feed self due to being left-handed and becomes frustrated very easily. The patient has been eating very little and has lost 2 lbs. The patient asks the nurse, "How can I go home like this? I'm not getting better." Which health care team members will the nurse need to consult? (Select all that apply.) a. Dietitian b. Physical therapist c. Respiratory therapist d. Cardiac rehabilitation therapist e. Occupational therapist f. Psychologist

A, B, E, F.

A nurse writes the following outcomes for a patient who has chronic obstructive pulmonary disease to improve activity level: Diastolic blood pressure will remain below 70 mm Hg with systolic below 130 mm Hg. Resting heart rate will range between 65 and 75. The last goal is that the patient will exercise 3 times a week. Which evaluative findings indicate successful goal achievement? (Select all that apply.) a. Resting heart rate 70 b. Blood pressure 126/64 c. Blood pressure 140/90 d. Reports doing stretching and flexibility exercises 2 times this week e. Reports doing resistive training 1 time and aerobics 2 times this week

A, B, E.

The patient is being encouraged to purchase a portable automatic blood pressure device to monitor blood pressure at home. Which information will the nurse present as benefits for this type of treatment? (Select all that apply.) a. Patients can actively participate in their treatment. b. Self-monitoring helps with compliance and treatment. c. The risk of obtaining an inaccurate reading is decreased. d. Blood pressures can be obtained if pulse rates become irregular. e. Patients can provide information about patterns to health care providers.

A, B, E.

Which skills must a patient with a new colostomy be taught before discharge from the hospital? A. How to change the pouch B. How to empty the pouch C. How to open and close the pouch D. How to irrigate the colostomy E. How to determine if the ostomy is healing appropriately

A. How to change the pouch B. How to empty the pouch C. How to open and close the pouch E. How to determine if the ostomy is healing appropriately The patient must be able to do these tasks to successfully manage his or her colostomy when going home.

The nurse is caring for a patient who needs oxygen via a nasal cannula. Which task can the nurse delegate to the nursing assistive personnel? a. Applying the nasal cannula b. Adjusting the oxygen flow c. Assessing lung sounds d. Setting up the oxygen

ANS: A The skill of applying (not adjusting oxygen flow) a nasal cannula or oxygen mask can be delegated to nursing assistive personnel (NAP). The nurse is responsible for assessing the patient's respiratory system, response to oxygen therapy, and setup of oxygen therapy, including adjustment of oxygen flow rate.

Which are key points that the nurse should include in patient education for a person with complaints of chronic constipation? A. Increase fiber and fluids in the diet B. Use a low-volume enema daily C. Avoid gluten in the diet D. Take laxatives twice a day E. Exercise for 30 minutes every day F. Schedule time to use the toilet at the same time every day G. Take probiotics 5 times a week

A. Increase fiber and fluids in the diet E. Exercise for 30 minutes every day F. Schedule time to use the toilet at the same time every day These steps are the initial ones to take to resolve chronic problems with constipation before considering regular laxative or enema use.

What should the nurse teach a young woman with a history of urinary tract infections about UTI prevention? A. Keep the bowels regular. B. Limit water intake to 1-2 glasses a day C. Wear cotton underwear D. Cleanse the perineum from front to back. E. Practice pelvic muscle exercise (Kegel) daily.

A. Keep the bowels regular. C. Wear cotton underwear D. Cleanse the perineum from front to back. All are interventions that lead to healthy bladder habits. Adequate hydration will ensure that the bladder is regularly flushed out and will help prevent a UTI. Pelvic muscle exercises promote pelvic health but not necessarily prevent UTI.

The patient with cardiovascular disease is receiving dietary instructions from the nurse. Which information from the patient indicates teaching is successful? a. Maintain a prescribed carbohydrate intake. b. Eat fish at least 5 times per week. c. Limit trans fat to less than 1%. d. Avoid high-fiber foods.

ANS: C American Heart Association guidelines recommend limiting saturated fat to less than 7%, trans fat to less than 1%, and cholesterol to less than 300 mg/day. Diet therapy includes eating fish at least 2 times per week and eating whole grain high-fiber foods. Maintaining a prescribed carbohydrate intake is necessary for diabetes mellitus.

The nurse will irrigate a patient's nasogastric (NG) tube. Which action should the nurse take? a. Instill solution into pigtail slowly. b. Check placement after instillation of solution. c. Immediately aspirate after instilling fluid. d. Prepare 60 mL of tap water into Asepto syringe.

ANS: C After instilling saline, immediately aspirate or pull back slowly on syringe to withdraw fluid. Do not introduce saline through blue "pigtail" air vent of Salem sump tube. Checking placement before instillation of normal saline prevents accidental entrance of irrigating solution into lungs. Draw up 30 mL of normal saline into Asepto syringe to minimize loss of electrolytes from stomach fluids.

The nurse is caring for a patient with a Stage IV pressure ulcer. Which nursing diagnosis does the nurse add to the care plan? a. Readiness for enhanced nutrition b. Impaired physical mobility c. Impaired skin integrity d. Chronic pain

ANS: C After the assessment is completed and the information that the patient has a Stage IV pressure ulcer is gathered, a diagnosis of Impaired skin integrity is selected. Readiness for enhanced nutrition would be selected for an individual with an adequate diet that could be improved. Impaired physical mobility and Chronic pain do not support the current data in the question.

A nurse is planning care for a group of patients. Which task will the nurse assign to the nursing assistive personnel? a. Obtaining a midstream urine specimen b. Interpreting a bladder scan result c. Inserting a straight catheter d. Irrigating a catheter

ANS: A The skill of collecting midstream (clean-voided) urine specimens can be delegated to nursing assistive personnel. The nurse must first determine the timing and frequency of the bladder scan measurement and interprets the measurements obtained. Inserting a straight or an indwelling catheter cannot be delegated. Catheter irrigation or instillation cannot be delegated to nursing assistive personnel.

The nurse is planning care for a group of patients. Which task will the nurse assign to the nursing assistive personnel? a. Measuring capillary blood glucose level b. Measuring nasoenteric tube for insertion c. Measuring pH in gastrointestinal aspirate d. Measuring the patient's risk for aspiration

ANS: A The skill of measuring blood glucose level after skin puncture (capillary puncture) can be delegated to nursing assistive personnel. The other skills cannot be delegated. A nurse must measure a nasoenteric tube for insertion, pH in gastrointestinal aspirate, and patient's risk for aspiration.

A patient develops a foodborne disease from Escherichia coli. When taking a health history, which food item will the nurse most likely find the patient ingested? a. Improperly home-canned food b. Undercooked ground beef c. Soft cheese d. Custard

ANS: B Undercooked ground beef is the usual food source for Escherichia coli. Botulism is associated with improperly home-canned foods. Soft cheese is the usual food source for listeriosis. Custards are associated with salmonellosis and Staphylococcus.

The nurse is completing a skin risk assessment using the Braden Scale. The patient has slight sensory impairment, has skin that is rarely moist, walks occasionally, and has slightly limited mobility, along with excellent intake of meals and no apparent problem with friction and shear. Which score will the nurse document for this patient? a. 15 b. 17 c. 20 d. 23

ANS: C With use of the Braden Scale, the total score is a 20. The patient receives 3 for slight sensory perception impairment, 4 for skin being rarely moist, 3 for walks occasionally, 3 for slightly limited mobility, 4 for intake of meals, and 4 for no problem with friction and shear.

A nurse is providing care to a group of patients. Which patient will the nurse see first? a. A child about to receive a normal saline enema b. A teenager about to receive loperamide for diarrhea c. An older patient with glaucoma about to receive an enema d. A middle-aged patient with myocardial infarction about to receive docusate sodium

ANS: C An enema is contradicted in a patient with glaucoma; this patient should be seen first. All the rest are expected. A child can receive normal saline enemas since they are isotonic. Loperamide, an antidiarrheal, is given for diarrhea. Docusate sodium is given to soften stool for patients with myocardial infarction to prevent straining.

The nurse reviews the laboratory results for a patient and determines the viscosity of the blood is thick. Which laboratory result did the nurse check? a. Arterial blood gas b. Blood culture c. Hematocrit d. Potassium

C

The patient is being admitted to the emergency department following a motor vehicle accident. The patient's jaw is broken with several broken teeth. The patient is ashen, has cool skin, and is diaphoretic. Which route will the nurse use to obtain an accurate temperature reading? a. Oral b. Axillary c. Tympanic d. Temporal

C

The patient is found to be unresponsive and not breathing. Which pulse site will the nurse use? a. Radial b. Apical c. Carotid d. Brachial

C

The patient reports being tired and weak and lacks energy. Upon assessment, the nurse finds that patient has gained weight, and blood pressure and pulse are elevated after climbing stairs. Which nursing diagnosis will the nurse add to the care plan? a. Fatigue b. Ineffective coping c. Activity intolerance d. Decreased cardiac output

C

When taking an adult blood pressure, the onset of the sound the nurse hears is at 138, the muffled sound the nurse hears is at 70, and the disappearance of the sound the nurse hears is at 62. How should the nurse record this finding? a. 68 b. 76 c. 138/62 d. 138/70

C

What is the correct sequence for suctioning a patient?1. Open kit and basin.2. Apply gloves.3. Lubricate catheter.4. Verify functioning of suction device and pressure.5. Connect suctioning tubing to suction catheter.6. Increase supplemental oxygen.7. Reapply oxygen.8. Suction airway. A. 6, 4, 3, 1, 2, 5, 8, 7 B. 4, 6, 1, 2, 3, 8, 5, 7 C. 4, 6, 1, 3, 2, 5, 8, 7 D. 6, 4, 1, 3, 2, 5, 7, 8

C. 4, 6, 1, 3, 2, 5, 8, 7 These steps allow for smooth completion of procedure while helping to maintain patient's level of oxygenation.

A nurse is watching a nursing assistive personnel (NAP) perform a postvoid bladder scan on a female with a previous hysterectomy. Which action will require the nurse to follow up? a. Palpates the patient's symphysis pubis b. Wipes scanner head with alcohol pad c. Applies a generous amount of gel d. Sets the scanner to female

ANS: D The nurse will follow up if the NAP sets the scanner to female. Women who have had a hysterectomy should be designated as male. All the rest are correct and require no follow-up. The NAP should palpate the symphysis pubis, the scanner head should be cleaned with an alcohol pad, and a generous amount of gel should be applied.

The nurse is teaching the patient to obtain a specimen for fecal occult blood testing using fecal immunochemical (FIT) testing at home. How does the nurse instruct the patient to collect the specimen? A. Three fecal smears from one bowel movement B. One fecal smear from an early-morning bowel movement C. One fecal smear from three separate bowel movements D. Three fecal smears when blood can be seen in the bowel movement

C. One fecal smear from three separate bowel movements Samples from three separate bowel movements decrease the risk of a false-negative or a false-positive result.

A nurse is evaluating care of an immobilized patient. Which action will the nurse take? a. Focus on whether the interdisciplinary team is satisfied with the care. b. Compare the patient's actual outcomes with the outcomes in the care plan. c. Involve primarily the patient's family and health care team to determine goal achievement. d. Use objective data solely in determining whether interventions have been successful.

Compare the patient's actual outcomes with the outcomes in the care plan.

Data Validation

Comparison of data with another source to determine accuracy

The nurse is caring for a patient in labor and delivery. When near completing an assessment of the patient's cervix, the electronic infusion device being used on the intravenous (IV) infusion alarms. Which sequence of actions is most appropriate for the nurse to take? a. Complete the assessment, remove gloves, and silence the alarm. b. Discontinue the assessment, silence the alarm, and assess the intravenous site. c. Complete the assessment, remove gloves, wash hands, and assess the intravenous infusion. d. Discontinue the assessment, remove gloves, use hand gel, and assess the intravenous infusion.

Complete the assessment, remove gloves, wash hands, and assess the intravenous infusion.

The nurse is using critical thinking skills during the first phase of the nursing process. Which action indicates the nurse is in the first phase? a. Completes a comprehensive database b. Identifies pertinent nursing diagnoses c. Intervenes based on priorities of patient care d. Determines whether outcomes have been achieved

Completes a comprehensive database

A nursing instructor needs to evaluate students' abilities to synthesize data and identify relationships between nursing diagnoses. Which learning assignment is best suited for this instructor's needs? a. Concept mapping b. Reflective journaling c. Lecture and discussion d. Reading assignment with a written summary

Concept mapping

The nurse bathes the patient who has a fever with cool water. The nurse does this to increase heat loss by means of: 1. Radiation. 2. Convection. 3. Conduction. 4. Condensation.

Conduction

The patient has a temperature of 105.2° F. The nurse is attempting to lower temperature by providing tepid sponge baths and placing cool compresses in strategic body locations. Which technique is the nurse using to lower the patient's temperature? a. Radiation b. Conduction c. Convection d. Evaporation

Conduction

Tendons

Connect muscle to bone

Joints

Connections between bones. cartilaginous, fibrous, and synovial.

While interviewing an older female patient of Asian descent, the nurse notices that the patient looks at the ground when answering questions. What should the nurse do? a. Consider cultural differences during this assessment. b. Ask the patient to make eye contact to determine her affect. c. Continue with the interview and document that the patient is depressed. d. Notify the health care provider to recommend a psychological evaluation.

Consider cultural differences during this assessment.

Which action should the nurse take when using critical thinking to make clinical decisions? a. Make decisions based on intuition. b. Accept one established way to provide care. c. Consider what is important in a given situation. d. Read and follow the heath care provider's orders.

Consider what is imporatnt in a given situation.

Which behavior indicates the nurse is using a team approach when caring for a patient who is experiencing alterations in mobility? a. Delegates assessment of lung sounds to nursing assistive personnel b. Becomes solely responsible for modifying activities of daily living c. Consults physical therapy for strengthening exercises in the extremities d. Involves respiratory therapy for altered breathing from severe anxiety levels

Consults physical therapy for strengthening exercises in the extremities

Vehicles modfe of transmission

Contaminated items

Skeletal muscle

Contract and relax. The working elements of movement.

The nurse is preparing to assess the blood pressure of a 3-year-old. How should the nurse proceed? a. Use the diaphragm portion of the stethoscope to detect Korotkoff sounds. b. Obtain the reading before the child has a chance to "settle down." c. Choose the cuff that says "Child" instead of "Infant." d. Explain the procedure to the child.

D

The nursing assistive personnel (NAP) is taking vital signs and reports that a patient's blood pressure is abnormally low. What should the nurse do next? a. Ask the NAP retake the blood pressure. b. Instruct the NAP to assess the patient's other vital signs. c. Disregard the report and have it rechecked at the next scheduled time. d. Retake the blood pressure personally and assess the patient's condition.

D

The patient requires routine temperature assessment but is confused, easily agitated, and has a history of seizures. Which route will the nurse use to obtain the patient's temperature? a. Oral b. Rectal c. Axillary d. Tympanic

D

When assessing the blood pressure of a school aged child, using an adult cuff of normal size will affect the reading and produce a value that is: A. Accurate. B. Indistinct. C. Falsely low. D. Falsely high.

D

You have delegated vital signs to assistive personnel. The assistant informs you that the patient has just finished a bowl of hot soup. The nurse's most appropriate advice would be to A. Take a rectal temperature. B. Take the oral temperature as planned. C. Advise the patient to drink a glass of cold water. D. Wait 30 minutes and take an oral temperature

D

Commitment Critical Thinking

Anticipates when to make choices without assistance from others and accepts accountability for decisions made.

The patient has the nursing diagnosis of Impaired physical mobility related to pain in the left shoulder. Which priority action will the nurse take? a. Encourage the patient to do self-care. b. Keep the patient as mobile as possible. c. Encourage the patient to perform ROM. d. Assist the patient with comfort measures.

Assist the patient with comfort measures.

The patient is admitted to a skilled care unit for rehabilitation after the surgical procedure of fixation of a fractured left hip. The patient's nursing diagnosis is Impaired physical mobility related to musculoskeletal impairment from surgery and pain with movement. The patient is able to use a walker but needs assistance ambulating and transferring from the bed to the chair. Which nursing intervention is most appropriate for this patient? a. Obtain assistance and physically transfer the patient to the chair. b. Assist with ambulation and measure how far the patient walks. c. Give pain medication after ambulation so the patient will have a clear mind. d. Bring the patient to the cafeteria for group instruction on ambulation.

Assist with ambulation and measure how far the patient walks.

The nurse is developing a plan of care for a patient diagnosed with activity intolerance. Which strategy will the nurse use to provide the best chance of maintaining patient compliance? a. Performing 20 minutes of aerobic exercise 7 days a week with 10-minute warm-up and cool-down periods b. Instructing the patient to use an exercise log to record day, time, duration, and responses to exercise activity c. Stressing the harm of not exercising by getting the patient to take responsibility for current health status d. Arranging for the patient to join a gym that takes self-pay rather than insurance

B

The nurse is preparing to obtain an oxygen saturation reading on a toddler. Which action will the nurse take? a. Secure the sensor to the toddler's earlobe. b. Determine whether the toddler has a latex allergy. c. Place the sensor on the bridge of the toddler's nose. d. Overlook variations between an oximeter pulse rate and the toddler's pulse rate.

B

The nurse is preparing to transfer an uncooperative patient who does not have upper body strength. Which piece of equipment will be best for the nurses to obtain? a. Drawsheet b. Full body sling c. Overhead trapeze d. Friction-reducing slide sheet

B

The nurse is teaching a patient how to use a cane. Which information will the nurse include in the teaching session? a. Place the cane at the top of the hip bone. b. Place the cane on the stronger side of the body. c. Place the cane in front of the body and then move the good leg. d. Place the cane 10 to 15 inches in front of the body when walking.

B

The patient is admitted with shortness of breath and chest discomfort. Which laboratory value could account for the patient's symptoms? a. Red blood cell count of 5.0 million/mm3 b. Hemoglobin level of 8.0 g/100 mL c. Hematocrit level of 45% d. Pulse oximetry of 95%

B

The patient is being admitted to the emergency department with reports of shortness of breath. The patient has had chronic lung disease for many years but still smokes. What will the nurse do? a. Allow the patient to breathe into a paper bag. b. Use oxygen cautiously in this patient. c. Administer high levels of oxygen. d. Give CO2 via mask.

B

The patient is brought to the emergency department with possible injury to the left shoulder. Which area will the nurse assess to best determine joint mobility? a. The patient's gait b. The patient's range of motion c. The patient's ethnic influences d. The patient's fine-motor coordination

B

The patient requires temperatures to be taken every 2 hours. Which task will the nurse assign to an RN? a. Using appropriate route and device b. Assessing changes in body temperature c. Being aware of the usual values for the patient d. Obtaining temperature measurement at ordered frequency

B

The patient was found unresponsive in an apartment and is being brought to the emergency department. The patient has arm, hand, and leg edema, temperature is 95.6° F, and hands are cold secondary to a history of peripheral vascular disease. It is reported that the patient has a latex allergy. What should the nurse do to quickly measure the patient's oxygen saturation? a. Attach a finger probe to the patient's index finger. b. Place a nonadhesive sensor on the patient's earlobe. c. Attach a disposable adhesive sensor to the bridge of the patient's nose. d. Place the sensor on the same arm that the electronic blood pressure cuff is on.

B

When assessing the temperature of newborns and children, the nurse decides to utilize a temporal artery thermometer. What is the rationale for the nurse's action? a. It is not affected by skin moisture. b. It has no risk of injury to patient or nurse. c. It reflects rapid changes in radiant temperature. d. It is accurate even when the forehead is covered with hair.

B

You notice that a teenager has an irregular pulse. The best action you should take includes: A. Reading the history and physical. B. Assessing the apical pulse rate for 1 full minute. C. Auscultating for strength and depth of pulse. D. Asking whether the patient feels any palpitations or faintness of breath.

B

There is no urine when a catheter is inserted into a female's urethra. What should the nurse do next? A. Remove the catheter and start all over with a new kit and catheter. B.Leave the catheter there and start over with a new catheter. C. Pull the catheter back and re-insert at a different angle. D.Ask the patient to bear down and insert the catheter further.

B. Leave the catheter there and start over with a new catheter. The catheter may be in the vagina, leave the catheter in the vagina as landmark indicating where not to insert, and insert another sterile catheter. Pulling the catheter back and re-inserting is poor technique increasing the risk for CAUTI.

A patient is admitted to the emergency department with suspected carbon monoxide poisoning. Even though her color is ruddy not cyanotic, the nurse understands the patient is at a risk for decreased oxygen-carrying capacity of blood because carbon Monoxide does which of the following: A. Stimulates hyperventilation causing respiratory alkalosis B. Forms a strong bond with hemoglobin thus preventing oxygen binding in the lungs C. Stimulates hypoventilation causing respiratory acidosis D. Causes alveoli to overinflate leading to atelectasis

B. Forms a strong bond with hemoglobin thus preventing oxygen binding in the lungs Carbon monoxide strongly binds to hemoglobin making it unavailable for oxygen binding and transport.

The nurse is caring for a patient who has decreased mobility. Which intervention is a simple and cost-effective method for reducing the risks of pulmonary complication? A. Antibiotics B. Frequent change of position C. Oxygen humidification D. Chest physiotherapy

B. Frequent change of position Movement not only mobilizes secretions but helps strengthen respiratory muscles by impacting the effectiveness of gas exchange processes.

What nursing intervention decreases the risk for catheter associated urinary tract infection (CAUTI)? A. Cleanse the urinary meatus 3-4 times daily with antiseptic solution. B. Hang the urinary drainage bag below the level with the bladder. C. Empty the urinary drainage bag daily. D. Irrigate the urinary catheter with sterile water.

B. Hang the urinary drainage bag below the level with the bladder. Evidenced based interventions shown to decrease the risk for CAUTI include ensuring that there is a free flow of urine from the catheter to the drainage bag. Empty the urinary drainage bag daily.

For a patient who has a muscle sprain, localized hemorrhage, or hematoma, which wound care product helps prevent edema formation, control bleeding, and anesthetize the body part? A. Binder B. Ice bag C. Elastic bandage D. Absorptive dressing

B. Ice bag An ice bag helps to constrict excess fluid in tissues, which prevents edema. The blood vessels become constricted, help to control bleeding, and can decrease pain where the ice bag is placed.

Which of the following skills can be delegated to nursing assistive personnel (NAP)? A. Nasotracheal suctioning B. Oropharyngeal suctioning of a stable patient C. Suctioning a new artificial airway D. Permanent tracheostomy tube suctioning E. Care of an endotracheal tube (ETT)

B. Oropharyngeal suctioning of a stable patient D. Permanent tracheostomy tube suctioning Oropharyngeal suctioning of a stable patient and permanent tracheostomy tube suctioning may be safely delegated to a NAP. The other skills require nursing assessment and clinical decision making as the skill progresses.

The patient has a risk for skin impairment and has a 15 on the Braden Scale upon admission. The nurse has implemented interventions. Upon reassessment, which Braden score will be the best sign that the risk for skin breakdown is removed? a. 12 b. 13 c. 20 d. 23

ANS: D The best sign is a perfect score of 23. The Braden Scale is composed of six subscales: sensory perception, moisture, activity, mobility, nutrition, and friction and shear. The total score ranges from 6 to 23, and a lower total score indicates a higher risk for pressure ulcer development. The cutoff score for onset of pressure ulcer risk with the Braden Scale in the general adult population is 18.

A nurse is teaching a patient about proteins that must be obtained through the diet and cannot be synthesized in the body. Which term used by the patient indicates teaching is successful? a. Amino acids b. Triglycerides c. Dispensable amino acids d. Indispensable amino acids

ANS: D The body does not synthesize indispensable amino acids, so these need to be provided in the diet. The simplest form of protein is the amino acid. The body synthesizes dispensable amino acids. Triglycerides are made up of three fatty acids attached to a glycerol.

The nurse is teaching a health class about the gastrointestinal tract. The nurse will explain that which portion of the digestive tract absorbs most of the nutrients? a. Ileum b. Cecum c. Stomach d. Duodenum

ANS: D The duodenum and jejunum absorb most nutrients and electrolytes in the small intestine. The ileum absorbs certain vitamins, iron, and bile salts. Food is broken down in the stomach. The cecum is the beginning of the large intestine.

The nurse is working on an orthopedic rehabilitation unit that requires lifting and positioning of patients. Which personal injury will the nurse most likely try to prevent? a. Arm b. Hip c. Back d. Ankle

Back

Pyrogens

Bacteria and viruses that elevate body temperature

Aerobic bacteria

Bacteria that require oxygen for survival

Anaerobic bacteria

Bacteria that thrive with little or no free oxygen

A nurse prepares to insert a Foley catheter. The procedure manual calls for the patient to lie in the dorsal recumbent position. The patient complains of having back pain when lying on her back. Despite this, the nurse positions the patient supine with knees flexed as the manual recommends and begins to indert the catheter. This is an example of: A. Accuracy. B. Reflection. C. Risk taking. D. Basic critical thinking.

Basic critical thinking.

Levels of Critical Thinking in Nursing

Basic, Complex, Commitment

Perserverance

Be cautious of an easy answer; look for a pattern and find a solution.

Risk Taking

Be willing to recommend alternative approaches to Nursing Care.

The nurse is caring for a group of patients. Which patient will the nurse see first? a. A patient with chronic obstructive pulmonary disease doing stretching exercises b. A patient with diabetes mellitus carrying hard candy while doing exercises c. A patient with a heart attack doing isometric exercises d. A patient with hypertension doing Tai Chi exercises

C

The nurse is caring for a small child and needs to obtain vital signs. Which site choice from the nursing assistive personnel (NAP) will cause the nurse to praise the NAP? a. Ulnar site b. Radial site c. Brachial site d. Femoral site

C

The nurse is evaluating care of a patient for crutches. Which finding indicates a successful outcome? a. The top of the crutch is three to four finger widths from the armpit. b. The elbows are slightly flexed at 30 to 35 degrees when the patient is standing. c. The tip of the crutch is 4 to 6 inches anterior to the front of the patient's shoes. d. The position of the handgrips allows the axilla to support the patient's body weight.

C

The nurse is providing a blood pressure clinic for the community. Which group will the nurse most likely address? a. Non-Hispanic Caucasians b. European Americans c. African-Americans d. Asian Americans

C

The nurse evaluates which laboratory values to assess a patient's potential for wound healing? A.Fluid status B.Potassium C.Lipids D.Nitrogen balance

D. Nitrogen balance Nitrogen balance is important to determining serum protein status. A negative nitrogen balance is present when catabolic states exist. When a patient has a decreased protein level, he or she is at risk for delayed wound healing.

A patient continues to report postsurgical incision pain at a level of 9 out of 10 after pain medicine is given. The next dose of pain medicine is not due for another hour. What should the critically thinking nurse do first? a. Explore other options for pain relief. b. Discuss the surgical procedure and reason for the pain. c. Explain to the patient that nothing else has been ordered. d. Offer to notify the health care provider after morning rounds are completed.

Explore other options for pain relief.

Celcius to Fareinheit

F= (9/5 x C) + 32

The nurse is providing teaching to an immobilized patient with impaired skin integrity about diet. Which diet will the nurse recommend? a. High protein, high calorie b. High carbohydrate, low fat c. High vitamin A, high vitamin E d. Fluid restricted, bland

High protein, high calorie

Which action demonstrates a nurse utilizing reflection to improve clinical decision making? a. Obtains data in an orderly fashion. b. Uses an objective approach in patient situations. c. Improves a plan of care while thinking back on interventions effectiveness. d. Provides evidence-based explanations and research for care of assigned patients.

Improves a plan of care while thinking back on interventions effectiveness.

A nurse who is caring for a patient with a pressure ulcer applies the recommended dressing according to hospital policy. Which standard is the nurse following? a. Fairness b. Intellectual standards c. Independent reasoning d. Institutional practice guidelines

Institutional practice guidelines

The nurse is caring for a patient who has cultured positive for Clostridium difficile. Which action will the nurse take next? a. Instruct assistive personnel to use soap and water rather than sanitizer. b. Wear an N95 respirator when entering the patient room. c. Place the patient on droplet precautions. d. Teach the patient cough etiquette.

Instruct assistive personnel to use soap and water rather than sanitizer.

Vector mode of transmission

Internal and external transmissions

The patient appears to be in no apparent distress, but vital signs taken by assistive personnel reveal an extremely low pulse. The nurse then auscultates an apical pulse and asks the patient whether there is any history of heart problems. The nurse is utilizing which critical thinking skill? a. Evaluation b. Explanation c. Interpretation d. Self-regulation

Interpretation

Convalescence period

Interval when acute symptoms of infection disappear.

Illness stage

Interval when patient manifests signs and symptoms specific to type of infection

The nursing process organizes your approach to delivering nursing care. To provide care to your patients, you will need to incorporate nursing process and: A. Decision making. B. Problem solving. C. Interview Process D. Intellectual Standards.

Interview process

The nurse is caring for a patient with leukemia and is preparing to provide fluids through a vascular access (IV) device. Which nursing intervention is a priority in this procedure? a. Review the procedure with the patient. b. Position the patient comfortably. c. Maintain surgical aseptic technique. d. Gather available supplies.

Maintain surgical aseptic technique.

Hand rolls

Maintain the thumb in slight adduction and in opposition to the fingers

Two patient deaths have occurred on a medical unit in the last month. The staff notices that everyone feels pressured and team members are getting into more arguments. As a nurse on the unit, what will best help you manage this stress? A. Keep a journal B. Participate in a unit meeting to discuss feelings about the patient deaths C. Ask the nurse manager to assign you to less difficult patients D. Review the policy and procedure manual on proper care of patients after death

Participate in a unit meeting to discuss feelings about the patient deaths

Gait

Particuar manner or style of walking. Mechanics involve coordination of skeletal, neurological, and muscular systems.

Carriers

Persons who show no symptoms of illness but who have the pathogens that are transferred to others

Heat exhaustion

Profuse diaphoresis (sweating) with excess water and electrolyte loss

Disuse atrophy

Prolonged inactivity causes muscles to shrink in mass. Result from bedrest, trauma, casting, or local nerve damage.

The nurse puts elastic stockings on a patient following major abdominal surgery. The nurse teaches the patient that the stockings are used after a surgical procedure to __________________________.

Promotes venous return to the heart

The nurse on the surgical team and the surgeon have completed a surgery. After donning gloves, gathering instruments, and placing in the transport carrier, what is the next step in handling the instruments used during the procedure? a. Sending to central sterile for cleaning and sterilization b. Sending to central sterile for cleaning and disinfection c. Sending to central sterile for cleaning and boiling d. Sending to central sterile for cleaning

Sending to central sterile for cleaning and sterilization

The information obtained in an review of systems is: 1. Objective. 2. Subjective. 3. Based on the nurse's perspective. 4. Based on physical examination findings.

Subjective

A patient expresses fear of going home and being alone. Vital signs are stable and the incision is nearly completely healed. What can the nurse infer from the subjective data? a. The patient can now perform the dressing changes without help. b. The patient can begin retaking all of the previous medications. c. The patient is apprehensive about discharge. d. The patient's surgery was not successful.

The patient is apprehensive about discharge.

Sims position

The patient is lying on the left side with the right knee and thigh drawn up with the left arm placed along the back

Which goal is most appropriate for a patient who has had a total hip replacement? a. The patient will ambulate briskly on the treadmill by the time of discharge. b. The patient will walk 100 feet using a walker by the time of discharge. c. The nurse will assist the patient to ambulate in the hall 2 times a day. d. The patient will ambulate by the time of discharge.

The patient will walk 100 feet using a walker by the time of discharge.

The nurse is interviewing a patient with a hearing deficit. Which area should the nurse use to conduct this interview? a. The patient's room with the door closed b. The waiting area with the television turned off c. The patient's room before administration of pain medication d. The waiting room while the occupational therapist is working on leg exercises

The patient's room with the door closed

Family History

To determine whether the patient is at risk for illnesses of a genetic or familial nature.

Which of the following nursing interventions should be implemented to maintain a patent airway in a patient on bed rest? 1. Isometric exercises 2. Administration of low-dose heparin 3. Suctioning every 4 hours 4. Use of incentive spirometer every 2 hours while awake

Use of incentive spirometer every 2 hours while awake

Which nursing action will most likely increase a patient's risk for developing a health care-associated infection? a. Uses surgical aseptic technique to suction an airway b. Uses a clean technique for inserting a urinary catheter c. Uses a cleaning stroke from the urinary meatus toward the rectum d. Uses a sterile bottled solution more than once within a 24-hour period

Uses a clean technique for inserting a urinary catheter

A nurse is using professional standards to influence clinical decisions. What is the rationale for the nurse's actions? a. Establishes minimal passing standards for testing. b. Utilizes evidence-based practice based on nurses' needs. c. Bypasses the patient's feelings to promote ethical standards. d. Uses critical thinking for the highest level of quality nursing care.

Uses critical thinking for the highest level of quality nursing care.

Which action by a nurse indicates application of the critical thinking model to make the best clinical decisions? a. Drawing on past clinical experiences to formulate standardized care plans. b. Relying on recall of information from past lectures and textbooks. c. Depending on the charge nurse to determine priorities of care. d. Using the nursing process.

Using the nursing process.

The patient experienced a surgical procedure, and Betadine was utilized as the surgical prep. Two days postoperatively, the nurse's assessment indicates that the incision is red and has a small amount of purulent drainage. The patient reports tenderness at the incision site. The patient's temperature is 100.5° F, and the WBC is 10,500/mm3. Which action should the nurse take first? a. Plan to change the surgical dressing during the shift. b. Utilize SBAR to notify the primary health care provider. c. Reevaluate the temperature and white blood cell count in 4 hours. d. Check to see what solution was used for skin preparation in surgery.

Utilize SBAR to notify the primary health care provider.

Infectious disease

an illness caused by living pathogenic organisms such as meningitis or pneumonia.

The nurse is caring for a patient with an incision. Which actions will best indicate an understanding of medical and surgical asepsis for a sterile dressing change? a. Donning clean goggles, gown, and gloves to dress the wound b. Donning sterile gown and gloves to remove the wound dressing c. Utilizing clean gloves to remove the dressing and sterile supplies for the new dressing d. Utilizing clean gloves to remove the dressing and clean supplies for the new dressing

Utilizing clean gloves to remove the dressing and sterile supplies for the new dressing

Which statement by a nurse indicates a good understanding about the differences between data validation and data interpretation? a. "Data interpretation occurs before data validation." b. "Validation involves looking for patterns in professional standards." c. "Validation involves comparing data with other sources for accuracy." d. "Data interpretation involves discovering patterns in professional standards."

Validation involves comparing data with other sources for accuracy.

A nurse is assessing the body alignment of a standing patient. Which finding will the nurse report as normal? a. When observed laterally, the spinal curves align in a reversed "S" pattern. b. When observed posteriorly, the hips and shoulders form an "S" pattern. c. The arms should be crossed over the chest or in the lap. d. The feet should be close together with toes pointed out.

When observed laterally, the spinal curves align in a reversed "S" pattern.

Diagnostic and Laboratory data provide

further explanation of alterations or problems identified during the history and physical examination.

Evidence Based Knowledge

knowledge based on research or clinical expertise, makes you an informed critical thinker.

Droplet mode of transmission

large particles that travel up to 3 feet during coughing, sneezing, or talking and come in contact with susceptible host.

When assessing the apical pulse consider:

rate and rhythm

When assessing the radial pulse consider:

rate, rhythm, strength and equality

Hydrostatic Pneumonia

Inflammation of the lung from stasis or pooling of secretions

Cue

Information that you can obtain through your senses.

Nervous system related to movement

Regulates movementand posture

The nurse is admitting a patient with an infectious disease process. Which question will be most appropriate for a nurse to ask about the patient's susceptibility to this infectious process? a. "Do you have a spouse?" b. "Do you have a chronic disease?" c. "Do you have any children living in the home?" d. "Do you have any religious beliefs that will influence your care?"

"Do you have a chronic disease?"

A female adult patient presents to the clinic with reports of a white discharge and itching in the vaginal area. A nurse is taking a health history. Which question is the priority? a. "When was the last time you visited your primary health care provider?" b. "Has this condition affected your eating habits in any way?" c. "What medications are you currently taking?" d. "Are you able to sleep at night?"

"What medications are you currently taking?"

The nurse is attempting to prompt the patient to elaborate on the reports of daytime fatigue. Which question should the nurse ask? a. "Is there anything that you are stressed about right now that I should know?" b. "What reasons do you think are contributing to your fatigue?" c. "What are your normal work hours?" d. "Are you sleeping 8 hours a night?"

"What reasons do you think are contributing to your fatigue?"

A 55-year-old female patient was in a motor vehicle accident and is admitted to a surgical unit after repair of a fractured left arm and left leg. She also has a laceration on her forehead. An intravenous (IV) line is infusing in the right antecubital fossa, and pneumatic compression stockings are on the right lower leg. She is receiving oxygen via a simple face mask. Which sites do you instruct the nursing assistant to use for obtaining the patient's blood pressure and temperature? 1. Right antecubital and tympanic membrane 2. Right popliteal and rectal 3. Left antecubital and oral 4. Left popliteal and temporal artery

1

A patient is experiencing some problems with joint stability. The doctor has prescribed crutches for the patient to use while still being allowed to bear weight on both legs. Which of the following gaits should the patient be taught to use? 1. Four-point 2. Three-point 3. Two-point 4. Swing-through

1

The licensed practical nurse (LPN) provides you with the change-of-shift vital signs on four of your patients. Which patient do you need to assess first? 1. 84-year-old man recently admitted with pneumonia, RR 28, SpO2 89% 2. 54-year-old woman admitted after surgery for fractured arm, BP 160/86 mm Hg, HR 72 3. 63-year-old man with venous ulcers from diabetes, temperature 37.3° C (99.1° F), HR 84 4. 77-year-old woman with left mastectomy 2 days ago, RR 22, BP 148/62

1

The nurse recognizes that the older adult's progressive loss of total bone mass and tendency to take smaller steps with feet kept closer together will most likely: 1. Increase the patient's risk for falls and injuries. 2. Result in less stress on the patient's joints. 3. Decrease the amount of work required for patient movement. 4. Allow for mobility in spite of the aging effects on the patient's joints.

1

Before transferring a patient from the bed to a stretcher, which assessment data do the nurse need to gather? (Select all that apply.) 1. Patient's weight 2. Patient's level of cooperation 3. Patient's ability to assist 4. Presence of medical equipment 5. Nutritional intake

1, 2, 3, 4.

A nurse is teaching a community group about ways to minimize the risk of developing osteoporosis. Which of the following statements reflect understanding of what was taught? (Select all that apply.) 1. "I usually go swimming with my family at the YMCA 3 times a week." 2. "I need to ask my doctor if I should have a bone mineral density check this year." 3. "If I don't drink milk at dinner, I'll eat broccoli or cabbage to get the calcium that I need in my diet." 4. "I'll check the label of my multivitamin. If it has calcium, I can save money by not taking another pill." 5. "My lactose intolerance should not be a concern when considering my calcium intake."

1, 2, 3.

A patient who has been isolated for Clostridium difficile (C. difficile) asks you to explain what he should know about this organism. What is the most appropriate information to include in patient teaching? (Select all that apply.) 1. The organism is usually transmitted through the fecal-oral route. 2. Hands should always be cleaned with soap and water versus alcohol-based hand sanitizer. 3. Everyone coming into the room must be wearing a gown and gloves. 4. While the patient is in contact precautions, he cannot leave the room. 5. C. difficile dies quickly once outside the body.

1, 2, 3.

Identify the variety of sources where data can be obtained.

1. The patient through interviews. 2. Family memebers or significant others. 3. Other memebers of the heath team. 4. Medical record information. 5. Scientific or medical literature.

Korotkoff sounds (5)

1. Clear, rhythmic tapping (systolic bp). 2. Blowing or swishing sound as the cuff deflates. 3. Crisp more intense tapping. 4. Muffled low pitch as the cuff continues to deflate. 5. The disappearance of sound (diastolic bp)

Two steps of the nursing assessment

1. Collection and verification of data from primary and secondary sources. 2. Inerpretation and validation of data.

Gordon's Functional Health Patterns

1. Health perception-health management pattern. 2. Nutritional-metabolic pattern. 3. Elimination pattern. 4. Activity-exercise pattern. 5. Sleep-rest pattern. 6. Cognitive-perceptual pattern. 7. Self-perception-self-concept pattern. 8. Roles-relationships pattern. 9. Sexuality-reproductive pattern. 10. Coping-stress tolerance pattern. 11. Values-beliefs pattern.

Four stages of infection process

1. Incubation. 2. Prodromal stage. 3. Illness stage. 4. Convalescence.

Three phases of patient centered interviews

1. Orientation and setting an agenda. 2. Working phase: collecting assessment. 3. Terminating an interview.

Respiratory rate: Adults

12-20

Respiratory rate: Adolescents

16-20

An older adult has limited mobility as a result of a total knee replacement. During assessment you note that the patient has difficulty breathing while lying flat. Which of the following assessment data support a possible pulmonary problem related to impaired mobility? (Select all that apply.) 1. B/P = 128/84 2. Respirations 26/min on room air 3. HR 114 4. Crackles over lower lobes heard on auscultation 5. Pain reported as 3 on scale of 0 to 10 after medication

2, 3, 4.

The nurse evaluates that the NAP has applied a patient's sequential compression device (SCD) appropriately when which of the following is observed? (Select all that apply.) 1. Initial patient measurement is made around the calves 2. Inflation pressure averages 40 mm Hg 3. Patient's leg placed in SCD sleeve with back of knee aligned with popliteal opening on the sleeve. 4. Stockings are removed every 2 hours during application. 5. Yellow light indicates SCD device is functioning.

2, 3.

Which of the following actions by the nurse comply with core principles of surgical asepsis? (Select all that apply.) 1. Set up sterile field before patient and other staff come to the operating suite. 2. Keep the sterile field in view at all times. 3. Consider the outer 2.5 cm (1 inch) of the sterile field as contaminated. 4. Only health care personnel within the sterile field must wear personal protective equipment. 5. The sterile gown must be put on before the surgical scrub is performed.

2, 3.

A patient is admitted for dehydration caused by pneumonia and shortness of breath. He has a history of heart disease and cardiac dysrhythmias. The nursing assistant reports his admitting vital signs to the nurse. Which measurements should the nurse reassess? (Select all that apply.) 1. Right arm BP: 118/72 2. Radial pulse rate: 72 and irregular 3. Temporal temperature: 37.4° C (99.3° F) 4. Respiratory rate: 28 5. Oxygen saturation: 99%

2, 4, 5.

When should a nurse wear a mask? (Select all that apply.) 1. The patient's dental hygiene is poor. 2. The nurse is assisting with an aerosolizing respiratory procedure such as suctioning. 3. The patient has acquired immunodeficiency syndrome (AIDS) and a congested cough. 4. The patient is in droplet precautions. 5. The nurse is assisting a health care provider in the insertion of a central line catheter.

2, 4, 5.

The nurse manager is evaluating current infection control data for the intensive care unit. The nurse compares past patient data with current data to look for trends. The nurse manager examines the infection chain for possible solutions. In which order will the nurse arrange the items for the infection chain beginning with the first step? 1. A mode of transmission 2. An infectious agent or pathogen 3. A susceptible host 4. A reservoir or source for pathogen growth 5. A portal of entry to a host 6. A portal of exit from the reservoir

2, 4, 6, 1, 5, 3.

Respiratory rate: Children

20-30

A patient is diagnosed with methicillin-resistant Staphylococcus aureus (MRSA) pneumonia. Which type of isolation precaution is most appropriate for this patient? 1. Reverse isolation 2. Droplet precautions 3. Standard precautions 4. Contact precautions

2.

The nurse is caring for a patient who needs a protective environment. The nurse has provided the care needed and is now leaving the room. In which order will the nurse remove the personal protective equipment, beginning with the first step? 1. Remove eyewear/face shield and goggles. 2. Perform hand hygiene, leave room, and close door. 3. Remove gloves. 4. Untie gown, allow gown to fall from shoulders, and do not touch outside of gown; dispose of properly. 5. Remove mask by strings; do not touch outside of mask. 6. Dispose of all contaminated supplies and equipment in designated receptacles.

3, 1, 4, 5, 2, 6.

Respiratory rate: Infants

30-50

Respiratory rate: Newborn

30-60

The patient with heart failure is restless with a temperature of 102.2° F (39° C). Which action will the nurse take? a. Place the patient on oxygen. b. Encourage the patient to cough. c. Restrict the patient's fluid intake. d. Increase the patient's metabolic rate.

A

A 52-year-old woman is admitted with dyspnea and discomfort in her left chest with deep breaths. She has smoked for 35 years and recently lost over 10 lbs. Her vital signs on admission are: HR 112, BP 138/82, RR 22, tympanic temperature 36.8° C (98.2° F), and oxygen saturation 94%. She is receiving oxygen at 2 L via a nasal cannula. Which vital sign reflects a positive outcome of the oxygen therapy? 1. Temperature: 37° C (98.6° F) 2. Radial pulse: 112 3. Respiratory rate: 24 4. Oxygen saturation: 96% 5. Blood pressure: 134/78

4

A family member is providing care to a loved one who has an infected leg wound. What should the nurse instruct the family member to do after providing care and handling contaminated equipment or organic material? 1. Wear gloves before eating or handling food. 2. Place any soiled materials into a bag and double bag it. 3. Have the family member check with the health care provider about need for immunization. 4. Perform hand hygiene after care and/or handling contaminated equipment or material.

4

A patient has been admitted for a cerebrovascular accident (stroke). She cannot move her right arm, and she has a right-sided facial droop. She is able to eat with her dentures in place and swallow safely. The nursing assistive personnel (NAP) reports to you that the patient will not keep the oral thermometer probe in her mouth. What direction do you provide to the NAP? 1. Direct the NAP to hold the thermometer in place with her gloved hand 2. Direct the NAP to switch the thermometer probe to the left sublingual pocket 3. Direct the NAP to obtain a right tympanic temperature 4. Direct the NAP to use a temporal artery thermometer from right to left

4

The nurse encourages a patient with type 2 diabetes to engage in a regular exercise program primarily to improve the patient's: 1. Gastric motility, thereby facilitating glucose digestion. 2. Respiratory effort, thereby decreasing activity intolerance. 3. Overall cardiac output, thereby resuming resting heart rate. 4. Use of glucose and fatty acids, thereby decreasing blood glucose level.

4

The skin plays a role in temperature regulation by: 1. Insulating the body. 2. Constricting blood vessels. 3. Sensing external temperature variations. 4. All of the above.

4

Pulse range: Adults

60-100

Pulse range: Adolescents

60-90

After taking the patient's temperature, the nurse documents the value and the route used to obtain the reading. What is the reason for the nurse's action? a. Temperatures vary depending on the route used. b. Temperatures are readings of core measurements. c. Rectal temperatures are cooler than when taken orally. d. Axillary temperatures are higher than oral temperatures.

A

The health care provider prescription reads "Metoprolol (Lopressor) 50 mg PO daily. Do not give if blood pressure is less than 100 mm Hg systolic." The patient's blood pressure is 92/66. The nurse does not give the medication. Which action should the nurse take? a. Documents that the medication was not given because of low blood pressure b. Does not inform the health care provider that the medication was held c. Does not tell the patient what the blood pressure is d. Documents only what the blood pressure was.

A

The nurse is caring for a newborn infant in the hospital nursery and notices that the infant is breathing rapidly but is pink, warm, and dry. Which normal respiratory rate will the nurse consider when planning care for this newborn? a. 30 to 60 b. 22 to 28 c. 16 to 20 d. 10 to 15

A

The nurse needs to obtain a radial pulse from a patient. What must the nurse do to obtain a correct measurement? a. Place the tips of the first two fingers over the groove along the thumb side of the patient's wrist. b. Place the tips of the first two fingers over the groove along the little finger side of the patient's wrist. c. Place the thumb over the groove along the little finger side of the patient's wrist. d. Place the thumb over the groove along the thumb side of the patient's wrist.

A

The patient weighs 450 lbs (204.5 kg) and reports shortness of breath with any exertion. The health care provider has recommended beginning an exercise program. The patient states that she can hardly get out of bed and just cannot do anything around the house. Which nursing diagnosis will the nurse add to the care plan? a. Activity intolerance related to excessive weight b. Impaired physical mobility related to bed rest c. Imbalanced nutrition: less than body requirements d. Impaired gas exchange related to shortness of breath

A

Heatstroke

A dangerous heat emergency

Which of the following is a physiological effects of prolonged bed rest? 1. An increase in cardiac output. 2. A decreased in lean body mass. 3. A decrease in lung expansion. 4. A decrease in urinary excretion of nitrogen.

A decrease in lung expansion.

The nurse is caring for a patient who is immobile and needs to be turned every 2 hours. The patient has poor lower extremity circulation, and the nurse is concerned about irritation of the patient's toes. Which device will the nurse use? a. Hand rolls b. A foot cradle c. A trapeze bar d. A trochanter roll

A foot cradle

Which patient will cause the nurse to select a nursing diagnosis of Impaired physical mobilityfor a care plan? a. A patient who is completely immobile b. A patient who is not completely immobile c. A patient at risk for single-system involvement d. A patient who is at risk for multisystem problems

A patient who is not completely immobile

Body system defense

Organ systems that have unique defenses against infection i.e. skin, mouth, eye, respiratory tract, urinary tract, gastrointestinal tract, vagina.

The nurse is assessing a new patient admitted to home health. Which questions will be mostappropriate for the nurse to ask to determine the risk of infection? (Select all that apply.) a. "Can you explain the risk for infection in your home?" b. "Have you traveled outside of the United States?" c. "Will you demonstrate how to wash your hands?" d. "What are the signs and symptoms of infection?" e. "Are you able to walk to the mailbox?" f. "Who runs errands for you?"

A, B, C, D.

Which findings will alert the nurse that stress is present when making a clinical decision? (Select all that apply.) a. Tense muscles b. Reactive responses c. Trouble concentrating d. Very tired feelings e. Managed emotions

A, B, C, D.

A nurse is assessing activity tolerance of a patient. Which areas will the nurse assess? (Select all that apply.) a. Skeletal abnormalities b. Emotional factors c. Pregnancy status d. Race e. Age

A, B, C, E.

A nurse is teaching the staff about alterations in breathing patterns. Which information will the nurse include in the teaching session? (Select all that apply.) a. Apnea—no respirations b. Tachypnea—regular, rapid respirations c. Kussmaul's—abnormally deep, regular, fast respirations d. Hyperventilation—labored, increased in depth and rate respirations e. Cheyne-Stokes—abnormally slow and depressed ventilation respirations f. Biot's—irregular with alternating periods of apnea and hyperventilation respirations

A, B, C.

A nurse on a busy medicine unit is assigned to four patients. It is 10am. Two patients have medications due and one of those has a specimen of urine to be collected. One patient is having complications from surgery and is being prepared to return to the operating room. The fourth patient requires instructions before going home this afternoon. Which of the following should the nurse use in making clinical decisions appropriate for the patient group? (select all that apply) A. Consider availability of assistive personnel to obtain the specimen. B. Combine activities to resove more than one patient problem. C. Analyze the diagnoses/problems and decide which are most urgent based on patient needs. D. Plan a family conference for tomorrow to make decisions about resources the patient will need to go home. E. Identify the nursing diagnoses for the patient going home.

A, B, C.

The nurse and the student nurse are caring for two different patients on the medical-surgical unit. One patient is in airborne precautions, and one is in contact precautions. The nurse explains to the student different interventions for care. Which information will the nurse include in the teaching session? (Select all that apply.) a. Dispose of supplies to prevent the spread of microorganisms. b. Wash hands before entering and leaving both of the patients' rooms. c. Be consistent in nursing interventions since there is only one difference in the precautions. d. Apply the knowledge the nurse has of the disease process to prevent the spread of microorganisms. e. Have patients in airborne precautions wear a mask during transportation to other departments. f. Check the working order of the negative-pressure room for the airborne precaution patient on admission and at discharge.

A, B, D, E.

The nurse has received a report from the emergency department that a patient with tuberculosis will be coming to the unit. Which items will the nurse need to care for this patient? (Select all that apply.) a. Private room b. Negative-pressure airflow in room c. Surgical mask, gown, gloves, eyewear d. N95 respirator, gown, gloves, eyewear e. Communication signs for droplet precautions f. Communication signs for airborne precautions

A, B, D, F.

The nurse is assessing the patient and family for probable familial causes of the patient's hypertension. The nurse begins by analyzing the patient's personal history, as well as family history and current lifestyle situation. Which findings will the nurse consider to be risk factors? (Select all that apply.) a. Obesity b. Cigarette smoking c. Recent weight loss d. Heavy alcohol intake e. Regular exercise sessions

A, B, D.

The nurse is preparing to lift a patient. Which action will the nurse take first? a. Position a drawsheet under the patient. b. Assess weight and determine assistance needs. c. Delegate the task to a nursing assistive personnel. d. Attempt to manually lift the patient alone before asking for assistance.

Assess weight and determine assistance needs.

The nurse is caring for a patient with impaired physical mobility. Which potential complications will the nurse monitor for in this patient? (Select all that apply.) a. Footdrop b. Somnolence c. Hypostatic pneumonia d. Impaired skin integrity e. Increased socialization

A, C, D.

Upon assessment a nurse discovers that a patient has erythema. Which actions will the nurse take? (Select all that apply.) a. Consult a dietitian. b. Increase fiber in the diet. c. Place on chest physiotherapy. d. Increase frequency of turning. e. Place on pressure-relieving mattress.

A, D, E.

A nurse is working in a facility that uses no-lift policies. Which benefits will the nurse observe in the facility? (Select all that apply.) a. Reduced number of work-related injuries b. Increased musculoskeletal accidents c. Reduced safety of patients d. Improved health of nurses e. Increased indirect costs

A, D.

What is the correct sequence of steps when performing a wound irrigation? 1. Use slow continuous pressure to irrigate wound. 2. Attach angio catheter to syringe 3. Fill syringe with irrigation fluid 4. Place water proof bag near bed 5. Position angio catheter over wound A. 4, 3, 2, 5, 1 B. 3, 4, 2, 1, 5 C. 4, 2, 3, 5, 1 D. 2, 3, 4, 5, 1

A. 4, 3, 2, 5, 1 Organized steps ensure a safe effective irrigation of the wound.

The nurse is inserting a small-bore nasoenteric tube before starting enteral feedings. What is the correct order of steps to perform this procedure?1. Place patient in high-Fowler's position.2. Have patient flex head toward chest.3. Assess patient's gag reflex.4. Determine length of the tube to be inserted.5. Obtain radiological confirmation of tube placement.6. Check pH of gastric aspirate for verifying placement.7. Identify patient with two identifiers. A. 7, 1, 3, 4, 2, 5, 6 B. 1, 3, 4, 7, 2, 6, 5 C. 7, 1, 3, 2, 4, 6, 5 D. 1, 7, 3, 2, 4, 5, 6

A. 7, 1, 3, 4, 2, 5, 6

A patient has not had a bowel movement for 4 days. Now she has nausea and severe cramping throughout her abdomen. On the basis of these findings, what should the nurse suspect? A. An intestinal obstruction B. Irritation of the intestinal mucosa C. Gastroenteritis D. A fecal impaction

A. An intestinal obstruction Absence of bowel movement, nausea, cramping, and possibly vomiting are characteristic of an intestinal obstruction.

A patient is scheduled to have an intravenous pyelogram (IVP) the next morning. Which nursing measures should be implemented prior to the test? Select all that apply A. Ask the patient about any allergies and reactions. B. Instruct the patient that a full bladder is required for the test. C. Instruct the patient to save all urine in a special container. D. Ensure that informed consent has been obtained. E. Explain that the test includes instrumentation of the urinary tract.

A. Ask the patient about any allergies and reactions. D. Ensure that informed consent has been obtained. An intravenous pyelogram (IVP) involves intravenous injection of an iodine based contrast media. Patients that have had a previous hypersensitivity reaction to contrast media in the past are at high risk for another reaction. Informed consent is required. There is no need for a full bladder such as with a pelvic ultrasound or to save any urine for testing. There is no instrumentation of the urinary tract such as with a cystoscopy.

The nurse is teaching a program on healthy nutrition at the senior community center. Which points should be included in the program for older adults? A. Avoid grapefruit and grapefruit juice, which impair drug absorption. B. Increase the amount of carbohydrates for energy. C. Take a multivitamin that includes vitamin D for bone health. D. Cheese and eggs are good sources of protein. E. Limit fluids to decrease the risk of edema.

A. Avoid grapefruit and grapefruit juice, which impair drug absorption. C. Take a multivitamin that includes vitamin D for bone health. D. Cheese and eggs are good sources of protein. Cheese, eggs, and peanut butter are also useful high-protein alternatives. Vitamin D supplements are important for improving strength and balance, strengthening bone health, and preventing bone fractures and falls. Grapefruit and grapefruit juice can interfere with warfarin (Coumadin) (anticoagulant), preventing its breakdown. This would lead to an increased risk of bleeding.

What best describes measurement of post-void residual (PVR)? A. Bladder scan the patient immediately after voiding. B. Catheterize the patient 30 minutes after voiding. C. Bladder scan the patient when they report a strong urge to void. D. Catheterize the patient with a 16 Fr/10 mL catheter

A. Bladder scan the patient immediately after voiding. A PVR or post void residual is the measurement of urine in the bladder within 15 minutes of normal voiding. It would not be a true measurement of PVR if the bladder was full, or if after 30 minutes of voiding. A 16 Fr/10 mL catheter and would not be appropriate to use when catheterizing for PVR.

Which of the following symptoms are warning signs of possible colorectal cancer according to the American Cancer Society guidelines? A. Change in bowel habits B. Blood in the stool C. A larger-than-normal bowel movement D. Fecal impaction E. Muscle aches F. Incomplete emptying of the colon G. Food particles in the stool H.Unexplained abdominal or back pain

A. Change in bowel habits B. Blood in the stool F. Incomplete emptying of the colon H. Unexplained abdominal or back pain According to the American Cancer Society current guidelines, anyone with these symptoms should seek medical evaluation because they may have colon cancer. Other conditions may also cause these symptoms; but, if colon cancer is present, early diagnosis is important.

What is the removal of devitalized tissue from a wound called? A. Debridement B. Pressure reduction C. Negative pressure wound therapy D. Sanitization

A. Debridement Debridement is the removal of nonliving tissue, cleaning the wound to move toward healing.

An 86 year old woman is admitted to the unit with chills and a fever of 104 degrees F. What physiological process explains why she is at risk for dyspnea? A. Fever increases metabolic demands requiring increased oxygen need. B. Blood glucose stores are depleted and the cells do not have energy to use oxygen. C. Carbon dioxide production increases due to hyperventilation. D. Carbon dioxide production decreases due to hypoventilation.

A. Fever increases metabolic demands requiring increased oxygen need. When the body cannot meet the increased oxygenation need, the increased metabolic rate causes breakdown of protein and wasting of respiratory muscles increasing the work of breathing.

A patient who is receiving parenteral nutrition (PN) through a central venous catheter (CVC) has an air embolus. What would the nurse do first? A. Have the patient perform a Valsalva procedure B. Clamp the intravenous (IV) tubing to prevent more air from entering the line C. Have the patient take a deep breath and hold it D. Notify the health care provider immediately

A. Have the patient perform a Valsalva procedure Turn the patient on his or her left side to prevent air from entering the left side of the heart. Then have the patient perform a Valsalva maneuver (holding the breath and "bearing down")

The nurse is caring for an infant and is obtaining the patient's vital signs. Which artery will the nurse use to best obtain the infant's pulse? a. Radial b. Brachial c. Femoral d. Popliteal

B

After surgery the patient with a closed abdominal wound reports a sudden "pop" after coughing. When the nurse examines the surgical wound site, the sutures are open, and pieces of small bowel are noted at the bottom of the now-opened wound. Which are the priority nursing interventions? A. Notify the surgeon B. Allow the area to be exposed to air until all drainage has stopped C. Place several cold packs over the area, protecting the skin around the wound D. Cover the area with sterile, saline-soaked towels and immediately. E. Cover the area with sterile gauze and apply an abdominal binder

A. Notify the surgeon D. Cover the area with sterile, saline-soaked towels and immediately. If a patient has an opening in the surgical incision and a portion of the small bowel is noted, the small bowel must be protected until an emergency surgical repair can be done. The small bowel and abdominal cavity should be maintained in a sterile environment; thus sterile towels that are moistened with sterile saline should be used over the exposed bowel for protection and to keep the bowel moist.

The nurse goes to assess a new patient and finds him short of breath with a rate of 32 and lying supine in bed. What is the priority nursing action? A. Raise the head of the bed to 60 degrees or higher. B. Get his oxygen saturation with a pulse oximeter. C. Take his blood pressure and respiratory rate. D. Notify the health care provider of his shortness of breath.

A. Raise the head of the bed to 60 degrees or higher. Raising the head of the bed will bring the diaphragm down and allow for better chest expansion thus improving oxygenation.

Two hours after surgery, the nurse assesses a patient who had a chest tube inserted during surgery. There is 200 ml of dark red drainage in the chest tube at this time. What is the appropriate action for the nurse to perform? A. Record the amount and continue to monitor drainage. B. Notify the physician. C. Strip the chest tube starting at the chest D. Increase the suction by 10 mm Hg

A. Record the amount and continue to monitor drainage. Dark red drainage after surgery (50-200 ml per hour in first 3 hours) is expected but be aware of sudden increases greater than 100 ml per hour after the first three hours especially if becomes bright red in color.

A patient was admitted following a motor vehicle accident with multiple fractured ribs. Respiratory assessment includes signs/symptoms of secondary pneumothorax. Which are the most common assessment findings associated with a pneumothorax? (Select all that apply). A. Sharp pleuritic pain that worsens on inspiration B. Crackles over lung bases of affected lung C. Tracheal deviation toward the affected lung D. Worsening dyspnea E. Absent lung sounds to auscultation on affected side

A. Sharp pleuritic pain that worsens on inspiration D. Worsening dyspnea E. Absent lung sounds to auscultation on affected side When the lung collapses, as with a pneumothorax, the thoracic space fills with air, which irritates the parietal pleura causing inspiratory pain. Because of the collapsed lung there is reduced gas exchange in the affected area, reduced oxygenation and dyspnea result. When an area of the lung collapses, breath sounds over affected area are absent.

During the administration of a warm tap-water enema, the patient complains of cramping abdominal pain that he rates 6 out of 10. What is the first thing the nurse should do? A. Stop the instillation B. Ask the patient to take deep breaths to decrease the pain C. Add soapsuds to the enema D. Tell the patient to bear down as he would when having a bowel movement

A. Stop the instillation When a patient complains of pain during an enema, the instillation should be stopped, and an assessment done before discontinuing or resuming the procedure.

A nurse plans to provide education to the parents of school-age children, which includes the increased prevalence of __________________ as a result of children being less physically active outside of school.

Childhood obesity

The nurse is educating the patient and his family about the parenteral nutrition. Which aspect related to this form of nutrition would be appropriate to include? A. The purpose of the fat emulsion in parenteral nutrition is to prevent a deficiency in essential fatty acids. B. We can give you parenteral nutrition through your peripheral intravenous line to prevent further infection. C. The fat emulsion will help control hyperglycemia during periods of stress. D. The parenteral nutrition will help your wounds heal. E. Since we just started the parenteral nutrition, we will only infuse it at 50% of your daily needs for the next 6 hours.

A. The purpose of the fat emulsion in parenteral nutrition is to prevent a deficiency in essential fatty acids. C. The fat emulsion will help control hyperglycemia during periods of stress. D. The parenteral nutrition will help your wounds heal. Sometimes adding intravenous fat emulsions to parenteral nutrition supports the patient's need for supplemental kilocalories, prevents essential fatty acid deficiencies, and helps control hyperglycemia during periods of stress. Parenteral nutrition is administered at 50% of the patient's daily needs for the first 24 hours to assess how he or she is tolerating the infusion.

When is an application of a warm compress to an ankle muscle sprain indicated? A. To relieve edema B. To reduce shivering C. To improve blood flow to an injured part D. To protect bony prominences from pressure ulcers E. To immobilize area

A. To relieve edema C. To improve blood flow to an injured part Warm compresses can improve circulation by dilating blood vessels, and they reduce edema. The moisture of the compress conducts heat.

Which of the following are measures to reduce tissue damage from shear? A. Use a transfer device, e.g. transfer board B. Have head of bed elevated when transferring patient C. Have head of bed flat when re positioning patients D. Raise head of bed 60 degrees when patient positioned supine E. Raise head of bed 30 degrees when patient positioned supine

A. Use a transfer device, e.g. transfer board C. Have head of bed flat when re positioning patients E. Raise head of bed 30 degrees when patient positioned supine A transfer device can pick up a patient and prevent his or her skin from sticking to the bed sheet as he is repositioned. Positioning the patient flat when repositioning reduces shear. Positioning the patient with the head of the bed to be elevated at 30 degrees prevents him or her from sliding. The head of bed in higher position will cause patient to slide down, causing shear.

The nurse is caring for an older-adult patient and notes that the temperature is 96.8° F (36° C). How will the nurse interpret this finding? a. The patient has hyperthermia. b. The patient has a normal temperature. c. The patient is suffering from hypothermia. d. The patient is demonstrating increased metabolism.

B

Which skin care measures are used to manage a patient who is experiencing fecal and/or urinary incontinence? A.Frequent position changes. B. Keeping the buttocks exposed to air at all times C. Using a large absorbent diaper, changing when saturated D. Using an incontinence cleaner E. Frequent cleaning, applying an ointment, and covering the areas with a thick absorbent towel F. Applying a moisture barrier ointment

A.Frequent position changes. D. Using an incontinence cleaner F. Applying a moisture barrier ointment Skin that is in contact with stool and urine can become moist and soft, allowing it to become damaged. The stool contains bacteria and in some cases enzymes that can harm the skin if in contact for a prolonged period of time. The use of an incontinence cleaner provides a gentle removal of stool and urine, and the use of the moisture-barrier ointment provides a protective layer between the skin and the next incontinence episode. However, skin care and moisture barriers must also be used with frequent position changes to assist in reducing the risk for pressure ulcers.

Question 14 of 15 The nurse is caring for a patient with pneumonia who has severe malnutrition. The nurse recognizes that, because of the nutritional status, the patient is at increased risk for: A. Heart disease. B. Sepsis. C. Pleural effusion. D. Cardiac arrhythmias. E. Diarrhea.

ANS B. Sepsis. C. Pleural effusion. D. Cardiac arrhythmias. Patients who are malnourished on admission are at greater risk of life-threatening complications such as arrhythmia, pleural effusions, sepsis, or hemorrhage during hospitalization.

A nurse is caring for a patient who has poor tissue perfusion as the result of hypertension. When the patient asks what to eat for breakfast, which meal should the nurse suggest? a. A cup of nonfat yogurt with granola and a handful of dried apricots b. Whole wheat toast with butter and a side of bacon c. A bowl of cereal with whole milk and a banana d. Omelet with sausage, cheese, and onions

ANS: A A 2000-calorie diet of fruits, vegetables, and low-fat dairy foods that are high in fiber, potassium, calcium, and magnesium and low in saturated and total fat helps prevent and reduce the effects of hypertension. Nonfat yogurt with granola is a good source of calcium, fiber, and potassium; dried apricots add a second source of potassium. Although cereal and a banana provide fiber and potassium, skim milk should be substituted for whole milk to decrease fat. An omelet with sausage and cheese is high in fat. Butter and bacon are high in fat.

A nurse is providing discharge teaching for a patient who is going home with a guaiac test. Which statement by the patient indicates the need for further education? a. "If I get a blue color that means the test is negative." b. "I should not get any urine on the stool I am testing." c. "If I eat red meat before my test, it could give me false results." d. "I should check with my doctor to stop taking aspirin before the test."

ANS: A A blue color indicates a positive guaiac, or presence of fecal occult blood; the patient needs more teaching to correct this misconception. Proper patient education is important for viable results. Be sure specimen is free of toilet paper and not contaminated with urine. The patient needs to avoid certain foods, like red meat, to rule out a false positive. While the health care provider should be consulted before asking a patient to stop any medication, if there are no contraindications, the patient should be instructed to stop taking aspirin, ibuprofen, naproxen or other nonsteroidal antiinflammatory drugs for 7 days because these could cause a false-positive test result.

A patient is receiving opioids for pain. Which bowel assessment is a priority? a. Clostridium difficile b. Constipation c. Hemorrhoids d. Diarrhea

ANS: B Patients receiving opiates for pain after surgery often require a stool softener or laxative to prevent constipation. C. difficile occurs from antibiotics, not opioids. Hemorrhoids are caused by conditions other than opioids. Diarrhea does not occur as frequently as constipation

A nurse is caring for a hospitalized patient with a urinary catheter. Which nursing action best prevents the patient from acquiring an infection? a. Maintaining a closed urinary drainage system b. Inserting the catheter using strict clean technique c. Disconnecting and replacing the catheter drainage bag once per shift d. Fully inflating the catheter's balloon according to the manufacturer's recommendation

ANS: A A key intervention to prevent infection is maintaining a closed urinary drainage system. A catheter should be inserted in the hospital setting using sterile technique. Inflating the balloon fully prevents dislodgment and trauma, not infection. Disconnecting the drainage bag from the catheter creates a break in the system and an open portal of entry and increases risk of infection.

A patient has carbon dioxide retention from lung problems. Which type of diet will the nurse most likely suggest for this patient? a. Low-carbohydrate b. Low-caffeine c. High-caffeine d. High-carbohydrate

ANS: A A low-carbohydrate diet is best. Diets high in carbohydrates play a role in increasing the carbon dioxide load for patients with carbon dioxide retention. As carbohydrates are metabolized, an increased load of carbon dioxide is created and excreted via the lungs. A low- or high-caffeine diet is not as important as the carbohydrate load.

The patient is experiencing angina pectoris. Which assessment finding does the nurse expect when conducting a history and physical examination? a. Experiences chest pain after eating a heavy meal b. Experiences adequate oxygen saturation during exercise c. Experiences crushing chest pain for more than 20 minutes d. Experiences tingling in the left arm that lasts throughout the morning

ANS: A Angina pectoris is chest pain that results from limited oxygen supply. Often pain is precipitated by activities such as exercise, stress, and eating a heavy meal and lasts 3 to 5 minutes. Symptoms of angina pectoris are relieved by rest and/or nitroglycerin. Adequate oxygen saturation occurs with rest; inadequate oxygen saturation occurs during exercise. Pain lasting longer than 20 minutes or arm tingling that persists could be a sign of myocardial infarction.

A small-bore feeding tube is placed. Which technique will the nurse use to best verify tube placement? a. X-ray b. pH testing c. Auscultation d. Aspiration of contents

ANS: A At present, the most reliable method for verification of placement of small-bore feeding tubes is x-ray examination. Aspiration of contents and pH testing are not infallible. The nurse would need a more precise indicator to help differentiate the source of tube feeding aspirate. Auscultation is no longer considered a reliable method for verification of tube placement because a tube inadvertently placed in the lungs, pharynx, or esophagus transmits sound similar to that of air entering the stomach.

A patient requests the nurse's help to the bedside commode and becomes frustrated when unable to void in front of the nurse. How should the nurse interpret the patient's inability to void? a. The patient can be anxious, making it difficult for abdominal and perineal muscles to relax enough to void. b. The patient does not recognize the physiological signals that indicate a need to void. c. The patient is lonely, and calling the nurse in under false pretenses is a way to get attention. d. The patient is not drinking enough fluids to produce adequate urine output.

ANS: A Attempting to void in the presence of another can cause anxiety and tension in the muscles that make voiding difficult. Anxiety can impact bladder emptying due to inadequate relaxation of the pelvic floor muscles and urinary sphincter. The nurse should give the patient privacy and adequate time if appropriate. No evidence suggests that an underlying physiological (does not recognize signals or not drinking enough fluids) or psychological (lonely) condition exists.

A nurse is teaching a patient about the large intestine in elimination. In which order will the nurse list the structures, starting with the first portion? a. Cecum, ascending, transverse, descending, sigmoid, and rectum b. Ascending, transverse, descending, sigmoid, rectum, and cecum c. Cecum, sigmoid, ascending, transverse, descending, and rectum d. Ascending, transverse, descending, rectum, sigmoid, and cecum

ANS: A The large intestine is divided into the cecum, ascending colon, transverse colon, descending colon, sigmoid colon, and rectum. The large intestine is the primary organ of bowel elimination.

Exogeneous Infection

Comes from microorganisms outside the individual that do not exsist in normal floras.

A nurse is providing care to a patient with an indwelling catheter. Which practice indicates the nurse is following guidelines for avoiding catheter-associated urinary tract infection (CAUTI)? a. Drapes the urinary drainage tubing with no dependent loops b. Washes the drainage tube toward the meatus with soap and water c. Places the urinary drainage bag gently on the floor below the patient d. Allows the spigot to touch the receptacle when emptying the drainage bag

ANS: A Avoid dependent loops in urinary drainage tubing. Prevent the urinary drainage bag from touching or dragging on the floor. When emptying the urinary drainage bag, use a separate measuring receptacle for each patient. Do not let the drainage spigot touch the receptacle. Using a clean washcloth, soap, and water, with your dominant hand wipe in a circular motion along the length of the catheter for about 10 cm (4 inches), starting at the meatus and moving away.

The nurse is caring for a patient who has a wound drain with a collection device. The nurse notices that the collection device has a sudden decrease in drainage. Which action will the nurse take next? a. Call the health care provider; a blockage is present in the tubing. b. Chart the results on the intake and output flow sheet. c. Do nothing, as long as the evacuator is compressed. d. Remove the drain; a drain is no longer needed.

ANS: A Because a drainage system needs to be patent, look for drainage flow through the tubing, as well as around the tubing. A sudden decrease in drainage through the tubing may indicate a blocked drain, and you will need to notify the health care provider. The health care provider, not the nurse, determines the need for drain removal and removes drains. Charting the results on the intake and output flow sheet does not take care of the problem. The evacuator may be compressed even when a blockage is present.

The nurse is caring for a patient on the medical-surgical unit with a wound that has a drain and a dressing that needs changing. Which action should the nurse take first? a. Provide analgesic medications as ordered. b. Avoid accidentally removing the drain. c. Don sterile gloves. d. Gather supplies.

ANS: A Because removal of dressings is painful, if often helps to give an analgesic at least 30 minutes before exposing a wound and changing the dressing. The next sequence of events includes gathering supplies for the dressing change, donning gloves, and avoiding the accidental removal of the drain during the procedure.

The nurse is careful to monitor a patient's cardiac output. Which goal is the nurse trying to achieve? a. To determine peripheral extremity circulation b. To determine oxygenation requirements c. To determine cardiac dysrhythmias d. To determine ventilation status

ANS: A Cardiac output indicates how much blood is being circulated systemically throughout the body to the periphery. The amount of blood ejected from the left ventricle each minute is the cardiac output. Oxygen status would be determined by pulse oximetry and the presence of cyanosis. Cardiac dysrhythmias are an electrical impulse monitored through ECG results. Ventilation status is measured by respiratory rate, pulse oximetry, and capnography. Capnography provides instant information about the patient's ventilation. Ventilation status does not depend solely on cardiac output.

The nurse is describing the ChooseMyPlate program to a patient. Which statement from the patient indicates successful learning? a. "I can use this to make healthy lifestyle food choices." b. "I can use this to count specific calories of food." c. "I can use this for my baby girl." d. "I can use this when I am sick."

ANS: A ChooseMyPlate serves as a basic guide for making food choices for a healthy lifestyle. The ChooseMyPlate program was developed by the U.S. Department of Agriculture to replace the MyFoodPyramid program. It helps balance calories but does not provide specific calories of food. These guidelines are for Americans over the age of 2 years. These guidelines are provided for health, not sickness.

A patient with chronic obstructive pulmonary disease (COPD) asks the nurse why clubbing occurs. Which response by the nurse is most therapeutic? a. "Your disease doesn't send enough oxygen to your fingers." b. "Your disease affects both your lungs and your heart, and not enough blood is being pumped." c. "Your disease will be helped if you pursed-lip breathe." d. "Your disease often makes patients lose mental status."

ANS: A Clubbing of the nail bed can occur with COPD and other diseases that cause prolonged oxygen deficiency or chronic hypoxemia. Pursed-lipped breathing helps the alveoli stay open but is not the cause of clubbing. Loss of mental status is not a normal finding with COPD and will not result in clubbing. Low oxygen and not low circulating blood volume is the problem in COPD that results in clubbing.

A patient has severe flank pain. The urinalysis reveals presence of calcium phosphate crystals. The nurse will anticipate an order for which diagnostic test? a. Intravenous pyelogram b. Mid-stream urinalysis c. Bladder scan d. Cystoscopy

ANS: A Flank pain and calcium phosphate crystals are associated with renal calculi. An intravenous pyelogram allows the provider to observe pathological problems such as obstruction of the ureter. A mid-stream urinalysis is performed for a routine urinalysis or if an infection is suspected; a urinalysis was already performed, a mid-stream would not be obtained again. A cystoscopy is used to detect bladder tumors and obstruction of the bladder outlet and urethra. A bladder scan measures the amount of urine in the bladder.

The patient is taking phenazopyridine. When assessing the urine, what will the nurse expect? a. Red color b. Orange color c. Dark amber color d. Intense yellow color

ANS: B Some drugs change the color of urine (e.g., phenazopyridine—orange, riboflavin—intense yellow). Eating beets, rhubarb, and blackberries causes red urine. Dark amber urine is the result of high concentrations of bilirubin in patients with liver disease.

A nurse is caring for a patient with a continent urinary reservoir. Which action will the nurse take? a. Teach the patient how to self-cath the pouch. b. Teach the patient how to perform Kegel exercises. c. Teach the patient how to change the collection pouch. d. Teach the patient how to void using the Valsalva technique.

ANS: A In a continent urinary reservoir, the ileocecal valve creates a one-way valve in the pouch through which a catheter is inserted through the stoma to empty the urine from the pouch. Patients must be willing and able to catheterize the pouch 4 to 6 times a day for the rest of their lives. The second type of continent urinary diversion is called an orthotopic neobladder, which uses an ileal pouch to replace the bladder. Anatomically, the pouch is in the same position as the bladder was before removal, allowing a patient to void through the urethra using a Valsalva technique. In a ureterostomy or ileal conduit the patient has no sensation or control over the continuous flow of urine through the ileal conduit, requiring the effluent (drainage) to be collected in a pouch. Kegel exercises are ineffective for a patient with a continent urinary reservoir.

The patient is breathing normally. Which process does the nurse consider is working properly when the patient inspires? a. Stimulation of chemical receptors in the aorta b. Reduction of arterial oxygen saturation levels c. Requirement of elastic recoil lung properties d. Enhancement of accessory muscle usage

ANS: A Inspiration is an active process, stimulated by chemical receptors in the aorta. Reduced arterial oxygen saturation levels indicate hypoxemia, an abnormal finding. Expiration is a passive process that depends on the elastic recoil properties of the lungs, requiring little or no muscle work. Prolonged use of the accessory muscles does not promote effective ventilation and causes fatigue.

A nurse is preparing to lavage a patient in the emergency department for an overdose. Which tube should the nurse obtain? a. Ewald b. Dobhoff c. Miller-Abbott d. Sengstaken-Blakemore

ANS: A Lavage is irrigation of the stomach in cases of active bleeding, poisoning, or gastric dilation. The types of tubes include Levin, Ewald, and Salem sump. Sengstaken-Blakemore is used for compression by internal application of pressure by means of inflated balloon to prevent internal esophageal or GI hemorrhage. Dobhoff is used for enteral feeding. Miller-Abbott is used for gastric decompression.

A nurse is caring for a patient with chronic obstructive pulmonary disease (COPD) who is receiving 2 L/min of oxygen. Which oxygen delivery device is most appropriate for the nurse to administer the oxygen? a. Nasal cannula b. Simple face mask c. Non-rebreather mask d. Partial non-rebreather mask

ANS: A Nasal cannulas deliver oxygen from 1 to 6 L/min. All other devices (simple face mask, non-rebreather mask, and partial non-rebreather mask) are intended for flow rates greater than 6 L/min.

The nurse is completing an assessment of the patient's skin's integrity. Which assessment is the priority? a. Pressure points b. Breath sounds c. Bowel sounds d. Pulse points

ANS: A Observe pressure points such as bony prominences. The nurse continually assesses the skin for signs of ulcer development. Assessment for tissue pressure damage includes visual and tactile inspection of the skin. Assessment of pulses, breath sounds, and bowel sounds is part of a head-to-toe assessment and could influence the function of the body and ultimately skin integrity; however, this assessment is not a specific part or priority of a skin assessment

A patient is experiencing oliguria. Which action should the nurse perform first? a. Assess for bladder distention. b. Request an order for diuretics. c. Increase the patient's intravenous fluid rate. d. Encourage the patient to drink caffeinated beverages.

ANS: A Oliguria is diminished urinary output in relation to fluid intake. The nurse first should gather all assessment data to determine the potential cause of oliguria. It could be that the patient does not have adequate intake, or it could be that the bladder sphincter is not functioning and the patient is retaining water. Increasing fluids is effective if the patient does not have adequate intake or if dehydration occurs. Caffeine can work as a diuretic but is not helpful if an underlying pathology is present. An order for diuretics can be obtained if the patient was retaining water, but this should not be the first action.

30.The patient has just started on enteral feedings, and the patient is reporting abdominal cramping. Which action will the nurse take next? a. Slow the rate of tube feeding. b. Instill cold formula to "numb" the stomach. c. Change the tube feeding to a high-fat formula. d. Consult with the health care provider about prokinetic medication.

ANS: A One possible cause of abdominal cramping is a rapid increase in rate or volume. Lowering the rate of delivery may increase tolerance. Another possible cause of abdominal cramping is the use of cold formula. The nurse should warm the formula to room temperature. High-fat formulas are also a cause of abdominal cramping. Consult with the health care provider regarding prokinetic medication for increasing gastric motility for delayed gastric emptying.

The nurse is working on a medical-surgical unit that has been participating in a research project associated with pressure ulcers. Which risk factor will the nurse assess for that predisposes a patient to pressure ulcer development? a. Decreased level of consciousness b. Adequate dietary intake c. Shortness of breath d. Muscular pain

ANS: A Patients who are confused or disoriented or who have changing levels of consciousness are unable to protect themselves. The patient may feel the pressure but may not understand what to do to relieve the discomfort or to communicate that he or she is feeling discomfort. Impaired sensory perception, impaired mobility, shear, friction, and moisture are other predisposing factors. Shortness of breath, muscular pain, and an adequate dietary intake are not included among the predisposing factors.

A patient asks about treatment for stress urinary incontinence. Which is the nurse's bestresponse? a. Perform pelvic floor exercises. b. Avoid voiding frequently. c. Drink cranberry juice. d. Wear an adult diaper.

ANS: A Poor muscle tone leads to an inability to control urine flow. The nurse should recommend pelvic muscle strengthening exercises such as Kegel exercises; this solution best addresses the patient's problem. Evidence has shown that patients with urgency, stress, and mixed urinary incontinence can eventually achieve continence when treated with pelvic floor muscle training. Drinking cranberry juice is a preventative measure for urinary tract infection. The nurse should not encourage the patient to reduce voiding; residual urine in the bladder increases the risk of infection. Wearing an adult diaper could be considered if attempts to correct the root of the problem fail.

The nurse is caring for a patient with fluid volume overload. Which physiological effect does the nurse most likely expect? a. Increased preload b. Increased heart rate c. Decreased afterload d. Decreased tissue perfusion

ANS: A Preload refers to the amount of blood in the left ventricle at the end of diastole; an increase in circulating volume would increase the preload of the heart. Afterload refers to resistance; increased pressure would lead to increased resistance, and afterload would increase. A decrease in tissue perfusion would be seen with hypovolemia. A decrease in fluid volume would cause an increase in heart rate as the body is attempting to increase cardiac output.

A patient has fallen several times in the past week when attempting to get to the bathroom. The patient gets up 3 or 4 times a night to urinate. Which recommendation by the nurse is most appropriate in correcting this urinary problem? a. Limit fluid and caffeine intake before bed. b. Leave the bathroom light on to illuminate a pathway. c. Practice Kegel exercises to strengthen bladder muscles. d. Clear the path to the bathroom of all obstacles before bedtime.

ANS: A Reducing fluids, especially caffeine and alcohol, before bedtime can reduce nocturia. To prevent nocturia, suggest that the patient avoid drinking fluids 2 hours before bedtime. Clearing a path to the bathroom, illuminating the path, or shortening the distance to the bathroom may reduce falls but will not correct the urination problem. Kegel exercises are useful if a patient is experiencing stress incontinence.

The patient has right-sided heart failure. Which finding will the nurse expect when performing an assessment? a. Peripheral edema b. Basilar crackles c. Chest pain d. Cyanosis

ANS: A Right-sided heart failure results from inability of the right side of the heart to pump effectively, leading to a systemic backup. Peripheral edema, distended neck veins, and weight gain are signs of right-sided failure. Basilar crackles can indicate pulmonary congestion from left-sided heart failure. Cyanosis and chest pain result from inadequate tissue perfusion.

The patient is on parenteral nutrition and is lethargic. The patient reports thirst and headache and has had increased urination. Which problem does the nurse prepare to address? a. Hyperglycemia b. Hypoglycemia c. Hypercapnia d. Hypocapnia

ANS: A Signs and symptoms of hyperglycemia are thirst, headache, lethargy, and increased urination. Hypocapnia is not associated with parenteral nutrition. Hypercapnia increases oxygen consumption and increases CO2 levels. Ventilator-dependent patients are at greatest risk for this. Hypoglycemia is characterized by diaphoresis, shakiness, confusion, and loss of consciousness.

The nurse is providing home care for a patient diagnosed with acquired immunodeficiency syndrome (AIDS). Which dietary intervention will the nurse add to the care plan? a. Provide small, frequent nutrient-dense meals for maximizing kilocalories. b. Prepare hot meals because they are more easily tolerated by the patient. c. Avoid salty foods and limit liquids to preserve electrolytes. d. Encourage intake of fatty foods to increase caloric intake.

ANS: A Small, frequent, nutrient-dense meals that limit fatty foods and overly sweet foods are easier to tolerate. Restorative care of malnutrition resulting from AIDS focuses on maximizing kilocalories and nutrients. Patients benefit from eating cold foods and drier or saltier foods with fluid in between.

When planning care for an adolescent who plays sports, which modification should the nurse include in the care plan? a. Increasing carbohydrates to 55% to 60% of total intake b. Providing vitamin and mineral supplements c. Decreasing protein intake to 0.75 g/kg/day d. Limiting water before and after exercise

ANS: A Sports and regular moderate to intense exercise necessitate dietary modification to meet increased energy needs for adolescents. Carbohydrates, both simple and complex, are the main source of energy, providing 55% to 60% of total daily kilocalories. Protein needs increase to 1 to 1.5 g/kg/day. Fat needs do not increase. Adequate hydration is very important. Adolescents need to ingest water before and after exercise to prevent dehydration, especially in hot, humid environments. Vitamin and mineral supplements are not required, but intake of iron-rich foods is required to prevent anemia.

A nurse is assigned most of the patients with pressure ulcers. The nurse leaves the pressure ulcer open to air and does not apply a dressing. To which patient did the nurse provide care? a. A patient with a clean Stage I b. A patient with a clean Stage II c. A patient with a clean Stage III d. A patient with a clean Stage IV

ANS: A Stage I intact pressure ulcers that resolve slowly without epidermal loss over 7 to 14 days do not require a dressing. A composite film, hydrocolloid, or hydrogel can be utilized on a clean Stage II. A hydrocolloid, hydrogel covered with foam, calcium alginate, and gauze can be utilized with a clean Stage III. Hydrogel covered with foam, calcium alginate, and gauze can be utilized with a clean Stage IV. An unstageable wound covered with eschar should utilize a dressing of adherent film or gauze with an ordered solution of enzymes

A patient is using laxatives three times daily to lose weight. After stopping laxative use, the patient has difficulty with constipation and wonders if laxatives should be taken again. Which information will the nurse share with the patient? a. Long-term laxative use causes the bowel to become less responsive to stimuli, and constipation may occur. b. Laxatives can cause trauma to the intestinal lining and scarring may result, leading to decreased peristalsis. c. Long-term use of emollient laxatives is effective for treatment of chronic constipation and may be useful in certain situations. d. Laxatives cause the body to become malnourished, so when the patient begins eating again, the body absorbs all of the food, and no waste products are produced.

ANS: A Teach patients about the potential harmful effects of overuse of laxatives, such as impaired bowel motility and decreased response to sensory stimulus. Make sure the patient understands that laxatives are not to be used long term for maintenance of bowel function. Increasing fluid and fiber intake can help with this problem. Laxatives do not cause scarring. Even if malnourished, the body will produce waste if any substance is consumed.

A nurse is reviewing the electrocardiogram (ECG) results. Which portion of the conduction system does the nurse consider when evaluating the P wave? a. SA node b. AV node c. Bundle of His d. Purkinje fibers

ANS: A The P wave represents the electrical conduction through both atria; the SA node initiates electrical conduction through the atria. The AV node conducts down through the bundle of His and the Purkinje fibers to cause ventricular contraction.

A nurse is teaching a nutrition class about the different daily values. When teaching about the referenced daily intakes (RDIs), which information should the nurse include? a. Have values for protein, vitamins, and minerals b. Are based on percentages of fat, cholesterol, and fiber c. Have replaced recommended daily allowances (RDAs) d. Are used to develop diets for chronic illnesses requiring 1800 cal/day

ANS: A The RDIs are the first set, comprising protein, vitamins, and minerals based on the RDA. The daily reference values (DRVs) make up the second set and consist of nutrients such as total fat, saturated fat, cholesterol, carbohydrates, fiber, sodium, and potassium. Combined, both sets make up the daily values used on food labels. Daily values did not replace RDAs but provided a separate, more understandable format for the public. Daily values are based on percentages of a diet consisting of 2000 kcal/day for adults and children 4 years or older.

A patient is diagnosed with a bowel obstruction. Which type of tube is the best for the nurse to obtain for gastric decompression? a. Salem sump b. Small bore c. Levin d. 8 Fr

ANS: A The Salem sump tube is preferable for stomach decompression. The Salem sump tube has two lumina: one for removal of gastric contents and one to provide an air vent. When the main lumen of the sump tube is connected to suction, the air vent permits free, continuous drainage of secretions. While the Levin tube can be used for decompression, it is only a single-lumen tube with holes near the tip. Large-bore tubes, 12 Fr and above, are usually used for gastric decompression or removal of gastric secretions. Fine- or small-bore tubes are frequently used for medication administration and enteral feedings.

A nurse explains the function of the alveoli to a patient with respiratory problems. Which information about the alveoli's function will the nurse share with the patient? a. Carries out gas exchange b. Regulates tidal volume c. Produces hemoglobin d. Stores oxygen

ANS: A The alveolus is a capillary membrane that allows gas exchange of oxygen and carbon dioxide during respiration. The alveoli do not store oxygen, regulate tidal volume, or produce hemoglobin.

Which coughing technique will the nurse use to help a patient clear central airways? a. Huff b. Quad c. Cascade d. Incentive spirometry

ANS: A The huff cough stimulates a natural cough reflex and is generally effective only for clearing central airways. While exhaling, the patient opens the glottis by saying the word huff. The quad cough technique is for patients without abdominal muscle control such as those with spinal cord injuries. While the patient breathes out with a maximal expiratory effort, the patient or nurse pushes inward and upward on the abdominal muscles toward the diaphragm, causing the cough. With the cascade cough the patient takes a slow, deep breath and holds it for 2 seconds while contracting expiratory muscles. Then he or she opens the mouth and performs a series of coughs throughout exhalation, thereby coughing at progressively lowered lung volumes. This technique promotes airway clearance and a patent airway in patients with large volumes of sputum. Incentive spirometry encourages voluntary deep breathing by providing visual feedback to patients about inspiratory volume. It promotes deep breathing and prevents or treats atelectasis in the postoperative patient.

A nurse is caring for a group of patients. Which patient should the nurse see first? a. A patient with hypercapnia wearing an oxygen mask b. A patient with a chest tube ambulating with the chest tube unclamped c. A patient with thick secretions being tracheal suctioned first and then orally d. A patient with a new tracheostomy and tracheostomy obturator at bedside

ANS: A The mask is contraindicated for patients with carbon dioxide retention (hypercapnia) because retention can be worsened; the nurse must see this patient first to correct the problem. All the rest are using correct procedures and do not need to be seen first. A chest tube should not be clamped when ambulating. Clamping a chest tube is contraindicated when ambulating or transporting a patient. Clamping can result in a tension pneumothorax. Use nasotracheal suctioning before pharyngeal suctioning whenever possible. The mouth and pharynx contain more bacteria than the trachea. Keep tracheostomy obturator at bedside with a fresh (new) tracheostomy to facilitate reinsertion of the outer cannula if dislodged.

The nurse is devising a plan of care for a patient with the nursing diagnosis of Constipation related to opioid use. Which outcome will the nurse evaluate as successful for the patient to establish normal defecation? a. The patient reports eliminating a soft, formed stool. b. The patient has quit taking opioid pain medication. c. The patient's lower left quadrant is tender to the touch. d. The nurse hears bowel sounds in all four quadrants.

ANS: A The nurse's goal is for the patient to take opioid medication and to have normal bowel elimination. Normal stools are soft and formed. Ceasing pain medication is not a desired outcome for the patient. Tenderness in the left lower quadrant indicates constipation and does not indicate success. Bowel sounds indicate that the bowels are moving; however, they are not an indication of defecation.

The nurse is caring for a patient with a healing Stage III pressure ulcer. Upon entering the room, the nurse notices an odor and observes a purulent discharge, along with increased redness at the wound site. What is the next best step for the nurse? a. Complete the head-to-toe assessment, including current treatment, vital signs, and laboratory results. b. Notify the health care provider by utilizing Situation, Background, Assessment, and Recommendation (SBAR). c. Consult the wound care nurse about the change in status and the potential for infection. d. Check with the charge nurse about the change in status and the potential for infection.

ANS: A The patient is showing signs and symptoms associated with infection in the wound. The nurse should complete the assessment: gather all data such as current treatment modalities, medications, vital signs including temperature, and laboratory results such as the most recent complete blood count or white cell count. The nurse can then notify the primary care provider and receive treatment orders for the patient. It is important to notify the charge nurse and consult the wound nurse on the patient's status and on any new orders.

Which nursing observation will indicate the patient is at risk for pressure ulcer formation? a. The patient has fecal incontinence. b. The patient ate two thirds of breakfast. c. The patient has a raised red rash on the right shin. d. The patient's capillary refill is less than 2 seconds.

ANS: A The presence and duration of moisture on the skin increase the risk of ulcer formation by making it susceptible to injury. Moisture can originate from wound drainage, excessive perspiration, and fecal or urinary incontinence. Bacteria and enzymes in the stool can enhance the opportunity for skin breakdown because the skin is moistened and softened, causing maceration. Eating a balanced diet is important for nutrition, but eating just two thirds of the meal does not indicate that the individual is at risk. A raised red rash on the leg again is a concern and can affect the integrity of the skin, but it is located on the shin, which is not a high-risk area for skin breakdown. Pressure can influence capillary refill, leading to skin breakdown, but this capillary response is within normal limits.

A patient with a fecal impaction has an order to remove stool digitally. In which order will the nurse perform the steps, starting with the first one? 1. Obtain baseline vital signs. 2. Apply clean gloves and lubricate. 3. Insert index finger into the rectum. 4. Identify patient using two identifiers. 5. Place patient on left side in Sims' position. 6. Massage around the feces and work down to remove. a. 4, 1, 5, 2, 3, 6 b. 1, 4, 2, 5, 3, 6 c. 4, 1, 2, 5, 3, 6 d. 1, 4, 5, 2, 3, 6

ANS: A The steps for removing a fecal impaction are as follows: identify patient using two identifiers; obtain baseline vital signs; place on left side in Sims' position; apply clean gloves and lubricate; insert index finger into the rectum; and gently loosen the fecal mass by massaging around it and work the feces downward toward the end of the rectum.

The patient is an 80-year-old male who is visiting the clinic today for a routine physical examination. The patient's skin turgor is fair, but the patient reports fatigue and weakness. The skin is warm and dry, pulse rate is 116 beats/min, and urinary sodium level is slightly elevated. Which instruction should the nurse provide? a. Drink more water to prevent further dehydration. b. Drink more calorie-dense fluids to increase caloric intake. c. Drink more milk and dairy products to decrease the risk of osteoporosis. d. Drink more grapefruit juice to enhance vitamin C intake and medication absorption.

ANS: A Thirst sensation diminishes, leading to inadequate fluid intake or dehydration; the patient should be encouraged to drink more water/fluids. Symptoms of dehydration in older adults include confusion, weakness, hot dry skin, furrowed tongue, and high urinary sodium. Milk continues to be an important food for older woman and men, who need adequate calcium to protect against osteoporosis; the patient's problem is dehydration, not osteoporosis. Caution older adults to avoid grapefruit and grapefruit juice because these will decrease absorption of many drugs. The patient needs fluids not calories; drinking calorie-dense fluids is unnecessary.

A nurse is performing an assessment on a patient who has not had a bowel movement in 3 days. The nurse will expect which other assessment finding? a. Hypoactive bowel sounds b. Increased fluid intake c. Soft tender abdomen d. Jaundice in sclera

ANS: A Three or more days with no bowel movement indicates hypomotility of the GI tract. Assessment findings would include hypoactive bowel sounds, a firm distended abdomen, and pain or discomfort upon palpation. Increased fluid intake would help the problem; a decreased intake can lead to constipation. Jaundice does not occur with constipation but can occur with liver disease.

The nurse is creating a plan of care for an obese patient who is suffering from fatigue related to ineffective breathing. Which intervention best addresses a short-term goal the patient could achieve? a. Sleeping on two to three pillows at night b. Limiting the diet to 1500 calories a day c. Running 30 minutes every morning d. Stopping smoking immediately

ANS: A To achieve a short-term goal, the nurse should plan a lifestyle change that the patient can make immediately that will have a quick effect. Sleeping on several pillows at night will immediately relieve orthopnea and open the patient's airway, thereby reducing fatigue. Running 30 minutes a day will improve cardiopulmonary health, but a patient needs to build up exercise tolerance. Smoking cessation is another process that many people have difficulty doing immediately. A more realistic short-term goal would be to gradually reduce the number of cigarettes smoked. Limiting caloric intake can help a patient lose weight, but this is a gradual process and is not reasonable for a short-term goal.

To reduce patient discomfort during a closed intermittent catheter irrigation, what should the nurse do? a. Use room temperature irrigation solution. b. Administer the solution as quickly as possible. c. Allow the solution to sit in the bladder for at least 1 hour. d. Raise the bag of the irrigation solution at least 12 inches above the bladder.

ANS: A To reduce discomfort use room temperature solution. Using cold solutions and instilling solutions too quickly can cause discomfort. During an irrigation, the solution does not sit in the bladder; it is allowed to drain. A container is not raised about the bladder 12 inches when performing a closed intermittent catheter irrigation.

The nurse is caring for a patient who has suffered a stroke and has residual mobility problems. The patient is at risk for skin impairment. Which initial actions should the nurse take to decrease this risk? a. Use gentle cleansers, and thoroughly dry the skin. b. Use therapeutic bed and mattress. c. Use absorbent pads and garments. d. Use products that hold moisture to the skin.

ANS: A Use cleansers with nonionic surfactants that are gentle to the skin. After you clean the skin, make sure that it is completely dry. Absorbent pads and garments are controversial and should be considered only when other alternatives have been exhausted. Depending on the needs of the patient, a specialty bed may be needed, but again, this does not provide the initial defense for skin breakdown. Use only products that wick moisture away from the patient's skin.

A nurse is caring for a patient with continuous cardiac monitoring for heart dysrhythmias. Which rhythm will cause the nurse to intervene immediately? a. Ventricular tachycardia b. Atrial fibrillation c. Sinus rhythm d. Paroxysmal supraventricular tachycardia

ANS: A Ventricular tachycardia and ventricular fibrillation are life-threatening rhythms that require immediate intervention. Ventricular tachycardia is a life-threatening dysrhythmia because of the decreased cardiac output and the potential to deteriorate into ventricular fibrillation or sudden cardiac death. Atrial fibrillation is a common dysrhythmia in older adults and is not as serious as ventricular tachycardia. Sinus rhythm is normal. Paroxysmal supraventricular tachycardia is a sudden, rapid onset of tachycardia originating above the AV node. It often begins and ends spontaneously.

The nurse is preparing to apply an external catheter. Which action will the nurse take? a. Allow 1 to 2 inches of space between the tip of the penis and the end of the catheter. b. Spiral wrap the penile shaft using adhesive tape to secure the catheter. c. Twist the catheter before applying drainage tubing to the end of the catheter. d. Shave the pubic area before applying the catheter.

ANS: A When applying an external catheter, allow 2.5 to 5 cm (1 to 2 inches) of space between the tip of the penis and the end of the catheter. Spiral wrap the penile shaft with supplied elastic adhesive. The strip should not overlap. The elastic strip should be snug but not tight. NOTE: Never use adhesive tape. Connect drainage tubing to the end of the condom catheter. Be sure the condom is not twisted. Connect the catheter to a large-volume drainage bag or leg. Clip hair at the base of the penile shaft, as necessary. Do not shave the pubic area.

A nurse is teaching about risk factors for cardiopulmonary disease. Which risk factor should the nurse describe as modifiable? a. Stress b. Allergies c. Family history d. Gender

ANS: A Young and middle-age adults are exposed to multiple cardiopulmonary risk factors: an unhealthy diet, lack of exercise, stress, over-the-counter and prescription drugs not used as intended, illegal substances, and smoking. Reducing these modifiable factors decreases a patient's risk for cardiac or pulmonary diseases. A nonmodifiable risk factor is family history; determine familial risk factors such as a family history of lung cancer or cardiovascular disease. Other nonmodifiable risk factors include allergies and gender.

A nurse is caring for patients with dysphagia. Which patient has neurogenic dysphagia? a. A patient with benign peptic stricture b. A patient with muscular dystrophy c. A patient with myasthenia gravis d. A patient with stroke

ANS: D Stroke is the only cause of dysphagia in this list that is considered neurogenic. Myasthenia gravis and muscular dystrophy are considered myogenic in origin, whereas benign peptic stricture is considered obstructive.

A nurse is teaching a health class about colorectal cancer. Which information should the nurse include in the teaching session? (Select all that apply.) a. A risk factor is smoking. b. A risk factor is high intake of animal fats or red meat. c. A warning sign is rectal bleeding. d. A warning sign is a sense of incomplete evacuation. e. Screening with a colonoscopy is every 5 years, starting at age 50. f. Screening with flexible sigmoidoscopy is every 10 years, starting at age 50.

ANS: A, B, C, D Risk factors for colorectal cancer are a diet high in animal fats or red meat and low intake of fruits and vegetables; smoking and heavy alcohol consumption are also risk factors. Warning signs are change in bowel habits, rectal bleeding, a sensation of incomplete evacuation, and unexplained abdominal or back pain. A flexible sigmoidoscopy is every 5 years, starting at age 50, while a colonoscopy is every 10 years, starting at age 50.

The nurse is caring for a patient with a wound healing by full-thickness repair. Which phases will the nurse monitor for in this patient? (Select all that apply.) a. Hemostasis b. Maturation c. Inflammatory d. Proliferative e. Reproduction f. Reestablishment of epidermal layers

ANS: A, B, C, D The four phases involved in the healing process of a full-thickness wound are hemostasis, inflammatory, proliferative, and maturation. Three components are involved in the healing process of a partial-thickness wound: inflammatory response, epithelial proliferation (reproduction) and migration, and reestablishment of the epidermal layers.

The nurse is completing a skin assessment on a medical-surgical patient. Which nursing assessment questions should be included in a skin integrity assessment? (Select all that apply.) a. "Can you easily change your position?" b. "Do you have sensitivity to heat or cold?" c. "How often do you need to use the toilet?" d. "What medications do you take?" e. "Is movement painful?" f. "Have you ever fallen?"

ANS: A, B, C, E Changing positions is important for decreasing the pressure associated with long periods of time in the same position. If the patient is able to feel heat or cold and is mobile, she can protect herself by withdrawing from the source. Knowing toileting habits and any potential for incontinence is important because urine and feces in contact with the skin for long periods can increase skin breakdown. Knowing whether the patient has problems with painful movement will alert the nurse to any potential for decreased movement and increased risk for skin breakdown. Medications and falling are safety risk questions.

In providing diabetic teaching for a patient with type 1 diabetes mellitus, which instructions will the nurse provide to the patient? a. Insulin is the only consideration that must be taken into account. b. Saturated fat should be limited to less than 7% of total calories. c. Nonnutritive sweeteners can be used without restriction. d. Cholesterol intake should be greater than 200 mg/day.

ANS: B The diabetic patient should limit saturated fat to less than 7% of total calories and cholesterol intake to less than 200 mg/day. Type 1 diabetes requires both insulin and dietary restrictions for optimal control. Nonnutritive sweeteners can be eaten as long as the recommended daily intake levels are followed.

A nurse is preparing a bowel training program for a patient. Which actions will the nurse take? (Select all that apply.) a. Record times when the patient is incontinent. b. Help the patient to the toilet at the designated time. c. Lean backward on the hips while sitting on the toilet. d. Maintain normal exercise within the patient's physical ability. e. Apply pressure with hands over the abdomen, and strain while pushing. f. Choose a time based on the patient's pattern to initiate defecation-control measures.

ANS: A, B, D, F A successful program includes the following: Assessing the normal elimination pattern and recording times when the patient is incontinent. Choosing a time based on the patient's pattern to initiate defecation-control measures. Maintaining normal exercise within the patient's physical ability. Helping the patient to the toilet at the designated time. Offering a hot drink (hot tea) or fruit juice (prune juice) (or whatever fluids normally stimulate peristalsis for the patient) before the defecation time. Instructing the patient to lean forward at the hips while sitting on the toilet, apply manual pressure with the hands over the abdomen, and bear down but do not strain to stimulate colon emptying.

A nurse is teaching a health class about the nutritional requirements throughout the life span. Which information should the nurse include in the teaching session? (Select all that apply.) a. Infants triple weight at 1 year. b. Toddlers become picky eaters. c. School-age children need to avoid hot dogs and grapes. d. Breastfeeding women need an additional 750 kcal/day. e. Older adults have altered food flavor from a decrease in taste cells.

ANS: A, B, E An infant usually doubles birth weight at 4 to 5 months and triples it at 1 year. Toddlers exhibit strong food preferences and become picky eaters. Older adults often experience a decrease in taste cells that alters food flavor and may decrease intake. Toddlers need to avoid hot dogs and grapes, not school-age children. The lactating woman needs 500 kcal/day above the usual allowance because the production of milk increases energy requirements.

The nurse is using different toileting schedules. Which principles will the nurse keep in mind when planning care? (Select all that apply.) a. Habit training uses a bladder diary. b. Timed voiding is based upon the patient's urge to void. c. Prompted voiding includes asking patients if they are wet or dry. d. Elevation of feet in patients with edema can decrease nighttime voiding. e. Bladder retraining teaches patients to follow the urge to void as quickly as possible.

ANS: A, C Habit training is a toileting schedule based upon the patient's usual voiding pattern. Using a bladder diary, the usual times a patient voids are identified. It is at these times that the patient is then toileted. Prompted voiding is a program of toileting designed for patients with mild or moderately cognitive impairment. Patients are toileted based upon their usual voiding pattern. Caregivers ask the patient if they are wet or dry, give positive feedback for dryness, prompt the patient to toilet, and reward the patient for desired behavior. Timed voiding or scheduled toileting is toileting based upon a fixed schedule, not the patient's urge to void. The schedule maybe set by a time interval, every 2 to 3 hours or at times of day such as before and after meals.

The patient is asking the nurse about the best way to stay healthy. The nurse explains to the patient which teaching points? (Select all that apply.) a. Increase physical activity. b. Keep total fat intake to 10% or less. c. Maintain body weight in a healthy range. d. Choose and prepare foods with little salt. e. Increase intake of meat and other high-protein foods.

ANS: A, C, D Recommendations include maintaining body weight in a healthy range; increasing physical activity and decreasing sedentary activities; increasing intake of fruits, vegetables, whole grain products, and fat-free or low-fat milk; eating moderate amount of lean meats, poultry, and eggs; keeping fat intake between 20% and 35% of total calories, with most fats coming from polyunsaturated or monounsaturated fatty acids (most meats contain saturated fatty acids); and choosing prepared foods with little salt while at the same time eating potassium-rich foods

A nurse is teaching a patient about the urinary system. In which order will the nurse present the structures, following the flow of urine? a. Kidney, urethra, bladder, ureters b. Kidney, ureters, bladder, urethra c. Bladder, kidney, ureters, urethra d. Bladder, kidney, urethra, ureters

ANS: B The flow of urine follows these structures: kidney, ureters, bladder, and urethra.

When assessing patient with nutritional needs, which patients will require follow-up from the nurse? (Select all that apply.) a. A patient with infection taking tetracycline with milk b. A patient with irritable bowel syndrome increasing fiber c. A patient with diverticulitis following a high-fiber diet daily d. A patient with an enteral feeding and 500 mL of gastric residual e. A patient with dysphagia being referred to a speech-language pathologist

ANS: A, C, D The nurse should follow up with the tetracycline, diverticulitis, and enteral feeding. Tetracycline has decreased drug absorption with milk and antacids and has decreased nutrient absorption of calcium from binding. Nutritional treatment for diverticulitis includes a moderate- or low-residue diet until the infection subsides. Afterward, prescribing a high-fiber diet for chronic diverticula problems ensues. A patient with a gastric residual volume of 500 mL needs to have the feeding withheld and reassessed for tolerance to feedings. All the rest are normal and expected and do not require follow-up. Patients manage irritable bowel syndrome by increasing fiber, reducing fat, avoiding large meals, and avoiding lactose or sorbitol-containing foods for susceptible individuals. Initiate consultation with a speech-language pathologist for swallowing exercises and techniques to improve swallowing and reduce risk of aspiration for a patient with dysphagia.

The nurse is caring for a patient who will have both a large abdominal bandage and an abdominal binder. Which actions will the nurse take before applying the bandage and binder? (Select all that apply.) a. Cover exposed wounds. b. Mark the sites of all abrasions. c. Assess the condition of current dressings. d. Inspect the skin for abrasions and edema. e. Cleanse the area with hydrogen peroxide. f. Assess the skin at underlying areas for circulatory impairment.

ANS: A, C, D, F Before applying a bandage or a binder, the nurse has several responsibilities. The nurse would need to inspect the skin for abrasions, edema, and discoloration or exposed wound edges. The nurse also is responsible for covering exposed wounds or open abrasions with a dressing and assessing the condition of underlying dressings and changing if soiled, as well as assessing the skin of underlying areas that will be distal to the bandage. This checks for signs of circulatory impairment, so that a comparison can be made after bandages are applied. Marking the sites of all abrasions is not necessary. Although it is important for the skin to be clean, and even though it may need to be cleaned with a noncytotoxic cleanser, cleansing with hydrogen peroxide can interfere with wound healing.

To honor cultural values of patients from different ethnic/religious groups, which actions demonstrate culturally sensitive care by the nurse? (Select all that apply.) a. Allows fasting on Yom Kippur for a Jewish patient b. Allows caffeine drinks for a Mormon patient c. Serves no ham products to a Muslim patient d. Serves kosher foods to a Christian patient e. Serves no meat or fish to a Hindu patient

ANS: A, C, E The Jewish religion fasts 24 hours on Yom Kippur and must adhere to kosher food preparation methods. Hinduism requires no meats or fish. Muslims do not eat pork. Mormons do not drink caffeinated or alcoholic drinks.

The nurse is caring for a patient with a surgical incision that eviscerates. Which actions will the nurse take? (Select all that apply.) a. Place moist sterile gauze over the site. b. Gently place the organs back. c. Contact the surgical team. d. Offer a glass of water. e. Monitor for shock.

ANS: A, C, E The presence of an evisceration (protrusion of visceral organs through a wound opening) is a surgical emergency. Immediately place damp sterile gauze over the site, contact the surgical team, do not allow the patient anything by mouth (NPO), observe for signs and symptoms of shock, and prepare the patient for emergency surgery.

The nurse is obtaining a 24-hour urine specimen collection from the patient. Which actions should the nurse take? (Select all that apply.) a. Keeping the urine collection container on ice when indicated b. Withholding all patient medications for the day c. Irrigating the sample as needed with sterile solution d. Testing the urine sample with a reagent strip by dipping it in the urine e. Asking the patient to void and discarding that urine to start the collection

ANS: A, E When obtaining a 24-hour urine specimen, it is important to keep the urine in cool conditions, depending upon the test. The patient should be asked to void and to discard the urine before the procedure begins. Medications do not need to be held unless indicated by the provider. If properly educated about the collection procedure, the patient can maintain autonomy and perform the procedure alone, taking care to maintain the integrity of the solution. A 24-hour urine specimen is not tested with a reagent strip.

A nurse is teaching a community health promotion class and discusses the flu vaccine. Which information will the nurse include in the teaching session? (Select all that apply.) a. It is given yearly. b. It is given in a series of four doses. c. It is safe for children allergic to eggs. d. It is safe for adults with acute febrile illnesses. e. The nasal spray is given to people over 50. f. The inactivated flu vaccine is given to people over 50.

ANS: A, F Annual (yearly) flu vaccines are recommended for all people 6 months and older. The inactivated flu vaccine should be given to these individuals with chronic health problems and those 50 and older. People with a known hypersensitivity to eggs or other components of the vaccine should consult their health care provider before being vaccinated. There is a flu vaccine made without egg proteins that is approved for adults 18 years of age and older. Adults with an acute febrile illness should schedule the vaccination after they have recovered. The live, attenuated nasal spray vaccine is given to people from 2 through 49 years of age if they are not pregnant or do not have certain long-term health problems such as asthma; heart, lung, or kidney disease; diabetes; or anemia.

The nurse is concerned about pulmonary aspiration when providing the patient with an intermittent tube feeding. Which action is the priority? a. Observe the color of gastric contents. b. Verify tube placement before feeding. c. Add blue food coloring to the enteral formula. d. Run the formula over 12 hours to decrease overload.

ANS: B A major cause of pulmonary aspiration is regurgitation of formula. The nurse needs to verify tube placement and elevate the head of the bed 30 to 45 degrees during feedings and for 2 hours afterward. While observing the color of gastric contents is a component, it is not the priority component; pH is the primary component. The addition of blue food coloring to enteral formula to assist with detection of aspirate is no longer used. Do not hang formula longer than 4 to 8 hours. Formula becomes a medium for bacterial growth after that length of time.

The nurse determines that an older-adult patient is at risk for infection due to decreased immunity. Which plan of care best addresses the prevention of infection for the patient? a. Inform the patient of the importance of finishing the entire dose of antibiotics. b. Encourage the patient to stay up-to-date on all vaccinations. c. Schedule patient to get annual tuberculosis skin testing. d. Create an exercise routine to run 45 minutes every day.

ANS: B A nursing care plan for preventative health measures should be reasonable and feasible. Keeping up-to-date on vaccinations is important because vaccine reduces the severity of illnesses and serious complications. Determine if and when the patient has had a pneumococcal or influenza (flu) vaccine. This is especially important when assessing older adults because of their increased risk for respiratory disease. Although it is important to finish the full course of antibiotics, it is not a preventative health measure. Scheduling annual tuberculosis skin tests does not address prevention and is an unreliable indictor of tuberculosis in older patients. The exercise routine should be reasonable to increase compliance; exercise is recommended only 3 to 4 times a week for 30 to 60 minutes, and walking, rather than running, is an efficient method.

The nurse anticipates a suprapubic catheter for which patient? a. A patient with recent prostatectomy b. A patient with a urethral stricture c. A patient with an appendectomy d. A patient with menopause

ANS: B A patient with a urethral stricture is most likely to have a suprapubic catheter. Suprapubic catheters are placed when there is blockage of the urethra (e.g., enlarged prostate, urethral stricture, after urological surgery). A patient with a recent prostatectomy indicates the enlarged prostate was removed and would not need a suprapubic catheter; however, continuous bladder irrigation may be needed. Appendectomies and menopause do not require a suprapubic catheter.

A nurse is caring for a patient who has had diarrhea for the past week. Which additional assessment finding will the nurse expect? a. Distended abdomen b. Decreased skin turgor c. Increased energy levels d. Elevated blood pressure

ANS: B Chronic diarrhea can result in dehydration. Patients with chronic diarrhea are dehydrated with decreased skin turgor and blood pressure. Diarrhea also causes loss of electrolytes, nutrients, and fluid, which decreases energy levels. A distended abdomen could indicate constipation.

The nurse is caring for a patient in the burn unit. Which type of wound healing will the nurse consider when planning care for this patient? a. Partial-thickness repair b. Secondary intention c. Tertiary intention d. Primary intention

ANS: B A wound involving loss of tissue such as a burn or a pressure ulcer or laceration heals by secondary intention. The wound is left open until it becomes filled with scar tissue. It takes longer for a wound to heal by secondary intention; thus the chance of infection is greater. A clean surgical incision is an example of a wound with little loss of tissue that heals by primary intention. The skin edges are approximated or closed, and the risk for infection is low. Partial-thickness repair is done on partial-thickness wounds that are shallow, involving loss of the epidermis and maybe partial loss of the dermis. These wounds heal by regeneration because the epidermis regenerates. Tertiary intention is seen when a wound is left open for several days, and then the wound edges are approximated. Wound closure is delayed until the risk of infection is resolved.

A patient presents to the emergency department with a laceration of the right forearm caused by a fall. After determining that the patient is stable, what is the next best step for the nurse to take? a. Inspect the wound for foreign bodies. b. Inspect the wound for bleeding. c. Determine the size of the wound. d. Determine the need for a tetanus antitoxin injection.

ANS: B After determining that a patient's condition is stable, inspect the wound for bleeding. An abrasion will have limited bleeding, a laceration can bleed more profusely, and a puncture wound bleeds in relation to the size and depth of the wound. Address any bleeding issues. Inspect the wound for foreign bodies; traumatic wounds are dirty and may need to be addressed. Determine the size of the wound. A large open wound may expose bone or tissue and be protected, or the wound may need suturing. When the wound is caused by a dirty penetrating object, determine the need for a tetanus vaccination.

The nurse needs to closely monitor the oxygen status of an older-adult patient undergoing anesthesia because of which age-related change? a. Thinner heart valves cause lipid accumulation and fibrosis. b. Diminished respiratory muscle strength may cause poor chest expansion. c. Alterations in mental status prevent patients' awareness of ineffective breathing. d. An increased number of pacemaker cells make proper anesthesia induction more difficult.

ANS: B Age-related changes in the thorax that occur from ossification of costal cartilage, decreased space between vertebrae, and diminished respiratory muscle strength lead to problems with chest expansion and oxygenation,whereby the patient will have difficulty excreting anesthesia gas. The nurse needs to monitor the patient's oxygen status carefully to make sure the patient does not retain too much of the drug. Older adults experience alterations in cardiac function as a result of calcification of the conduction pathways, thicker and stiffer heart valves caused by lipid accumulation and fibrosis, and a decrease in the number of pacemaker cells in the SA node. Altered mental status is not a normal age-related change; it indicates possible cardiac and/or respiratory problems.

The nurse is caring for a group of patients. Which task can the nurse delegate to the nursing assistive personnel? a. Assessing a surgical patient for risk of pressure ulcers b. Applying an elastic bandage to a medical-surgical patient c. Treating a pressure ulcer on the buttocks of a medical patient d. Implementing negative-pressure wound therapy on a stable patient

ANS: B Applying an elastic bandage to a medical-surgical patient can be delegated to the nursing assistive personnel (NAP). Assessing pressure ulcer risk, treating a pressure ulcer, and implementing negative-pressure wound therapy cannot be delegated to an NAP.

A nurse teaches a patient about atelectasis. Which statement by the patient indicates an understanding of atelectasis? a. "Atelectasis affects only those with chronic conditions such as emphysema." b. "It is important to do breathing exercises every hour to prevent atelectasis." c. "If I develop atelectasis, I will need a chest tube to drain excess fluid." d. "Hyperventilation will open up my alveoli, preventing atelectasis."

ANS: B Atelectasis develops when alveoli do not expand. Breathing exercises, especially deep breathing and incentive spirometry, increase lung volume and open the airways, preventing atelectasis. Deep breathing also opens the pores of Kohn between alveoli to allow sharing of oxygen between alveoli. Atelectasis can affect anyone who does not deep breathe. A chest tube is for pneumothorax or hemothorax. It is deep breathing, not hyperventilation, that prevents atelectasis.

The nurse is emptying an ileostomy pouch for a patient. Which assessment finding will the nurse report immediately? a. Liquid consistency of stool b. Presence of blood in the stool c. Malodorous stool d. Continuous output from the stoma

ANS: B Blood in the stool indicates a problem, and the health care provider should be notified. All other options are expected findings for an ileostomy. The stool should be liquid, there should be an odor, and the output should be continuous.

The home health nurse recommends that a patient with respiratory problems install a carbon monoxide detector in the home. What is the rationale for the nurse's action? a. Carbon monoxide detectors are required by law in the home. b. Carbon monoxide tightly binds to hemoglobin, causing hypoxia. c. Carbon monoxide signals the cerebral cortex to cease ventilations. d. Carbon monoxide combines with oxygen in the body and produces a deadly toxin.

ANS: B Carbon monoxide binds tightly to hemoglobin; therefore, oxygen is not able to bind to hemoglobin and be transported to tissues, causing hypoxia. A carbon monoxide detector is not required by law, does not signal the cerebral cortex to cease ventilations, and does not combine with oxygen but with hemoglobin to produce a toxin.

The nurse is caring for a patient with a healing Stage III pressure ulcer. The wound is clean and granulating. Which health care provider's order will the nurse question? a. Use a low-air-loss therapy unit. b. Irrigate with Dakin's solution. c. Apply a hydrogel dressing. d. Consult a dietitian.

ANS: B Clean pressure ulcers with noncytotoxic cleansers such as normal saline, which will not kill fibroblasts and healing tissue. Cytotoxic cleansers such as Dakin's solution, acetic acid, povidone-iodine, and hydrogen peroxide can hinder the healing process and should not be utilized on clean granulating wounds. Consulting a dietitian for the nutritional needs of the patient, utilizing a low-air-loss therapy unit to decrease pressure, and applying hydrogel dressings to provide a moist environment for healing are all orders that would be appropriate.

The nurse is preparing to check the gastric aspirate for pH. Which equipment will the nurse obtain? a. 10-mL Luer-Lok syringe b. Asepto syringe c. Sterile gloves d. Double gloves

ANS: B Cone-tipped or Asepto syringe is needed for testing of gastric aspirate for pH; these syringes are better than a Luer-Lok syringe. Clean gloves are needed, not sterile or double.

The nurse is educating a student nurse on caring for a patient with a chest tube. Which statement from the student nurse indicates successful learning? a. "I should clamp the chest tube when giving the patient a bed bath." b. "I should report if I see continuous bubbling in the water-seal chamber." c. "I should strip the drains on the chest tube every hour to promote drainage." d. "I should notify the health care provider first, if the chest tube becomes dislodged."

ANS: B Correct care of a chest tube involves knowing normal and abnormal functioning of the tube. A constant or intermittent bubbling in the water-seal chamber indicates a leak in the drainage system, and the health care provider must be notified immediately. Stripping the tube is not routinely performed as it increases pressure. If the tubing disconnects from the drainage unit, instruct the patient to exhale as much as possible and to cough. This maneuver rids the pleural space of as much air as possible. Temporarily reestablish a water seal by immersing the open end of the chest tube into a container of sterile water. The chest tube should not be clamped unless necessary; if so, the length of time clamped would be minimal to reduce the risk of pneumothorax.

The nurse, upon reviewing the history, discovers the patient has dysuria. Which assessment finding is consistent with dysuria? a. Blood in the urine b. Burning upon urination c. Immediate, strong desire to void d. Awakes from sleep due to urge to void

ANS: B Dysuria is burning or pain with urination. Hematuria is blood in the urine. Urgency is an immediate and strong desire to void that is not easily deferred. Nocturia is awakening form sleep due to urge to void.

. The nurse is caring for a patient with a wound. The patient appears anxious as the nurse is preparing to change the dressing. Which action should the nurse take? a. Turn on the television. b. Explain the procedure. c. Tell the patient "Close your eyes." d. Ask the family to leave the room.

ANS: B Explaining the procedure educates the patient regarding the dressing change and involves him in the care, thereby allowing the patient some control in decreasing anxiety. Telling the patient to close the eyes and turning on the television are distractions that do not usually decrease a patient's anxiety. If the family is a support system, asking support systems to leave the room can actually increase a patient's anxiety.

The nurse is caring for a patient who has a Stage IV pressure ulcer with grafted surgical sites. Which specialty bed will the nurse use for this patient? a. Low-air-loss b. Air-fluidized c. Lateral rotation d. Standard mattress

ANS: B For a patient with newly flapped or grafted surgical sites, the air-fluidized bed will be the best choice; this uses air and fluid support to provide pressure redistribution via a fluid-like medium created by forcing air through beads as characterized by immersion and envelopment. A low-air-loss bed is utilized for prevention or treatment of skin breakdown by preventing buildup of moisture and skin breakdown through the use of airflow. A standard mattress is utilized for an individual who does not have actual or potential altered or impaired skin integrity. Lateral rotation is used for treatment and prevention of pulmonary, venous stasis and urinary complications associated with mobility.

A patient has a decreased gag reflex, left-sided weakness, and drooling. Which action will the nurse take when feeding this patient? a. Position in semi-Fowler's. b. Flex head with chin tuck. c. Place food on left side. d. Offer fruit juice.

ANS: B Have the patient flex the head slightly to a chin-down position to help prevent aspiration. If the patient has unilateral weakness, teach him or her and the caregiver to place food in the stronger side of the mouth. Provide a 30-minute rest period before eating and position the patient in an upright, seated position in a chair or raise the head of the bed to 90 degrees. Thin liquids such as water and fruit juice are difficult to control in the mouth and are more easily aspirated.

The nurse is caring for a patient with a tracheostomy tube. Which nursing intervention is most effective in promoting effective airway clearance? a. Suctioning respiratory secretions several times every hour b. Administering humidified oxygen through a tracheostomy collar c. Instilling normal saline into the tracheostomy to thin secretions before suctioning d. Deflating the tracheostomy cuff before allowing the patient to cough up secretions

ANS: B Humidification from air humidifiers or humidified oxygen tracheostomy collars can help prevent drying of secretions that cause occlusion. Suctioning should be done only as needed; too frequent suctioning can damage the mucosal lining, resulting in thicker secretions. Normal saline should not be instilled into a tracheostomy; research showed no benefit with this technique. The purpose of the tracheostomy cuff is to keep secretions from entering the lungs; the nurse should not deflate the tracheostomy cuff unless instructed to do so by the health care provider.

Before administering a cleansing enema to an 80-year-old patient, the patient says "I don't think I will be able to hold the enema." Which is the next priority nursing action? a. Rolling the patient into right-lying Sims' position b. Positioning the patient in the dorsal recumbent position on a bedpan c. Inserting a rectal plug to contain the enema solution after administering d. Assisting the patient to the bedside commode and administering the enema

ANS: B If you suspect the patient of having poor sphincter control, position on bedpan in a comfortable dorsal recumbent position. Patients with poor sphincter control are unable to retain all of the enema solution. Administering an enema with the patient sitting on the toilet is unsafe because it is impossible to safely guide the tubing into the rectum, and it will be difficult for the patient to retain the fluid as he or she is in the position used for emptying the bowel. Rolling the patient into right-lying Sims' position will not help the patient retain the enema. Use of a rectal plug to contain the solution is inappropriate and unsafe.

The nurse is preparing to test a patient for postvoid residual with a bladder scan. Which action will the nurse take? a. Measure bladder before the patient voids. b. Measure bladder within 10 minutes after the patient voids. c. Measure bladder with head of bed raised to 60 degrees. d. Measure bladder with head of bed raised to 90 degrees.

ANS: B Measurement should be within 10 minutes of voiding. It is a postvoid so the measurement is after the patient voids and the urine volume is recorded. Patient is supine with head slightly elevated.

A nurse is caring for a patient with a wound. Which assessment data will be most important for the nurse to gather with regard to wound healing? a. Muscular strength assessment b. Pulse oximetry assessment c. Sensation assessment d. Sleep assessment

ANS: B Oxygen fuels the cellular functions essential to the healing process; the ability to perfuse tissues with adequate amounts of oxygenated blood is critical in wound healing. Pulse oximetry measures the oxygen saturation of blood. Assessment of muscular strength and sensation, although useful for fitness and mobility testing, does not provide any data with regard to wound healing. Sleep, although important for rest and for integration of learning and restoration of cognitive function, does not provide any data with regard to wound healing.

Which statement by the patient about an upcoming contrast computed tomography (CT) scan indicates a need for further teaching? a. "I will follow the food and drink restrictions as directed before the test is scheduled." b. "I will be anesthetized so that I lie perfectly still during the procedure." c. "I will complete my bowel prep program the night before the scan." d. "I will be drinking a lot of fluid after the test is over."

ANS: B Patients are not put under anesthesia for a CT scan; instead, the nurse should educate patients about the need to lie perfectly still and about possible methods of overcoming feelings of claustrophobia. The other options are correct and require no further teaching. Patients need to be assessed for an allergy to shellfish if receiving contrast for the CT scan. Bowel cleansing is often performed before CT scan. Another area to address is food and fluid restriction up to 4 hours prior to the test. After the procedure, encourage fluids to promote dye excretion.

The patient is admitted with facial trauma, including a broken nose, and has a history of esophageal reflux and of aspiration pneumonia. With which tube will the nurse most likely administer the feeding? a. Nasogastric tube b. Jejunostomy tube c. Nasointestinal tube d. Percutaneous endoscopic gastrostomy (PEG) tube

ANS: B Patients with gastroparesis or esophageal reflux or with a history of aspiration pneumonia may require placement of tubes beyond the stomach into the intestine. The jejunostomy tube is the only tube in the list that is beyond the stomach and is not contraindicated by facial trauma. The nasogastric tube and the PEG tube are placed in the stomach, and placement could lead to aspiration. The nasointestinal tube and the nasogastric tube may be contraindicated by facial trauma and the broken nose.

A nurse is pouching an ostomy on a patient with an ileostomy. Which action by the nurse is most appropriate? a. Changing the skin barrier portion of the ostomy pouch daily b. Emptying the pouch if it is more than one-third to one-half full c. Thoroughly cleansing the skin around the stoma with soap and water to remove excess stool and adhesive d. Measuring the correct size for the barrier device while leaving a 1/2-inch space around the stoma

ANS: B Pouches must be emptied when they are one-third to one-half full because the weight of the pouch may disrupt the seal of the adhesive on the skin. The barrier device should be changed every 3 to 7 days unless it is leaking or is no longer effective. Peristomal skin should be gently cleansed; vigorous rubbing can cause further irritation or skin breakdown. Avoid soap. It leaves a residue on skin, which may irritate the skin. The pouch opening should fit around the stoma and cover the peristomal skin to prevent contact with the effluent. Excess space, like 1/2 inch, allows fecal matter to have prolonged exposure to skin, resulting in skin breakdown.

The nurse is caring for a patient who was involved in an automobile accident 2 weeks ago. The patient sustained a head injury and is unconscious. Which priority element will the nurse consider when planning care to decrease the development of a decubitus ulcer? a. Resistance b. Pressure c. Weight d. Stress

ANS: B Pressure is the main element that causes pressure ulcers. Three pressure-related factors contribute to pressure ulcer development: pressure intensity, pressure duration, and tissue tolerance. When the intensity of the pressure exerted on the capillary exceeds 15 to 32 mm Hg, this occludes the vessel, causing ischemic injury to the tissues it normally feeds. High pressure over a short time and low pressure over a long time cause skin breakdown. Resistance, stress, and weight are not the priority causes of pressure ulcers.

The nurse is collaborating with the dietitian about a patient with a Stage III pressure ulcer. Which nutrient will the nurse most likely increase after collaboration with the dietitian? a. Fat b. Protein c. Vitamin E d. Carbohydrate

ANS: B Protein needs are especially increased in supporting the activity of wound healing. The physiological processes of wound healing depend on the availability of protein, vitamins (especially A and C), and the trace minerals of zinc and copper. Wound healing does not require increased amounts of fats or carbohydrates. Vitamin E will not be increased for wound healing.

The medical-surgical acute care patient has received a nursing diagnosis of Impaired skin integrity. Which health care team member will the nurse consult? a. Respiratory therapist b. Registered dietitian c. Case manager d. Chaplain

ANS: B Refer patients with pressure ulcers to the dietitian for early intervention for nutritional problems. Adequate calories, protein, vitamins, and minerals promote wound healing for the impaired skin integrity. The nurse is the coordinator of care, and collaborating with the dietitian would result in planning the best meals for the patient. The respiratory therapist can be consulted when a patient has issues with the respiratory system. Case management can be consulted when the patient has a discharge need. A chaplain can be consulted when the patient has a spiritual need.

A nurse is caring for a patient who was in a motor vehicle accident that resulted in cervical trauma to C4. Which assessment is the priority? a. Pulse b. Respirations c. Temperature d. Blood pressure

ANS: B Respirations and oxygen saturation are the priorities. Cervical trauma at C3 to C5 usually results in paralysis of the phrenic nerve. When the phrenic nerve is damaged, the diaphragm does not descend properly, thus reducing inspiratory lung volumes and causing hypoxemia. While pulse and blood pressure are important, respirations are the priority. Temperature is not a high priority in this situation.

The nurse is caring for a patient with a Stage IV pressure ulcer. Which type of healing will the nurse consider when planning care for this patient? a. Partial-thickness wound repair b. Full-thickness wound repair c. Primary intention d. Tertiary intention

ANS: B Stage IV pressure ulcers are full-thickness wounds that extend into the dermis and heal by scar formation because the deeper structures do not regenerate, hence the need for full-thickness repair. The full-thickness repair has four phases: hemostasis, inflammatory, proliferative, and maturation. A wound heals by primary intention when wounds such as surgical wounds have little tissue loss; the skin edges are approximated or closed, and the risk for infection is low. Partial-thickness repairs are done on partial-thickness wounds that are shallow, involving loss of the epidermis and maybe partial loss of the dermis. These wounds heal by regeneration because the epidermis regenerates. Tertiary intention is seen when a wound is left open for several days, and then the wound edges are approximated. Wound closure is delayed until risk of infection is resolved.

Which patient is most at risk for increased peristalsis? a. A 5-year-old child who ignores the urge to defecate owing to embarrassment b. A 21-year-old female with three final examinations on the same day c. A 40-year-old female with major depressive disorder d. An 80-year-old male in an assisted-living environment

ANS: B Stress can stimulate digestion and increase peristalsis, resulting in diarrhea; three finals on the same day is stressful. Ignoring the urge to defecate, depression, and age-related changes of the older adult (80-year-old man) are causes of constipation, which is from slowed peristalsis.

. The nurse is suctioning a patient with a tracheostomy tube. Which action will the nurse take? a. Set suction regulator at 150 to 200 mm Hg. b. Limit the length of suctioning to 10 seconds. c. Apply suction while gently rotating and inserting the catheter. d. Liberally lubricate the end of the suction catheter with a water-soluble solution.

ANS: B Suctioning passes should be limited to 10 seconds to avoid hypoxemia. Suction for a tracheostomy should be set at 100 to 150 mm Hg. Excessive lubrication can clog the catheter or occlude the airway; lubricant is not necessary for oropharyngeal or artificial airway (tracheostomy) suctioning. Suction should never be applied on insertion.

. The nurse collects the following assessment data: right heel with reddened area that does not blanch. Which nursing diagnosis will the nurse assign to this patient? a. Imbalanced nutrition: less than body requirements b. Ineffective peripheral tissue perfusion c. Risk for infection d. Acute pain

ANS: B The area on the heel has experienced a decreased supply of blood and oxygen (tissue perfusion), which has resulted in tissue damage. The most appropriate nursing diagnosis with this information is Ineffective peripheral tissue perfusion. Risk for infection, Acute pain, and Imbalancednutrition do not support the data in the question.

A nurse is teaching about the energy needed at rest to maintain life-sustaining activities for a specific period of time. What is the nurse discussing? a. Resting energy expenditure (REE) b. Basal metabolic rate (BMR) c. Nutrient density d. Nutrients

ANS: B The basal metabolic rate (BMR) is the energy needed at rest to maintain life-sustaining activities for a specific period of time. The resting energy expenditure (REE), or resting metabolic rate, is the amount of energy an individual needs to consume over a 24-hour period for the body to maintain all of its internal working activities while at rest. Nutrients are the elements necessary for body processes and function. Nutrient density is the proportion of essential nutrients to the number of kilocalories. High-nutrient density foods provide a large number of nutrients in relation to kilocalories.

Which nursing intervention is most effective in promoting normal defecation for a patient who has muscle weakness in the legs? a. Administer a soapsuds enema every 2 hours. b. Use a mobility device to place the patient on a bedside commode. c. Give the patient a pillow to brace against the abdomen while bearing down. d. Elevate the head of the bed 20 degrees 60 minutes after breakfast while on bedpan.

ANS: B The best way to promote normal defecation is to assist the patient into a posture that is as normal as possible for defecation. Using a mobility device promotes nurse and patient safety. Elevating the head of the bed is appropriate but is not the most effective; closer to 30 to 45 degrees is the proper position for the patient on a bedpan, and the patient is not on bed rest so a bedside commode is the best choice. Giving the patient a pillow may reduce discomfort, but this is not the best way to promote defecation. A soapsuds enema is indicated for a patient who needs assistance to stimulate peristalsis. It promotes non-natural defecation.

A nurse is teaching staff about the conduction of the heart. In which order will the nurse present the conduction cycle, starting with the first structure? 1. Bundle of His 2. Purkinje network 3. Intraatrial pathways 4. Sinoatrial (SA) node 5. Atrioventricular (AV) node a. 5, 4, 3, 2, 1 b. 4, 3, 5, 1, 2 c. 4, 5, 3, 1, 2 d. 5, 3, 4, 2, 1

ANS: B The conduction system originates with the SA node, the "pacemaker" of the heart. The electrical impulses are transmitted through the atria along intraatrial pathways to the AV node. It assists atrial emptying by delaying the impulse before transmitting it through the Bundle of His and the ventricular Purkinje network.

. A nurse is caring for a patient with left-sided hemiparesis who has developed bronchitis and has a heart rate of 105 beats/min, blood pressure of 156/90 mm Hg, and respiration rate of 30 breaths/min. Which nursing diagnosis is a priority? a. Risk for skin breakdown b. Impaired gas exchange c. Activity intolerance d. Risk for infection

ANS: B The most important nursing intervention is to maintain airway and circulation for this patient; therefore, Impaired gas exchange is the first nursing priority. Activity intolerance is a concern but is not the priority in this case. Risk for skin breakdown and Risk for infection are also important but do not address an immediate impairment with physiologic integrity.

A nurse is providing care to a group of patients. Which patient will the nurse see first? a. A patient who is dribbling small amounts on the way to the bathroom and has a diagnosis of urge incontinence b. A patient with reflex incontinence with elevated blood pressure and pulse rate c. A patient with an indwelling catheter that has stool on the catheter tubing d. A patient who has just voided and needs a postvoid residual test

ANS: B The nurse should see the patient with reflex incontinence first. Patients with reflex incontinence are at risk for developing autonomic dysreflexia, a life-threatening condition that causes severe elevation of blood pressure and pulse rate and diaphoresis. This is a medical emergency requiring immediate intervention; notify the health care provider immediately. A patient with urge incontinence will dribble, and this is expected. While a patient with a catheter and stool on the tubing does need to be cleaned, it is not life threatening. The nurse has 10 minutes before checking on the patient who has a postvoid residual test.

A nurse is caring for a group of patients. Which patient will the nurse see first? a. Patient receiving total parenteral nutrition of 2-in-1 for 50 hours b. Patient receiving total parenteral nutrition infusing with same tubing for 26 hours c. Patient receiving continuous enteral feeding with same feeding bag for 12 hours d. Patient receiving continuous enteral feeding with same tubing for 24 hours

ANS: B The nurse should see the patient with total parenteral nutrition that has the same tubing for 26 hours. To prevent infection, change the TPN infusion tubing every 24 hours. Change the administration system every 72 hours when infusing a 2-in-1 solution and every 24 hours for a 3-in-1 solution. Change bag and use a new administration set every 24 hours for a continuous enteral feeding. While the patient with the continuous enteral feeding has the same tubing for 24 hours, it has not extended the time like the total parenteral nutrition has.

While the nurse is changing the ties on a tracheostomy collar, the patient coughs, dislodging the tracheostomy tube. Which action will the nurse take first? a. Press the emergency response button. b. Insert a spare tracheostomy with the obturator. c. Manually occlude the tracheostomy with sterile gauze. d. Place a face mask delivering 100% oxygen over the nose and mouth.

ANS: B The nurse's first priority is to establish a stable airway by inserting a spare trach into the patient's airway; ideally an obturator should be used. The nurse could activate the emergency response team if the patient is still unstable after the tracheostomy is placed. A patient with a tracheostomy breathes through the tube, not the nose or mouth; a face mask would not be an effective method of getting air into the lungs. Manually occluding pressure over the tracheostomy site is not appropriate and would block the patient's only airway.

The nurse is performing a moist-to-dry dressing. The nurse has prepared the supplies, solution, and removed the old dressing. In which order will the nurse implement the steps, starting with the first one? 1. Apply sterile gloves. 2. Cover and secure topper dressing. 3. Assess wound and surrounding skin. 4. Moisten gauze with prescribed solution. 5. Gently wring out excess solution and unfold. 6. Loosely pack until all wound surfaces are in contact with gauze. a. 4, 3, 1, 5, 6, 2 b. 1, 3, 4, 5, 6, 2 c. 4, 1, 3, 5, 6, 2 d. 1, 4, 3, 5, 6, 2

ANS: B The steps for a moist-to-dry dressing are as follows: (1) Apply sterile gloves; (2) assess appearance of surrounding skin; (3) moisten gauze with prescribed solution. (4) Gently wring out excess solution and unfold; apply gauze as single layer directly onto wound surface. (5) If wound is deep, gently pack dressing into wound base by hand until all wound surfaces are in contact with gauze; (6) cover with sterile dry gauze and secure topper dressing.

The nurse is admitting an older patient from a nursing home. During the assessment, the nurse notes a shallow open reddish, pink ulcer without slough on the right heel of the patient. How will the nurse stage this pressure ulcer? a. Stage I b. Stage II c. Stage III d. Stage IV

ANS: B This would be a Stage II pressure ulcer because it presents as partial-thickness skin loss involving epidermis and dermis. The ulcer presents clinically as an abrasion, blister, or shallow crater. Stage I is intact skin with nonblanchable redness over a bony prominence. With a Stage III pressure ulcer, subcutaneous fat may be visible, but bone, tendon, and muscles are not exposed. Stage IV involves full-thickness tissue loss with exposed bone, tendon, or muscle

A female patient is having difficulty voiding in a bedpan but states that her bladder feels full. To stimulate micturition, which nursing intervention should the nurse try first? a. Exiting the room and informing the patient that the nurse will return in 30 minutes to check on the patient's progress. b. Utilizing the power of suggestion by turning on the faucet and letting the water run. c. Obtaining an order for a Foley catheter. d. Administering diuretic medication.

ANS: B To stimulate micturition, the nurse should attempt noninvasive procedures first. Running warm water or stroking the inner aspect of the upper thigh promotes sensory perception that leads to urination. A patient should not be left alone on a bedpan for 30 minutes because this could cause skin breakdown. Catheterization places the patient at increased risk of infection and should not be the first intervention attempted. Diuretics are useful if the patient is not producing urine, but they do not stimulate micturition

An 86-year-old patient is experiencing uncontrollable leakage of urine with a strong desire to void and even leaks on the way to the toilet. Which priority nursing diagnosis will the nurse include in the patient's plan of care? a. Functional urinary incontinence b. Urge urinary incontinence c. Impaired skin integrity d. Urinary retention

ANS: B Urge urinary incontinence is the leakage of urine associated with a strong urge to void. Patients leak urine on the way to or at the toilet and rush or hurry to the toilet. Urinary retention is the inability to empty the bladder. Functional urinary incontinence is incontinence due to causes outside the urinary tract, such as mobility or cognitive impairments. While Impaired skin integritycan occur, it is not the priority at this time, and there is no data to support this diagnosis.

While receiving a shift report on a patient, the nurse is informed that the patient has urinary incontinence. Upon assessment, which finding will the nurse expect? a. An indwelling Foley catheter b. Reddened irritated skin on buttocks c. Tiny blood clots in the patient's urine d. Foul-smelling discharge indicative of infection

ANS: B Urinary incontinence is uncontrolled urinary elimination; if the urine has prolonged contact with the skin, skin breakdown can occur. An indwelling Foley catheter is a solution for urine retention. Blood clots and foul-smelling discharge are often signs of infection.

The nurse has attempted to administer a tap water enema for a patient with fecal impaction with no success. The fecal mass is too large for the patient to pass voluntarily. Which is the nextpriority nursing action? a. Preparing the patient for a second tap water enema b. Obtaining an order for digital removal of stool c. Positioning the patient on the left side d. Inserting a rectal tube

ANS: B When enemas are not successful, digital removal of the stool may be necessary to break up pieces of the stool or to stimulate the anus to defecate. Tap water enemas should not be repeated because of risk of fluid imbalance. Positioning the patient on the left side does not promote defecation. A rectal tube is indicated for a patient with liquid stool incontinence or flatus but would not be applicable or effective for this patient.

. The nurse is caring for a patient who is at risk for skin impairment. The patient is able to sit up in a chair. The nurse includes this intervention in the plan of care. How long should the nurse schedule the patient to sit in the chair? a. At least 3 hours b. Less than 2 hours c. No longer than 30 minutes d. As long as the patient remains comfortable

ANS: B When patients are able to sit up in a chair, make sure to limit the amount of time to 2 hours or less. The chair sitting time should be individualized. In the sitting position, pressure on the ischial tuberosities is greater than in a supine position. Utilize foam, gel, or an air cushion to distribute weight. Sitting for longer than 2 hours can increase the chance of ischemia.

Upon auscultation of the patient's chest, the nurse hears a whooshing sound at the fifth intercostal space. What does this finding indicate to the nurse? a. The beginning of the systolic phase b. Regurgitation of the mitral valve c. The opening of the aortic valve d. Presence of orthopnea

ANS: B When regurgitation occurs, there is a backflow of blood into an adjacent chamber. For example, in mitral regurgitation the mitral leaflets do not close completely. When the ventricles contract, blood escapes back into the atria, causing a murmur, or "whooshing" sound. The systolic phase begins with ventricular filling and closing of the aortic valve, which is heard as the first heart sound, S1. Orthopnea is an abnormal condition in which a patient uses multiple pillows when reclining to breathe easier or sits leaning forward with arms elevated.

The nurse is caring for a patient who is immobile and is at risk for skin impairment. The plan of care includes turning the patient. Which is the best method for repositioning the patient? a. Place the patient in a 30-degree supine position. b. Utilize a transfer device to lift the patient. c. Elevate the head of the bed 45 degrees. d. Slide the patient into the new position.

ANS: B When repositioning the patient, obtain assistance and utilize a transfer device to lift rather than drag the patient. Sliding the patient into the new position will increase friction. The patient should be placed in a 30-degree lateral position, not a supine position. The head of the bed should be elevated less than 30 degrees to prevent pressure ulcer development from shearing forces.

The nurse is teaching a health class about the ChooseMyPlate program. Which guidelines will the nurse include in the teaching session? a. Balancing sodium and potassium b. Decreasing water consumption c. Increasing portion size d. Balancing calories

ANS: D The ChooseMyPlate program includes guidelines for balancing calories; decreasing portion size; increasing healthy foods; increasing water consumption; and decreasing fats, sodium, and sugars. It does not balance sodium and potassium.

Which findings should the nurse follow up on after removal of a catheter from a patient? (Select all that apply.) a. Increasing fluid intake b. Dribbling of urine c. Voiding in small amounts d. Voiding within 6 hours of catheter removal e. Burning with the first couple of times voiding

ANS: B, C Abdominal pain and distention, a sensation of incomplete emptying, incontinence, constant dribbling of urine, and voiding in very small amounts can indicate inadequate bladder emptying requiring intervention. All the rest are normal and do not require follow-up. The patient should increase intake. The first few times a patient voids after catheter removal may be accompanied by some discomfort, but continued complaints of painful urination indicate possible infection. Patient should void 6 to 8 hours after catheter removal.

A nurse administers an antimuscarinic to a patient. Which findings indicate the patient is having therapeutic effects from this medication? (Select all that apply.) a. Decrease in dysuria b. Decrease in urgency c. Decrease in frequency d. Decrease in prostate size e. Decrease in bladder infection

ANS: B, C When newly started on an antimuscarinic, you should monitor the patient for effectiveness, watching for a decrease in symptoms such as urgency, frequency, and urgency urinary incontinence episodes. Patients with painful urination are sometimes prescribed urinary analgesics that act on the urethral and bladder mucosa (e.g., phenazopyridine). Antibiotics are used to treat bladder infections. Agents that shrink the prostate include finasteride and dutasteride.

The nurse is caring for a patient with potential skin breakdown. Which components will the nurse include in the skin assessment? (Select all that apply.) a. Vision b. Hyperemia c. Induration d. Blanching e. Temperature of skin

ANS: B, C, D, E Assessment of the skin includes both visual and tactile inspection. Assess for hyperemia and palpate for blanching or nonblaching. Early signs of skin damage include induration, bogginess (less-than-normal stiffness), and increased warmth at the injury site compared to nearby areas. Changes in temperature can indicate changes in blood flow to that area of the skin. Vision is not included in the skin assessment.

The nurse is updating the plan of care for a patient with impaired skin integrity. Which findings indicate achievement of goals and outcomes? (Select all that apply.) a. The patient's expectations are not being met. b. Skin is intact with no redness or swelling. c. Nonblanchable erythema is absent. d. No injuries to the skin and tissues are evident. e. Granulation tissue is present.

ANS: B, C, D, E Optimal outcomes are to prevent injury to skin and tissues, reduce injury to skin, reduce injury to underlying tissues, and restore skin integrity. Skin intact, nonblanchable erythema absent, no injuries, and presence of granulation tissue are all findings indicating achievement of goals and outcomes. The patient's expectations not being met indicates no progression toward goals/outcomes.

Which nursing actions will the nurse implement when collecting a urine specimen from a patient? (Select all that apply.) a. Growing urine cultures for up to 12 hours b. Labeling all specimens with date, time, and initials c. Allowing the patient adequate time and privacy to void d. Wearing gown, gloves, and mask for all specimen handling e. Transporting specimens to the laboratory in a timely manner f. Collecting the specimen from the drainage bag of an indwelling catheter

ANS: B, C, E All specimens should be labeled appropriately and processed in a timely fashion. Allow patients time and privacy to void. Urine cultures can take up to 48 to 72 hours to develop. Only gloves are necessary to handle a urine specimen. Gown and mask are not needed unless otherwise indicated. Never collect the specimen from the drainage bag of a catheter; obtain the sample from the special sampling port.

A nurse is following the Ventilator Bundle standards to prevent ventilator-associated pneumonia. Which strategies is the nurse using? (Select all that apply.) a. Head of bed elevation to 90 degrees at all times b. Daily oral care with chlorhexidine c. Cuff monitoring for adequate seal d. Clean technique when suctioning e. Daily "sedation vacations" f. Heart failure prophylaxis

ANS: B, C, E The key components of the Institute for Healthcare Improvement (IHI) Ventilator Bundle are: Elevation of the head of the bed (HOB)—elevation is 30 to 45 degrees Daily "sedation vacations" and assessment of readiness to extubate Peptic ulcer disease prophylaxis Deep venous thrombosis prophylaxis Daily oral care with chlorhexidine Monitor cuff pressure frequently to ensure that there is an adequate seal to prevent aspiration of secretions is also included. Sterile technique is used for suctioning when on ventilators. Heart failure prophylaxis is not a component.

ANS: C Kayexalate binds to and helps excrete potassium, so it would be contraindicated in patients who are hypokalemic (have low potassium). Normal saline enemas can be repeated without risk of fluid or electrolyte imbalance. Hypertonic solutions are intended for patients who cannot handle large fluid volume and are contraindicated for dehydrated patients. Oil retention enemas lubricate the feces in the rectum and colon and are used for constipation.

ANS: C Abnormal findings such as a positive test (turns blue) should be reported to the provider. A fecal occult blood test is a clean procedure; sterile gloves are not needed. A thin specimen smear is all that is required. The quality control section should be developed after it is determined whether the sample is positive or negative.

The nurse will anticipate inserting a Coudé catheter for which patient? a. An 8-year-old male undergoing anesthesia for a tonsillectomy b. A 24-year-old female who is going into labor c. A 56-year-old male admitted for bladder irrigation d. An 86-year-old female admitted for a urinary tract infection

ANS: C A Coudé catheter has a curved tip that is used for patients with enlarged prostates. This would be indicated for a middle-aged male who needs bladder irrigation. Coudé catheters are not indicated for children or women.

The nurse suspects the patient has increased afterload. Which piece of equipment should the nurse obtain to determine the presence of this condition? a. Pulse oximeter b. Oxygen cannula c. Blood pressure cuff d. Yankauer suction tip catheter

ANS: C A blood pressure cuff is needed. The diastolic aortic pressure is a good clinical measure of afterload. Afterload is the resistance to left ventricular ejection. In hypertension the afterload increases, making cardiac workload also increase. A pulse oximeter is used to monitor the level of arterial oxygen saturation; it will not help determine increased afterload. While an oxygen cannula may be needed to help decrease the effects of increased afterload, it will not help determine the presence of afterload. A Yankauer suction tip catheter is used to suction the oral cavity.

. The nurse administers a cathartic to a patient. Which finding helps the nurse determine that the cathartic has a therapeutic effect? a. Reports decreased diarrhea. b. Experiences pain relief. c. Has a bowel movement. d. Passes flatulence.

ANS: C A cathartic is a laxative that stimulates a bowel movement. It would be effective if the patient experiences a bowel movement. The other options are not outcomes of administration of a cathartic. An antidiarrheal will provide relief from diarrhea. Pain medications will provide pain relief. Carminative enemas provide relief from gaseous distention (flatulence).

A patient is receiving a neomycin solution enema. Which primary goal is the nurse trying to achieve? a. Prevent gaseous distention b. Prevent constipation c. Prevent colon infection d. Prevent lower bowel inflammation

ANS: C A medicated enema is a neomycin solution, i.e., an antibiotic used to reduce bacteria in the colon before bowel surgery. Carminative enemas provide relief from gaseous distention. Bulk forming, emollient (wetting), and osmotic laxatives and cathartics help prevent constipation or treat constipation. An enema containing steroid medication may be used for acute inflammation in the lower colon.

A patient has a myocardial infarction. On which primary blood vessel will the nurse focus care to reduce ischemia? a. Superior vena cava b. Pulmonary artery c. Coronary artery d. Carotid artery

ANS: C A myocardial infarction is the lack of blood flow due to obstruction to the coronary artery, which supplies the heart with blood. The superior vena cava returns blood back to the heart. The pulmonary artery supplies deoxygenated blood to the lungs. The carotid artery supplies blood to the brain

. The nurse is caring for a patient who had a colostomy placed yesterday. The nurse should report which assessment finding immediately? a. Stoma is protruding from the abdomen. b. Stoma is flush with the skin. c. Stoma is purple. d. Stoma is moist.

ANS: C A purple stoma may indicate strangulation/necrosis or poor circulation to the stoma and may require surgical intervention. A stoma should be reddish-pink and moist in appearance. It can be flush with the skin, or it can protrude.

The patient is having lower abdominal surgery and the nurse inserts an indwelling catheter. What is the rationale for the nurse's action? a. The patient may void uncontrollably during the procedure. b. Local trauma sometimes promotes excessive urine incontinence. c. Anesthetics can decrease bladder contractility and cause urinary retention. d. The patient will not interrupt the procedure by asking to go to the bathroom.

ANS: C Anesthetic agents and other agents given during surgery can decrease bladder contractility and/or sensation of bladder fullness, causing urinary retention. Local trauma during lower abdominal and pelvic surgery sometimes obstructs urine flow, requiring temporary use of an indwelling urinary catheter. The patient is more likely to retain urine rather than experience uncontrollable voiding.

Which patient will the nurse assess most closely for an ileus? a. A patient with a fecal impaction b. A patient with chronic cathartic abuse c. A patient with surgery for bowel disease and anesthesia d. A patient with suppression of hydrochloric acid from medication

ANS: C Any surgery that involves direct manipulation of the bowel temporarily stops peristalsis. Anesthesia can also cause cessation of peristalsis. This condition, called an ileus, usually lasts about 24 to 48 hours. Fecal impaction, cathartic abuse, and medication to suppress hydrochloric acid will have bowel sounds, but they may be hypoactive or hyperactive.

The nurse is providing nutrition education to a Korean patient using the five food groups. In doing so, what should be the focus of the teaching? a. Discouraging the patient's ethnic food choices b. Changing the patient's diet to a more conventional American diet c. Including racial and ethnic practices with food preferences of the patient d. Comparing the patient's ethnic preferences with American dietary choices

ANS: C As a nurse, consider the food preferences of patients from different racial and ethnic groups, vegetarians, and others when planning diets. Initiation of a balanced diet is more important than conversion to what may be considered an American diet. Ethnic food choices may be just as nutritious as American choices. Foods should be chosen for their nutritive value and should not be compared with the American diet.

A nurse auscultates heart sounds. When the nurse hears S2, which valves is the nurse hearing close? a. Aortic and mitral b. Mitral and tricuspid c. Aortic and pulmonic d. Mitral and pulmonic

ANS: C As the ventricles empty, the ventricular pressures decrease, allowing closure of the aortic and pulmonic valves, producing the second heart sound, S2. The mitral and tricuspid produce the first heart sound, S1. The aortic and mitral do not close at the same time. The mitral and pulmonic do not close at the same time.

A nurse is caring for a patient who just underwent an intravenous pyelography that revealed a renal calculus obstructing the left ureter. What is the nurse's first priority in caring for this patient? a. Turn the patient on the right side to alleviate pressure on the left kidney. b. Encourage the patient to increase fluid intake to flush the obstruction. c. Monitor the patient for fever, rash, and difficulty breathing. d. Administer narcotic medications to the patient for pain.

ANS: C Assess for delayed hypersensitivity to the contrast media. Intravenous pyelography is performed by administering iodine-based dye to view functionality of the urinary system. Therefore, the first nursing priority is to assess the patient for an allergic reaction that could be life threatening. The nurse should then encourage the patient to drink fluids to flush dye resulting from the procedure. Narcotics can be administered but are not the first priority. Turning the patient on the side will not affect patient safety.

Before giving the patient an intermittent gastric tube feeding, what should the nurse do? a. Make sure that the tube is secured to the gown with a safety pin. b. Inject air into the stomach via the tube and auscultate. c. Have the tube feeding at room temperature. d. Check to make sure pH is at least 5.

ANS: C Be sure that the formula is at room temperature. Cold formula causes gastric cramping and discomfort because the mouth and the esophagus do not warm the liquid. Do not use safety pins. Safety pins can become unfastened and may cause harm to the patient. Auscultation is no longer considered a reliable method for verification of tube placement because a tube inadvertently placed in the lungs, pharynx, or esophagus transmits sound similar to that of air entering the stomach. Gastric fluid of patient who has fasted for at least 4 hours usually has a pH of 1 to 4, especially when the patient is not receiving gastric-acid inhibitor.

The nurse will anticipate which diagnostic examination for a patient with black tarry stools? a. Ultrasound b. Barium enema c. Endoscopy d. Anorectal manometry

ANS: C Black tarry stools are an indication of bleeding in the GI tract; endoscopy would allow visualization of the bleeding. No other option (ultrasound, barium enema, and anorectal manometry) would allow GI visualization.

The nurse is completing an assessment on a patient who has a Stage IV pressure ulcer. The wound is odorous, and a drain is currently in place. Which statement by the patient indicates issues with self-concept? a. "I am so weak and tired. I want to feel better." b. "I am thinking I will be ready to go home early next week." c. "I am ready for my bath and linen change right now since this is awful." d. "I am hoping there will be something good for dinner tonight."

ANS: C Body image changes can influence self-concept. The wound is odorous, and a drain is in place. The patient who is asking for a bath and change in linens and states that this is awful gives you a clue that he or she may be concerned about the smell in the room. Factors that affect the patient's perception of the wound include the presence of scars, drains, odor from drainage, and temporary or permanent prosthetic devices. The patient's stating that he or she wants to feel better, talking about going home, and caring about what is for dinner could be interpreted as positive statements that indicate progress along the health journey.

A patient has heart failure and cardiac output is decreased. Which formula can the nurse use to calculate cardiac output? a. Myocardial contractility × Myocardial blood flow b. Ventricular filling time/Diastolic filling time c. Stroke volume × Heart rate d. Preload/Afterload

ANS: C Cardiac output can be calculated by multiplying the stroke volume and the heart rate. The other options are not measures of cardiac output.

A nurse is caring for an immobile patient. Which metabolic alteration will the nurse monitor for in this patient? a. Increased appetite b. Increased diarrhea c. Increased metabolic rate d. Altered nutrient metabolism

Altered nutrient metabolism

When a nurse tries to understand a patient's and family caregiver's perspective of why a patient is falling at home, the nurse applies the intellectual standard of _________________________ to understand all viewpoints.

Broad

The nurse is cleansing a wound site. As the nurse administers the procedure, which intervention should be included? a. Allow the solution to flow from the most contaminated to the least contaminated. b. Scrub vigorously when applying noncytotoxic solution to the skin. c. Cleanse in a direction from the least contaminated area. d. Utilize clean gauze and clean gloves to cleanse a site.

ANS: C Cleanse in a direction from the least contaminated area, such as from the wound or incision, to the surrounding skin. While cleansing surgical or traumatic wounds by applying noncytotoxic solution with sterile gauze or by irrigations is correct, vigorous scrubbing is inappropriate and can cause damage to the skin. Use gentle friction when applying solutions to the skin, and allow irrigation to flow from the least to the most contaminated area.

. An older adult's perineal skin is dry and thin with mild excoriation. When providing hygiene care after episodes of diarrhea, what should the nurse do? a. Thoroughly scrub the skin with a washcloth and hypoallergenic soap. b. Tape an occlusive moisture barrier pad to the patient's skin. c. Apply a skin protective ointment after perineal care. d. Massage the skin with light kneading pressure.

ANS: C Cleansing with a no-rinse cleanser and application of a barrier ointment should be done after each episode of diarrhea. Tape and occlusive dressings can damage skin. Excessive pressure and massage are inappropriate and may cause skin breakdown.

The nurse suspects cystitis related to a lower urinary tract infection. Which clinical manifestation does the nurse expect the patient to report? a. Dysuria b. Flank pain c. Frequency d. Fever

ANS: C Cystitis is inflammation of the bladder; associated symptoms include hematuria, foul-smelling cloudy urine, and urgency/frequency. Dysuria is a common symptom of a lower urinary tract infection (bladder). Flank pain, fever, and chills are all signs of pyelonephritis (upper urinary tract).

The nurse is caring for a patient with a pressure ulcer on the left hip. The ulcer is black. Which next step will the nurse anticipate? a. Monitor the wound. b. Document the wound. c. Debride the wound. d. Manage drainage from wound.

ANS: C Debridement is the removal of nonviable necrotic (black) tissue. Removal of necrotic tissue is necessary to rid the ulcer of a source of infection, to enable visualization of the wound bed, and to provide a clean base for healing. A wound will not move through the phases of healing if the wound is infected. Documentation occurs after completion of skill. When treating a pressure ulcer, it is important to monitor and reassess the wound at least every 8 hours. Management of drainage will help keep the wound clean, but that is not the next step.

A nurse is reviewing urinary laboratory results. Which finding will cause the nurse to follow up? a. Protein level of 2 mg/100 mL b. Urine output of 80 mL/hr c. Specific gravity of 1.036 d. pH of 6.4

ANS: C Dehydration, reduced renal blood flow, and increase in antidiuretic hormone secretion elevate specific gravity. Normal specific gravity is 1.0053 to 1.030. An output of 30 mL/hr or less for 2 or more hours would be cause for concern; 80 mL/hr is normal. The normal pH of urine is between 4.6 and 8.0. Protein up to 8 mg/100 mL is acceptable; however, values in excess of this could indicate renal disease.

A patient had an ileostomy surgically placed 2 days ago. Which diet will the nurse recommend to the patient to ease the transition of the new ostomy? a. Eggs over easy, whole wheat toast, and orange juice with pulp b. Chicken fried rice with fresh pineapple and iced tea c. Turkey meatloaf with white rice and apple juice d. Fish sticks with sweet corn and soda

ANS: C During the first few days after ostomy placement, the patient should consume easy-to-digest soft foods such as poultry, rice, and noodles. Fried foods can irritate digestion. Foods high in fiber will be useful later in the recovery process but can cause food blockage if the GI tract is not accustomed to digesting with an ileostomy. Foods with indigestible fiber such as sweet corn, popcorn, raw mushrooms, fresh pineapple, and Chinese cabbage could cause this problem.

A guaiac test is ordered for a patient. Which type of blood is the nurse checking for in this patient's stool? a. Bright red blood b. Dark black blood c. Microscopic d. Mucoid

ANS: C Fecal occult blood tests are used to test for blood that may be present in stool but cannot be seen by the naked eye (microscopic). This is usually indicative of a gastrointestinal bleed. All other options are incorrect. Detecting bright red blood, dark black blood, and blood that contains mucus (mucoid) is not the purpose of a guaiac test.

In providing prenatal care to a pregnant patient, what does the nurse teach the expectant mother? a. Calcium intake is especially important in the first trimester. b. Protein intake needs to decrease to preserve kidney function. c. Folic acid is needed to help prevent birth defects and anemia. d. Extra vitamins and minerals should be taken as much as possible.

ANS: C Folic acid intake is particularly important for DNA synthesis and growth of red blood cells. Inadequate intake may lead to fetal neural tube defects, anencephaly, or maternal megaloblastic anemia. Protein intake throughout pregnancy needs to increase to 60 grams daily. Calcium intake is especially critical in the third trimester, when fetal bones mineralize. Prenatal care usually includes vitamin and mineral supplementation to ensure daily intakes; however, pregnant women should not take additional supplements beyond prescribed amounts.

The nurse is caring for a patient who is experiencing a full-thickness repair. Which type of tissue will the nurse expect to observe when the wound is healing? a. Eschar b. Slough c. Granulation d. Purulent drainage

ANS: C Granulation tissue is red, moist tissue composed of new blood vessels, the presence of which indicates progression toward healing. Soft yellow or white tissue is characteristic of slough—a substance that needs to be removed for the wound to heal. Black or brown necrotic tissue is called eschar, which also needs to be removed for a wound to heal. Purulent drainage is indicative of an infection and will need to be resolved for the wound to heal.

A nurse is assisting a patient in making dietary choices that promote healthy bowel elimination. Which menu option should the nurse recommend? a. Broccoli and cheese soup with potato bread b. Turkey and mashed potatoes with brown gravy c. Grape and walnut chicken salad sandwich on whole wheat bread d. Dinner salad topped with hard-boiled eggs, cheese, and fat-free dressing

ANS: C Grapes and whole wheat bread are high fiber and should be chosen. Cheese, eggs, potato bread, and mashed potatoes do not contain as much fiber as whole wheat bread. A healthy diet for the bowel should include foods high in bulk-forming fiber. Whole grains, fresh fruit, and fresh vegetables are excellent sources. Foods without much fiber and with high levels of fat can slow down peristalsis, causing constipation.

The patient has H. pylori. Which action should the nurse take? a. Encourage avoidance of wheat and oats. b. Encourage milkshakes as a nutritious snack. c. Encourage completion of antibiotic therapy. d. Encourage nonsteroidal antiinflammatory drugs.

ANS: C H. pylori, a bacterium that causes up to 85% of peptic ulcers, is confirmed by laboratory tests or a biopsy during endoscopy. Antibiotics treat and control the bacterial infection. Avoidance of wheat and oats are required for patients with celiac disease who must follow a gluten-free diet. Encourage patients to avoid foods that increase stomach acidity and pain such as caffeine, decaffeinated coffee, frequent milk intake, citric acid juices, and certain seasonings (hot chili peppers, chili powder, black pepper). Discourage smoking, alcohol, aspirin, and nonsteroidal antiinflammatory drugs (NSAIDs).

The patient has just been started on an enteral feeding and has developed diarrhea after being on the feeding for 2 hours. What does the nurse suspect is the most likely cause of the diarrhea? a. Antibiotic therapy b. Clostridium difficile c. Formula intolerance d. Bacterial contamination

ANS: C Hyperosmolar formulas can cause diarrhea or formula intolerance. If that is the case, the solution is to lower the rate, dilute the formula, or change to an isotonic formula. Antibiotics destroy normal intestinal flora and disturb the internal ecology, allowing for Clostridium difficile toxin buildup. However, this takes time (more than 2 hours), and no indication suggests that this patient is on antibiotics. Bacterial contamination of the feeding usually occurs when feedings are left hanging for longer than 8 hours.

Which observation by the nurse best indicates that a continuous bladder irrigation for a patient following genitourinary surgery is effective? a. Output that is smaller than the amount instilled b. Blood clots or sediment in the drainage bag c. Bright red urine turns pink in the tubing d. Bladder distention with tenderness

ANS: C If urine is bright red or has clots, increase irrigation rate until drainage appears pink, indicating successful irrigation. Expect more output than fluid instilled because of urine production. If output is smaller than the amount instilled, suspect that the tube may be clogged. The presence of blood clots indicates the patient is still bleeding, while sediment could mean an infection or bleeding. The bladder should not be distended or tender; the irrigant may not be flowing freely if these occur, or the tube may be kinked or blocked.

In general, when a patient's energy requirements are completely met by kilocalorie (kcal) intake in food, which assessment finding will the nurse observe? a. Weight increases. b. Weight decreases. c. Weight does not change. d. Weight fluctuates daily.

ANS: C In general, when energy requirements are completely met by kilocalorie (kcal) intake in food, weight does not change. When kilocalories ingested exceed a person's energy demands, the individual gains weight. If kilocalories ingested fail to meet a person's energy requirement, the individual loses weight. Fluid, not kilocalories, causes daily weight fluctuations.

A nurse is caring for a male patient with urinary retention. Which action should the nurse take first? a. Limit fluid intake. b. Insert a urinary catheter. c. Assist to a standing position. d. Ask for a diuretic medication.

ANS: C In some patients just helping them to a normal position to void prompts voiding. A urinary catheter would relieve urinary retention, but it is not the first measure; other nursing interventions should be tried before catheterization. Reducing fluids would reduce the amount of urine produced but would not alleviate the urine retention and is usually not recommended unless the retention is severe. Diuretic medication would increase urine production and may worsen the discomfort caused by urine retention.

A nurse is caring for a 5-year-old patient whose temperature is 101.2° F. The nurse expects this patient to hyperventilate. Which factor does the nurse remember when planning care for this type of hyperventilation? a. Anxiety over illness b. Decreased drive to breathe c. Increased metabolic demands d. Infection destroying lung tissues

ANS: C Increased body temperature (fever) increases the metabolic rate, thereby increasing carbon dioxide production. The increased carbon dioxide level stimulates an increase in the patient's rate and depth of respiration, causing hyperventilation. Anxiety can cause hyperventilation, but this is not the direct cause from a fever. Sleep causes a decreased respiratory drive; hyperventilation speeds up breathing. The cause of the fever in this question is unknown.

A nurse is checking orders. Which order should the nurse question? a. A normal saline enema to be repeated every 4 hours until stool is produced b. A hypertonic solution enema for a patient with fluid volume excess c. A Kayexalate enema for a patient with severe hypokalemia d. An oil retention enema for a patient with constipation

ANS: C Kayexalate binds to and helps excrete potassium, so it would be contraindicated in patients who are hypokalemic (have low potassium). Normal saline enemas can be repeated without risk of fluid or electrolyte imbalance. Hypertonic solutions are intended for patients who cannot handle large fluid volume and are contraindicated for dehydrated patients. Oil retention enemas lubricate the feces in the rectum and colon and are used for constipation.

While performing an assessment, the nurse hears crackles in the patient's lung fields. The nurse also learns that the patient is sleeping on three pillows to help with the difficulty breathing during the night. Which condition will the nurse most likely observe written in the patient's medical record? a. Atrial fibrillation b. Myocardial ischemia c. Left-sided heart failure d. Right-sided heart failure

ANS: C Left-sided heart failure results in pulmonary congestion, the signs and symptoms of which include shortness of breath, cough, crackles, and paroxysmal nocturnal dyspnea (difficulty breathing when lying flat). Right-sided heart failure is systemic and results in peripheral edema, weight gain, and distended neck veins. Atrial fibrillation is often described as an irregularly irregular rhythm; rhythm is irregular because of the multiple pacemaker sites. Myocardial ischemia results when the supply of blood to the myocardium from the coronary arteries is insufficient to meet myocardial oxygen demands, producing angina or myocardial infarction

A nurse is caring for an 8-year-old patient who is embarrassed about urinating in bed at night. Which intervention should the nurse suggest to reduce the frequency of this occurrence? a. "Set your alarm clock to wake you every 2 hours, so you can get up to void." b. "Line your bedding with plastic sheets to protect your mattress." c. "Drink your nightly glass of milk earlier in the evening." d. "Empty your bladder completely before going to bed."

ANS: C Nightly incontinence and nocturia are often resolved by limiting fluid intake 2 hours before bedtime. Setting the alarm clock to wake does not correct the physiological problem, nor does lining the bedding with plastic sheets. Emptying the bladder may help with early nighttime urination but will not affect urine produced throughout the night from late-night fluid intake.

A patient with a pneumothorax has a chest tube inserted and is placed on low constant suction. Which finding requires immediate action by the nurse? a. The patient reports pain at the chest tube insertion site that increases with movement. b. Fifty milliliters of blood gushes into the drainage device after the patient coughs. c. No bubbling is present in the suction control chamber of the drainage device. d. Yellow purulent discharge is seen leaking out from around the dressing site.

ANS: C No bubbling in the suction control chamber indicates an obstruction of the drainage system. An obstruction causes increased pressure, which can cause a tension pneumothorax, which can be life threatening. The nurse needs to determine whether the leak is inside the thorax or in the tubing and act from there. Occasional blood gushes from the lung owing to lung expansion, as during a cough; this is reserve drainage. Drainage over 100 mL/hr after 3 hours of chest tube placement is cause for concern. Yellow purulent drainage indicates an infection that should be reported to the health care provider but is not as immediately life threatening as the lack of bubbling in the suction control chamber.

A patient has developed a pressure ulcer. Which laboratory data will be important for the nurse to check? a. Vitamin E b. Potassium c. Albumin d. Sodium

ANS: C Normal wound healing requires proper nutrition. Serum proteins are biochemical indicators of malnutrition, and serum albumin is probably the most frequently measured of these parameters. The best measurement of nutritional status is prealbumin because it reflects not only what the patient has ingested but also what the body has absorbed, digested, and metabolized. Zinc and copper are the minerals important for wound healing, not potassium and sodium. Vitamins A and C are important for wound healing, not vitamin E.

A nurse is caring for a postoperative patient. Which finding will alert the nurse to a potential wound dehiscence? a. Protrusion of visceral organs through a wound opening b. Chronic drainage of fluid through the incision site c. Report by patient that something has given way d. Drainage that is odorous and purulent

ANS: C Patients often report feeling as though something has given way with dehiscence. Dehiscence occurs when an incision fails to heal properly and the layers of skin and tissue separate. It involves abdominal surgical wounds and occurs after a sudden strain such as coughing, vomiting, or sitting up in bed. Evisceration is seen when vital organs protrude through a wound opening. When there is an increase in serosanguineous drainage from a wound in the first few days after surgery, be alert for the potential for dehiscence. Infection is characterized by drainage that is odorous and purulent

A nurse is caring for a patient who is taking warfarin (Coumadin) and discovers that the patient is taking garlic to help with hypertension. Which condition will the nurse assess for in this patient? a. Increased cholesterol level b. Distended jugular vein c. Bleeding d. Angina

ANS: C Patients taking warfarin (Coumadin) for anticoagulation prolong the prothrombin time (PT)/international normalized ratio (INR) results if they are taking gingko biloba, garlic, or ginseng with the anticoagulant. The drug interaction can precipitate a life-threatening bleed. Increased cholesterol levels are associated with saturated fat dietary intake. A distended jugular vein and peripheral edema are associated with damage to the right side of the heart. Angina is temporary ischemia of the heart muscle.

Which nutritional instruction is a priority for the nurse to advise a patient about with an ileostomy? a. Keep fiber low. b. Eat large meals. c. Increase fluid intake. d. Chew food thoroughly.

ANS: C Patients with ileostomies will digest their food completely but will lose both fluid and salt through their stoma and will need to be sure to replace this to avoid dehydration. A good reminder for patients is to encourage drinking an 8-ounce glass of fluid when they empty their pouch. This helps patients to remember that they have greater fluid needs than they did before having an ileostomy. A low-fiber diet is not necessary. Eating large meals is not advised. While chewing food thoroughly is correct, it is not the priority; liquid is the priority.

A nurse is evaluating a nursing assistive personnel's (NAP) care for a patient with an indwelling catheter. Which action by the NAP will cause the nurse to intervene? a. Emptying the drainage bag when half full b. Kinking the catheter tubing to obtain a urine specimen c. Placing the drainage bag on the side rail of the patient's bed d. Securing the catheter tubing to the patient's thigh

ANS: C Placing the drainage bag on the side rail of the bed could allow the bag to be raised above the level of the bladder and urine to flow back into the bladder. The urine in the drainage bag is a medium for bacteria; allowing it to reenter the bladder can cause infection. A key intervention to prevent catheter-associated urinary tract infections is prevention of urine back flow from the tubing and bag into the bladder. All the rest are correct procedures and do not require follow-up. The drainage bag should be emptied when half full; an overfull drainage bag can create tension and pulling on the catheter, resulting in trauma to the urethra and/or urinary meatus and increasing risk for urinary tract infections. Urine specimens are obtained by temporarily kinking the tubing; a prolonged kink could lead to bladder distention. Failure to secure the catheter to the patient's thigh places the patient at risk for tissue injury from catheter dislodgment.

In determining malnourishment in a patient, which assessment finding is consistent with this disorder? a. Moist lips b. Pink conjunctivae c. Spoon-shaped nails d. Not easily plucked hair

ANS: C Spoon-shaped nails, koilonychia, is an indication of poor nutrition. All the others are normal findings. Lips should be moist, conjunctivae should be pink, and hair should not be easily plucked.

Which assessment question should the nurse ask if stress incontinence is suspected? a. "Do you think your bladder feels distended?" b. "Do you empty your bladder completely when you void?" c. "Do you experience urine leakage when you cough or sneeze?" d. "Do your symptoms increase with consumption of alcohol or caffeine?"

ANS: C Stress incontinence can be related to intraabdominal pressure causing urine leakage, as would happen during coughing or sneezing. Asking the patient about the fullness of the bladder would rule out retention and overflow. An inability to void completely can refer to urge incontinence. Physiological causes and medications can effect elimination, but this is not related to stress incontinence.

The patient is having at least 75% of nutritional needs met by enteral feeding, so the health care provider has ordered the parenteral nutrition (PN) to be discontinued. However, the nurse notices that the PN infusion has fallen behind. What should the nurse do? a. Increase the rate to get the volume caught up before discontinuing. b. Stop the infusion as ordered. c. Taper infusion gradually. d. Hang 5% dextrose.

ANS: C Sudden discontinuation of PN can cause hypoglycemia. PN must be tapered off. Usually, 10% dextrose is infused when PN solution is suddenly discontinued. Too rapid administration of hypertonic dextrose (PN) can result in an osmotic diuresis and dehydration. If an infusion falls behind schedule, the nurse should not increase the rate in an attempt to catch up.

A nurse is teaching the patient with mitral valve problems about the valves in the heart. Starting on the right side of the heart, describe the sequence of the blood flow through these valves. 1. Mitral 2. Aortic 3. Tricuspid 4. Pulmonic a. 1, 3, 2, 4 b. 4, 3, 2, 1 c. 3, 4, 1, 2 d. 2, 4, 1, 3

ANS: C The blood flows through the valves in the following direction: tricuspid, pulmonic, mitral, and aortic.

The nurse is caring for a patient who is immobile. The nurse wants to decrease the formation of pressure ulcers. Which action will the nurse take first? a. Offer favorite fluids. b. Turn the patient every 2 hours. c. Determine the patient's risk factors. d. Encourage increased quantities of carbohydrates and fats.

ANS: C The first step in prevention is to assess the patient's risk factors for pressure ulcer development. When a patient is immobile, the major risk to the skin is the formation of pressure ulcers. Nursing interventions focus on prevention. Offering favorite fluids, turning, and increasing carbohydrates and fats are not the first steps. Determining risk factors is first so interventions can be implemented to reduce or eliminate those risk factors.

. Which nursing intervention is most effective in preventing hospital-acquired pneumonia in an older-adult patient? a. Discontinue the humidification delivery device to keep excess fluid from lungs. b. Monitor oxygen saturation, and frequently auscultate lung bases. c. Assist the patient to cough, turn, and deep breathe every 2 hours. d. Decrease fluid intake to 300 mL a shift.

ANS: C The goal of the nursing action should be the prevention of pneumonia; the action that best addresses this is to cough, turn, and deep breathe to keep secretions from pooling at the base of the lungs. Humidification thins respiratory secretions, making them easier to expel and should be used. Monitoring oxygen status is important but is not a method of prevention. Hydration assists in preventing hospital-acquired pneumonia. The best way to maintain thin secretions is to provide a fluid intake of 1500 to 2500 mL/day unless contraindicated by cardiac or renal status. Restricting fluids is contraindicated in this situation since there is no data indicating cardiac or renal disease.

The nurse is caring for a group of patients. Which patient will the nurse see first? a. A patient with a Stage IV pressure ulcer b. A patient with a Braden Scale score of 18 c. A patient with appendicitis using a heating pad d. A patient with an incision that is approximated

ANS: C The nurse should see the patient with an appendicitis first. Warm applications are contraindicated when the patient has an acute, localized inflammation such as appendicitis because the heat could cause the appendix to rupture. Although a Stage IV pressure ulcer is deep, it is not as critical as the appendicitis patient. The total Braden score ranges from 6 to 23; a lower total score indicates a higher risk for pressure ulcer development. A score of 18 can be assessed later. A healing incision is approximated (closed); this is a normal finding and does not need to be seen first.

The nurse is caring for patients with ostomies. In which ostomy location will the nurse expect very liquid stool to be present? a. Sigmoid b. Transverse c. Ascending d. Descending

ANS: C The path of digestion goes from the ascending, across the transverse, to the descending and finally passing into the sigmoid; therefore, the least formed stool (very liquid) would be in the ascending.

The nurse is planning care for a group of patients. Which task will the nurse assign to the nursing assistive personnel (NAP)? a. Performing the first postoperative pouch change b. Maintaining a nasogastric tube c. Administering an enema d. Digitally removing stool

ANS: C The skill of administering an enema can be delegated to an NAP. The skill of inserting and maintaining a nasogastric (NG) tube cannot be delegated to an NAP. The nurse should do the first postoperative pouch change. Digitally removing stool cannot be delegated to nursing assistive personnel.

A nurse is preparing to administer an enteral feeding. In which order will the nurse implement the steps, starting with the first one? 1. Elevate head of bed to at least 30 degrees. 2. Check for gastric residual volume. 3. Flush tubing with 30 mL of water. 4. Verify tube placement. 5. Initiate feeding. a. 4, 2, 1, 5, 3 b. 2, 4, 1, 3, 5 c. 1, 4, 2, 3, 5 d. 2, 1, 4, 5, 3

ANS: C The steps for an enteral feeding are as follows: Place patient in high-Fowler's position or elevate head of bed to at least 30 (preferably 45) degrees; verify tube placement; check for gastric residual volume; flush tubing with 30 mL of water; and initiate feeding.

. A nurse is preparing a patient for nasotracheal suctioning. In which order will the nurse perform the steps, beginning with the first step? 1. Insert catheter. 2. Apply suction and remove. 3. Have patient deep breathe. 4. Encourage patient to cough. 5. Attach catheter to suction system. 6. Rinse catheter and connecting tubing. a. 1, 2, 3, 4, 5, 6 b. 4, 5, 1, 2, 3, 6 c. 5, 3, 1, 2, 4, 6 d. 3, 1, 2, 5, 4, 6

ANS: C The steps for nasotracheal suctioning are as follows: Verify that catheter is attached to suction; have patient deep breathe; insert catheter; apply intermittent suction for no more than 10 seconds and remove; encourage patient to cough; and rinse catheter and connecting tubing with normal saline

Which action will the nurse take to reduce the risk of excoriation to the mucosal lining of the patient's nose from a nasogastric tube? a. Instill Xylocaine into the nares once a shift. b. Tape tube securely with light pressure on nare. c. Lubricate the nares with water-soluble lubricant. d. Apply a small ice bag to the nose for 5 minutes every 4 hours.

ANS: C The tube constantly irritates the nasal mucosa, increasing the risk of excoriation. Frequent lubrication with a water-soluble lubricant decreases the likelihood of excoriation and is less toxic than oil-based if aspirated. Xylocaine is used to treat sore throat, not nasal mucosal excoriation. While the tape should be secure, pressure will increase excoriation. Ice is not applied to the nose.

Upon palpation, the nurse notices that the bladder is firm and distended; the patient expresses an urge to urinate. Which question is most appropriate? a. "Does your urinary problem interfere with any activities?" b. "Do you lose urine when you cough or sneeze?" c. "When was the last time you voided?" d. "Are you experiencing a fever or chills?"

ANS: C To obtain an accurate assessment, the nurse should first determine the source of the discomfort. Urinary retention causes the bladder to be firm and distended; time of last void is most appropriate. Further assessment to determine the pathology of the condition can be performed later. Questions concerning fever and chills, interference with any activities, and losing urine during coughing or sneezing focus on specific pathological conditions.

A nurse is teaching a health class about the heart. Which information from the class members indicates teaching by the nurse is successful for the flow of blood through the heart, starting in the right atrium? a. Right ventricle, left ventricle, left atrium b. Left atrium, right ventricle, left ventricle c. Right ventricle, left atrium, left ventricle d. Left atrium, left ventricle, right ventricle

ANS: C Unoxygenated blood flows through the venae cavae into the right atrium, where it is pumped down to the right ventricle; the blood is then pumped out the pulmonary artery and is returned oxygenated via the pulmonary vein to the left atrium, where it flows to the left ventricle and is pumped out to the rest of the body via the aorta.

The wound care nurse visits a patient in the long-term care unit. The nurse is monitoring a patient with a Stage III pressure ulcer. The wound seems to be healing, and healthy tissue is observed. How should the nurse document this ulcer in the patient's medical record? a. Stage I pressure ulcer b. Healing Stage II pressure ulcer c. Healing Stage III pressure ulcer d. Stage III pressure ulcer

ANS: C When a pressure ulcer has been staged and is beginning to heal, the ulcer keeps the same stage and is labeled with the words "healing stage" or healing Stage III pressure ulcer. Once an ulcer has been staged, the stage endures even as the ulcer heals. This ulcer was labeled a Stage III, and it cannot return to a previous stage such as Stage I or II. This ulcer is healing, so it is no longer labeled a Stage III.

A nurse is caring for a patient with a postsurgical wound. When planning care, which goal will be the priority? a. Reduce dependent nitrogen balance. b. Maintain negative nitrogen balance. c. Promote positive nitrogen balance. d. Facilitate neutral nitrogen balance.

ANS: C When intake of nitrogen is greater than output, the body is in positive nitrogen balance. Positive nitrogen balance is required for growth, normal pregnancy, maintenance of lean muscle mass and vital organs, and wound healing. Negative nitrogen balance occurs when the body loses more nitrogen than the body gains. Neutral nitrogen balance occurs when gain equals loss and is not optimal for tissue healing. There is no such term as dependent nitrogen balance.

A patient's heart rate increased from 94 to 164 beats/min. What will the nurse expect? a. Increase in diastolic filling time b. Decrease in hemoglobin level c. Decrease in cardiac output d. Increase in stroke volume

ANS: C With a sustained heart rate greater than 160 beats/min, diastolic filling time decreases, decreasing stroke volume and cardiac output. The hemoglobin level would not be affected.

A nurse is caring for a patient with sleep apnea. Which types of ventilator support should the nurse be prepared to administer for this patient? (Select all that apply.) a. Assist-control (AC) b. Pressure support ventilation (PSV) c. Bilevel positive airway pressure (BiPAP) d. Continuous positive airway pressure (CPAP) e. Synchronized intermittent mandatory ventilation (SIMV)

ANS: C, D Ventilatory support is achieved using a variety of modes, including continuous positive airway pressure (CPAP) and bilevel positive airway pressure (BiPAP). The purpose of CPAP and BiPAP is to maintain a positive airway pressure and improve alveolar ventilation. This prevents or treats atelectasis by inflating the alveoli, reducing pulmonary edema by forcing fluid out of the lungs back into circulation, and improving oxygenation in those with sleep apnea. AC, PSV, and SIMV are invasive mechanical ventilation and are not routinely used on patients with sleep apnea. AC delivers a set tidal volume (VT) with each breath, regardless of whether the breath was triggered by the patient or the ventilator. Synchronized intermittent mandatory ventilation like AC delivers a minimum number of fully assisted breaths per minute that are synchronized with the patient's respiratory effort. Any breaths taken between volume-cycled breaths are not assisted; the volume of these breaths is determined by the patient's strength, effort, and lung mechanics. PSV mode is often combined with SIMV mode: inspiratory pressure is added to spontaneous breaths to overcome the resistance of the endotracheal tube or to help increase the volume of the patient's spontaneous breaths.

The nurse is caring for a patient who has experienced a laparoscopic appendectomy. For which type of healing will the nurse focus the care plan? a. Partial-thickness repair b. Secondary intention c. Tertiary intention d. Primary intention

ANS: D A clean surgical incision is an example of a wound with little loss of tissue that heals with primary intention. The skin edges are approximated or closed, and the risk for infection is low. Partial-thickness repairs are done on partial-thickness wounds that are shallow, involving loss of the epidermis and maybe partial loss of the dermis. These wounds heal by regeneration because the epidermis regenerates. Tertiary intention is seen when a wound is left open for several days, and then the wound edges are approximated. Wound closure is delayed until the risk of infection is resolved. A wound involving loss of tissue such as a burn or a pressure ulcer or laceration heals by secondary intention. The wound is left open until it becomes filled with scar tissue. It takes longer for a wound to heal by secondary intention; thus the chance of infection is greater.

A patient has a fecal impaction. Which portion of the colon will the nurse assess? a. Descending b. Transverse c. Ascending d. Rectum

ANS: D A fecal impaction is a collection of hardened feces wedged in the rectum that cannot be expelled. It results from unrelieved constipation. Feces at this point in the colon contain the least amount of moisture. Feces found in the ascending, transverse, and descending colon still consist mostly of liquid and do not form a hardened mass

The nurse is caring for a patient who has experienced a total abdominal hysterectomy. Which nursing observation of the incision will indicate the patient is experiencing a complication of wound healing? a. The site is hurting. b. The site is approximated. c. The site has started to itch. d. The site has a mass, bluish in color.

ANS: D A hematoma is a localized collection of blood underneath the tissues. It appears as swelling, change in color, sensation, or warmth or a mass that often takes on a bluish discoloration. A hematoma near a major artery or vein is dangerous because it can put pressure on the vein or artery and obstruct blood flow. Itching is not a complication. Incisions should be approximated with edges together; this is a sign of normal healing. After surgery, when nerves in the skin and tissues have been traumatized by the surgical procedure, it is expected that the patient will experience pain.

A nurse is assessing a patient's wound. Which nursing observation will indicate the wound healed by secondary intention? a. Minimal loss of tissue function b. Permanent dark redness at site c. Minimal scar tissue d. Scarring that may be severe

ANS: D A wound healing by secondary intention takes longer than one healing by primary intention. The wound is left open until it becomes filled with scar tissue. If the scarring is severe, permanent loss of function often occurs. Wounds that heal by primary intention heal quickly with minimal scarring. Scar tissue contains few pigmented cells and has a lighter color than normal skin.

The patient has a calculated body mass index (BMI) of 34. How will the nurse classify this finding? a. Normal weight b. Underweight c. Overweight d. Obese

ANS: D BMI greater than 30 is defined as obesity. BMI between 25 and 30 is classified as overweight. BMI from 18.5 to 24.9 is normal. BMI under 18.5 is underweight.

A nurse is reviewing results from a urine specimen. What will the nurse expect to see in a patient with a urinary tract infection? a. Casts b. Protein c. Crystals d. Bacteria

ANS: D Bacteria in the urine along with other symptoms support a diagnosis of urinary tract infection. Crystals would be seen with renal stone formation. Casts indicate renal disease. Protein indicates kidney function and damage to the glomerular membrane such as in glomerulonephritis.

The nurse is caring for a patient with a Stage II pressure ulcer and has assigned a nursing diagnosis of Risk for infection. The patient is unconscious and bedridden. The nurse is completing the plan of care and is writing goals for the patient. Which is the best goal for this patient? a. The patient will state what to look for with regard to an infection. b. The patient's family will demonstrate specific care of the wound site. c. The patient's family members will wash their hands when visiting the patient. d. The patient will remain free of odorous or purulent drainage from the wound.

ANS: D Because the patient has an open wound and the skin is no longer intact to protect the tissue, the patient is at increased risk for infection. The nurse will be assessing the patient for signs and symptoms of infection, including an increase in temperature, an increase in white count, and odorous and purulent drainage from the wound. The patient is unconscious and is unable to communicate the signs and symptoms of infection. It is important for the patient's family to be able to demonstrate how to care for the wound and wash their hands, but these statements are not goals or outcomes for this nursing diagnosis

In teaching mothers-to-be about infant nutrition, which instruction should the nurse provide? a. Supplement breast milk with corn syrup. b. Give cow's milk during the first year of life. c. Add honey to infant formulas for increased energy. d. Provide breast milk or formula for the first 4 to 6 months.

ANS: D Breast milk or formula provides sufficient nutrition for the first 4 to 6 months of life. Infants should not have regular cow's milk during the first year of life. It is too concentrated for an infant's kidneys to manage, increases the risk of milk product allergies, and is a poor source of iron and vitamins C and E. Furthermore, children under 1 year of age should never ingest honey and corn syrup products because they are potential sources of the botulism toxin, which increases the risk of infant death.

A patient expresses concerns over having black stool. The fecal occult test is negative. Which response by the nurse is most appropriate? a. "This is probably a false negative; we should rerun the test." b. "You should schedule a colonoscopy as soon as possible." c. "Are you under a lot of stress?" d. "Do you take iron supplements?"

ANS: D Certain medications and supplements, such as iron, can alter the color of stool (black or tarry). Since the fecal occult test is negative, bleeding is not occurring. The fecal occult test takes three separate samples over a period of time and is a fairly reliable test. A colonoscopy is health prevention screening that should be done every 5 to 10 years; it is not the nurse's initial priority. Stress alters GI motility and stool consistency, not color.

The nurse is using a closed suction device. Which patient will be most appropriate for this suctioning method? a. A 5-year-old with excessive drooling from epiglottitis b. A 5-year-old with an asthma attack following severe allergies c. A 24-year-old with a right pneumothorax following a motor vehicle accident d. A 24-year-old with acute respiratory distress syndrome requiring mechanical ventilation

ANS: D Closed suctioning is most often used on patients who require invasive mechanical ventilation to support their respiratory efforts because it permits continuous delivery of oxygen while suction is performed and reduces the risk of oxygen desaturation. In this case, the acute respiratory distress syndrome requires mechanical ventilation. In the presence of epiglottitis, croup, laryngospasm, or irritable airway, the entrance of a suction catheter via the nasal route causes intractable coughing, hypoxemia, and severe bronchospasm, necessitating emergency intubation or tracheostomy. The 5-year-old child with asthma would benefit from an inhaler. A chest tube is needed for the pneumothorax.

The nurse is assessing a patient for nutritional status. Which action will the nurse take? a. Forego the assessment in the presence of chronic disease. b. Use the Mini Nutritional Assessment for pediatric patients. c. Choose a single objective tool that fits the patient's condition. d. Combine multiple objective measures with subjective measures.

ANS: D Combine multiple objective measures with subjective measures related to nutrition to adequately screen for nutritional problems. Using a single objective measure is ineffective in predicting risk of nutritional problems. Chronic disease and increased metabolic requirements are risk factors for the development of nutritional problems; these patients may be in critical need of this assessment. The Mini Nutritional Assessment is used for screening older adults in home care programs, nursing homes, and hospitals.

The nurse is caring for a patient with respiratory problems. Which assessment finding indicates a late sign of hypoxia? a. Elevated blood pressure b. Increased pulse rate c. Restlessness d. Cyanosis

ANS: D Cyanosis, blue discoloration of the skin and mucous membranes caused by the presence of desaturated hemoglobin in capillaries, is a late sign of hypoxia. Elevated blood pressure, increased pulse rate, and restlessness are early signs of hypoxia.

The patient has been diagnosed with cardiovascular disease and placed on a low-fat diet. The patient asks the nurse, "How much fat should I have? I guess the less fat, the better." Which information will the nurse include in the teaching session? a. Cholesterol intake needs to be less than 300 mg/day. b. Fats have no significance in health and the incidence of disease. c. All fats come from external sources so this can be easily controlled. d. Deficiencies occur when fat intake falls below 10% of daily nutrition.

ANS: D Deficiency occurs when fat intake falls below 10% of daily nutrition. While keeping cholesterol below 300 mg is correct according to the American Heart Association, it does not answer the patient's question about fat. Various types of fatty acids have significance for health and for the incidence of disease and are referred to in dietary guidelines. Linoleic acid and arachidonic acid are important for metabolic processes but are manufactured by the body when linoleic acid is available from the diet.

The nurse is teaching about the process of exchanging gases through the alveolar capillary membrane. Which term will the nurse use to describe this process? a. Ventilation b. Surfactant c. Perfusion d. Diffusion

ANS: D Diffusion is the process of gases exchanging across the alveoli and capillaries of body tissues.Ventilation is the process of moving gases into and out of the lungs. Surfactant is a chemical produced in the lungs to maintain the surface tension of the alveoli and keep them from collapsing. Perfusion is the ability of the cardiovascular system to carry oxygenated blood to tissues and return deoxygenated blood to the heart.

The nurse is caring for a patient receiving total parenteral nutrition (TPN). Which action will the nurse take? a. Run lipids for no longer than 24 hours. b. Take down a running bag of TPN after 36 hours. c. Clean injection port with alcohol 5 seconds before and after use. d. Wear a sterile mask when changing the central venous catheter dressing.

ANS: D During central venous catheter dressing changes, always use a sterile mask and gloves, and assess insertion sites for signs and symptoms of infection. To avoid infection, change the TPN infusion tubing every 24 hours, and do not hang a single container of PN for longer than 24 hours or lipids longer than 12 hours.

A nurse is asked how many kcal per gram fats provided. How should the nurse answer? a. 3 b. 4 c. 6 d. 9

ANS: D Fats (lipids) are the most calorie-dense nutrient, providing 9 kcal/g. Carbohydrates and protein provide 4 kcal/g.

Which clinical manifestation will the nurse expect to observe in a patient with excessive white blood cells present in the urine? a. Reduced urine specific gravity b. Increased blood pressure c. Abnormal blood sugar d. Fever with chills

ANS: D Fever and chills may be observed. The presence of white blood cells in urine indicates a urinary tract infection or inflammation. Overhydration, early renal disease, and inadequate antidiuretic hormone secretion reduce specific gravity. Increased blood pressure is associated with renal disease or damage and some medications. Abnormal blood sugars would be seen in someone with ketones in the urine or a patient with diabetes.

. The nurse is assessing a patient with emphysema. Which assessment finding requires further follow-up with the health care provider? a. Increased anterior-posterior diameter of the chest b. Accessory muscle used for breathing c. Clubbing of the fingers d. Hemoptysis

ANS: D Hemoptysis is an abnormal occurrence of emphysema, and further diagnostic studies are needed to determine the cause of blood in the sputum. Clubbing of the fingers, barrel chest (increased anterior-posterior chest diameter), and accessory muscle use are all normal findings in a patient with emphysema.

The nurse is caring for a postoperative medial meniscus repair of the right knee. Which action should the nurse take to assist with pain management? a. Monitor vital signs every 15 minutes. b. Check pulses in the right foot. c. Keep the leg dependent. d. Apply ice.

ANS: D Ice assists in preventing edema formation, controlling bleeding, and anesthetizing the body part. Elevation (not dependent) assists in preventing edema, which in turn can cause pain. Monitoring vital signs every 15 minutes is routine postoperative care and includes a pain assessment but in itself is not an intervention that decreases pain. Checking the pulses is important to monitor the circulation of the extremity but in itself is not a pain management intervention

A nurse is inserting a catheter into a female patient. When the nurse inserts the catheter, no urine is obtained. The nurse suspects the catheter is not in the urethra. What should the nurse do? a. Throw the catheter way and begin again. b. Fill the balloon with the recommended sterile water. c. Remove the catheter, wipe with alcohol, and reinsert after lubrication. d. Leave the catheter in the vagina as a landmark for insertion of a new, sterile catheter.

ANS: D If no urine appears, the catheter may be in the vagina. If misplaced, leave the catheter in the vagina as a landmark to indicate where not to insert, and insert another sterile catheter. The catheter should be left in place until the new, sterile catheter is inserted. The balloon should not be filled since the catheter is in the vagina. The catheter must be sterile; using alcohol will not make the catheter sterile.

A nurse is preparing to suction a patient. The pulse is 65 and pulse oximetry is 94%. Which finding will cause the nurse to stop suctioning? a. Pulse 75 b. Pulse 80 c. Oxygen saturation 91% d. Oxygen saturation 88%

ANS: D Stop when oxygen saturation is 88%. Monitor patient's vital signs and oxygen saturation during procedure; note whether there is a change of 20 beats/min (either increase or decrease) or if pulse oximetry falls below 90% or 5% from baseline. If this occurs, stop suctioning. A pulse rate of 75 is only 10 beats different from the start of the procedure. A pulse rate of 80 is 15 beats different from the start of suctioning. Oxygen saturation of 91% is not 5% from baseline or below 90%.

A patient with a hip fracture is having difficulty defecating into a bedpan while lying in bed. Which action by the nurse will assist the patient in having a successful bowel movement? a. Preparing to administer a barium enema b. Withholding narcotic pain medication c. Administering laxatives to the patient d. Raising the head of the bed

ANS: D Lying in bed is an unnatural position; raising the head of the bed assists the patient into a more normal position that allows proper contraction of muscles for elimination. Laxatives would not give the patient control over bowel movements. A barium enema is a diagnostic test, not an intervention to promote defecation. Pain relief measures should be given; however, preventative action should be taken to prevent constipation.

The nurse is caring for a surgical patient. Which intervention is most important for the nurse to complete to decrease the risk of pressure ulcers and encourage the patient's willingness and ability to increase mobility? a. Explain the risks of immobility to the patient. b. Turn the patient every 3 hours while in bed. c. Encourage the patient to sit up in the chair. d. Provide analgesic medication as ordered.

ANS: D Maintaining adequate pain control (providing analgesic medications) and patient comfort increases the patient's willingness and ability to increase mobility, which in turn reduces pressure ulcer risks. Although sitting in the chair is beneficial, it does not increase mobility or provide pain control. Explaining the risk of immobility is important for the patient because it may impact the patient's willingness but not his or her ability. Turning the patient is important for decreasing pressure ulcers but needs to be done every 2 hours and, again, does not influence the patient's ability to increase mobility

A patient is on a full liquid diet. Which food item choice by the patient will cause the nurse to intervene? a. Custard b. Frozen yogurt c. Pureed vegetables d. Mashed potatoes and gravy

ANS: D Mashed potatoes and gravy are on a dysphagia, mechanical soft, soft and regular diet but are not components of a full liquid diet. The nurse will need to provide teaching on what is allowed on the diet. Custard, frozen yogurt, and pureed vegetables are all on a full liquid diet.

The nurse is preparing to insert a nasogastric tube in a patient who is semiconscious. To determine the length of the tube needed to be inserted, how should the nurse measure the tube? a. From the tip of the nose to the earlobe b. From the tip of the earlobe to the xiphoid process c. From the tip of the earlobe to the nose to the xiphoid process d. From the tip of the nose to the earlobe to the xiphoid process

ANS: D Measure distance from the tip of the nose to the earlobe to the xiphoid process of the sternum. This approximates the distance from the nose to the stomach in 98% of patients. For duodenal or jejunal placement, an additional 20 to 30 cm is required.

In providing diet education for a patient on a low-fat diet, which information is important for the nurse to share? a. Polyunsaturated fats should be less than 7% of the total calories. b. Trans fat should be less than 7% of the total calories. c. Unsaturated fats are found mostly in animal sources. d. Saturated fats are found mostly in animal sources.

ANS: D Most animal fats have high proportions of saturated fatty acids, whereas vegetable fats have higher amounts of unsaturated and polyunsaturated fatty acids. Linoleic acid, an unsaturated fatty acid, is the only essential fatty acid in humans. Diet recommendations include limiting saturated fat to less than 7% and trans fat to less than 1%.

A nurse is preparing a patient for a magnetic resonance imaging (MRI) scan. Which nursing action is most important? a. Ensuring that the patient does not eat or drink 2 hours before the examination. b. Administering a colon cleansing product 6 hours before the examination. c. Obtaining an order for a pain medication before the test is performed. d. Removing all of the patient's metallic jewelry.

ANS: D No jewelry or metal products should be in the same room as an MRI machine because of the high-power magnet used in the machine. The patient needs to be NPO 4 to 6 hours before the examination. Colon cleansing products are not necessary for MRIs. Pain medication is not needed before the examination is performed.

A patient has inadequate stroke volume related to decreased preload. Which treatment does the nurse prepare to administer? a. Diuretics b. Vasodilators c. Chest physiotherapy d. Intravenous (IV) fluids

ANS: D Preload is affected by the circulating volume; if the patient has decreased fluid volume, it will need to be replaced with fluid or blood therapy. Preload is the amount of blood in the left ventricle at the end of diastole, often referred to as end-diastolic volume. Giving diuretics and vasodilators will make the situation worse. Diuretics causes fluid loss; the patient is already low on fluids or the preload would not be decreased. Vasodilators reduced blood return to the heart, making the situation worse; the patient does not have enough blood and fluid to the heart or the preload would not be decreased. Chest physiotherapy is a group of therapies for mobilizing pulmonary secretions. Chest physiotherapy will not help this cardiovascular problem.

The nurse is caring for a patient with Clostridium difficile. Which nursing actions will have the greatest impact in preventing the spread of the bacteria? a. Appropriate disposal of contaminated items in biohazard bags b. Monthly in-services about contact precautions c. Mandatory cultures on all patients d. Proper hand hygiene techniques

ANS: D Proper hand hygiene is the best way to prevent the spread of bacteria. Soap and water are mandatory. Monthly in-services place emphasis on education, not on action. Biohazard bags are appropriate but cannot be used on every item that C. difficile comes in contact with, such as a human. Mandatory cultures are expensive and unnecessary and would not prevent the spread of bacteria.

The nurse is caring for a patient after an open abdominal aortic aneurysm repair. The nurse requests an abdominal binder and carefully applies the binder. Which is the best explanation for the nurse to use when teaching the patient the reason for the binder? a. It reduces edema at the surgical site. b. It secures the dressing in place. c. It immobilizes the abdomen. d. It supports the abdomen.

ANS: D The patient has a large abdominal incision. This incision will need support, and an abdominal binder will support this wound, especially during movement, as well as during deep breathing and coughing. A binder can be used to immobilize a body part (e.g., an elastic bandage applied around a sprained ankle). A binder can be used to prevent edema, for example, in an extremity but in this case is not used to reduce edema at a surgical site. A binder can be used to secure dressings such as elastic webbing applied around a leg after vein stripping

To obtain a clean-voided urine specimen from a female patient, what should the nurse teach the patient to do? a. Cleanse the urethral meatus from the area of most contamination to least. b. Initiate the first part of the urine stream directly into the collection cup. c. Drink fluids 5 minutes before collecting the urine specimen. d. Hold the labia apart while voiding into the specimen cup.

ANS: D The patient should hold the labia apart to reduce bacterial levels in the specimen. The urethral meatus should be cleansed from the area of least contamination to greatest contamination (or front to back). The initial stream flushes out microorganisms in the urethra and prevents bacterial transmission in the specimen. Drink fluids 30 minutes before giving a specimen.

. The nurse is managing bowel training for a patient. To which patient is the nurse most likely providing care? a. A 25-year-old patient with diarrhea b. A 30-year-old patient with Clostridium difficile c. A 40-year-old patient with an ileostomy d. A 70-year-old patient with stool incontinence

ANS: D The patient with chronic constipation or fecal incontinence secondary to cognitive impairment may benefit from bowel training, also called habit training. An ileostomy, diarrhea, and C. difficile all relate to uncontrollable bowel movements, for which no method can be used to set up a schedule of elimination.

The patient has a catheter that must be irrigated. The nurse is using a needleless closed irrigation technique. In which order will the nurse perform the steps, starting with the first one? 1. Clean injection port. 2. Inject prescribed solution. 3. Twist needleless syringe into port. 4. Remove clamp and allow to drain. 5. Clamp catheter just below specimen port. 6. Draw up prescribed amount of sterile solution ordered. a. 3, 2, 6, 1, 5, 4 b. 5, 6, 1, 2, 3, 4 c. 1, 5, 6, 3, 2, 4 d. 6, 5, 1, 3, 2, 4

ANS: D The steps for irrigating with a needleless closed irrigation technique is as follows: Draw up in a syringe the prescribed amount of medication or sterile solution; clamp indwelling retention catheter just below specimen port; using circular motion, clean injection port with antiseptic swab; insert tip of needleless syringe using twisting motion into irrigation port; slowly and evenly inject fluid into catheter and bladder; and withdraw syringe, remove clamp, and allow solution to drain into drainage bag.

The patient is to receive multiple medications via the nasogastric tube. The nurse is concerned that the tube may become clogged. Which action is best for the nurse to take? a. Instill nonliquid medications without diluting. b. Irrigate the tube with 60 mL of water after all medications are given. c. Mix all medications together to decrease the number of administrations. d. Check with the pharmacy for availability of the liquid forms of medications.

ANS: D Use liquid medications when available to prevent tube occlusion. Irrigate with 30 mL of water before and after each medication per tube. Completely dissolve crushed medications in liquid if liquid medication is not available. Read pharmacological information on compatibility of drugs and formula before mixing medications.

A nurse is inserting an indwelling urinary catheter for a male patient. Which action will the nurse take? a. Hold the shaft of the penis at a 60-degree angle. b. Hold the shaft of the penis with the dominant hand. c. Cleanse the meatus 3 times with the same cotton ball from clean to dirty. d. Cleanse the meatus with circular strokes beginning at the meatus and working outward.

ANS: D Using the uncontaminated dominant hand, cleanse the meatus with cotton balls/swab sticks, using circular strokes, beginning at the meatus and working outward in a spiral motion. Repeat 3 times using a clean cotton ball/swabstick each time. With the nondominant hand (now contaminated), retract the foreskin (if uncircumcised) and gently grasp the penis at the shaft just below the glans. Hold the shaft of the penis at a right angle to the body.

The nurse is completing a skin assessment on a patient with darkly pigmented skin. Which item should the nurse use first to assist in staging an ulcer on this patient? a. Disposable measuring tape b. Cotton-tipped applicator c. Sterile gloves d. Halogen light

ANS: D When assessing a patient with darkly pigmented skin, proper lighting is essential to accurately complete the first step in assessment—inspection—and the entire assessment process. Natural light or a halogen light is recommended. Fluorescent light sources can produce blue tones on darkly pigmented skin and can interfere with an accurate assessment. Other items that could possibly be used during the assessment include gloves for infection control, a disposable measuring device to measure the size of the wound, and a cotton-tipped applicator to measure the depth of the wound, but these items are not the first items used.

Virulence

Ability to survive in the host or outside the body

Kussmaul respirations

Abnormally deep, regular and and increased in rate.

Tachycardia

Abnormally rapid heartbeat. >100bpm

Biot's respirations

Abnormally shallow for 2-3 breaths followed by irregular period of apnea

Bradycardia

Abnormally slow heartbeat. <60bpm

pH

Acidity of the environment.

Back channeling

Active listening technique that prompts a respondent to continue telling a story or describing a situation. Involves use of phrases such as "Go on," "Uh huh," and "Tell me more."

Which action should the nurse take to best develop critical thinking skills? a. Study 3 hours more each night. b. Attend all inservice opportunities. c. Actively participate in clinical experiences. d. Interview staff nurses about their nursing experiences.

Actively participate in clinical experiences.

While caring for a hospitalized older-adult female post hip surgery, the nurse is faced with the task of inserting an indwelling urinary catheter, which involves rotating the hip into a contraindicated position. Which action should the nurse take? a. Postpone catheter insertion until the next shift. b. Adapt the positioning technique to the situation. c. Notify the health care provider for a urologist consult. d. Follow textbook procedure with contraindicated position.

Adapt the positioning technique to the situation.

Ligaments

Aid joint flexability and support.

The patient has contracted a urinary tract infection (UTI) while in the hospital. Which action will most likely increase the risk of a patient contracting a UTI? a. Reusing the patient's graduated receptacle to empty the drainage bag. b. Allowing the drainage bag port to touch the graduated receptacle. c. Emptying the urinary drainage bag at least once a shift. d. Irrigating the catheter infrequently.

Allowing the drainage bag port to touch the graduated receptacle.

Trapeze bar

Allows the patient to pull with the upper extremities to raise the trunk off the bed, assist in transfer, or to perform exercises

To prevent complications of immobility, what would be the most effective activity on the first postoperative day for a patient who has had abdominal surgery? 1. Turn, cough, and deep breathe every 30 minutes while awake 2. Ambulate patient to chair in the hall 3. Passive range of motion 4 times a day 4. Immobility is not a concern the first postoperative day

Ambulate patient to chair in the hall

Pathogen

An Infectious agent

Dysrhythmia

An abnormal heart rhythm, including early, late or missed beats.

Systemic infection

An infection that affects the entire body.

Communicable Disease

An infectious disease that can be transmitted directly from one person to another

Anticipate how a patient might respond to a treatment

Analyticity

. The surgical mask the perioperative nurse is wearing becomes moist. Which action will the perioperative nurse take next? a. Apply a new mask. b. Reapply the mask after it air-dries. c. Change the mask when relieved by next shift. d. Do not change the mask if the nurse is comfortable.

Apply a new mask

A nurse has seen many cancer patients struggle with pain management because they are afraid of becoming addicted ot the medicine. Pain control is a priority for cancer care. By helping patients focus on their values and beliefs about pain control, a nurse can best make clinical decisions. This is an example of: A. Creativity. B. Fairness. C. Clinical reasoning. D. Applying ethical criteria

Applying ethical criteria

The nurse is caring for a patient who has had a stroke causing total paralysis of the right side. To help maintain joint function and minimize the disability from contractures, passive ROM will be initiated. When should the nurse begin this therapy? a. After the acute phase of the disease has passed b. As soon as the ability to move is lost c. Once the patient enters the rehab unit d. When the patient requests it

As soon as the ability to move is lost

A nurse has already set the agenda during a patient-centered interview. What will the nurse do next? a. Begin with introductions. b. Ask about the chief concerns or problems. c. Explain that the interview will be over in a few minutes. d. Tell the patient "I will be back to administer medications in 1 hour."

Ask about the chief concerns or problems.

After reviewing the database, the nurse discovers that the patient's vital signs have not been recorded by the nursing assistive personnel (NAP). Which clinical decision should the nurse make? a. Administer scheduled medications assuming that the NAP would have reported abnormal vital signs. b. Have the patient transported to the radiology department for a scheduled x-ray, and review vital signs upon return. c. Ask the NAP to record the patient's vital signs before administering medications. d. Omit the vital signs because the patient is presently in no distress.

Ask the NAP to record the patient's vital signs before administering medications.

The nurse enters a room to find the patient sitting up in bed crying. How will the nurse display a critical thinking attitude in this situation? a. Provide privacy and check on the patient 30 minutes later. b. Set a box of tissues at the patient's bedside before leaving the room. c. Limit visitors while the patient is upset. d. Ask the patient about the crying.

Ask the patient about the crying.

A patient verbalizes a low pain level of 2 out of 10 but exhibits extreme facial grimacing while moving around in bed. What is the nurse's initial action in response to these observations? a. Proceed to the next patient's room to make rounds. b. Determine the patient does not want any pain medicine. c. Ask the patient about the facial grimacing with movement. d. Administer the pain medication ordered for moderate to severe pain.

Ask the patient about the facial grimacing with movement.

While the patient's lower extremity, which is in a cast, is assessed, the patient tells the nurse about an inability to rest at night. The nurse disregards this information, thinking that no correlation has been noted between having a leg cast and developing restless sleep. Which action would have been best for the nurse to take? a. Tell the patient to just focus on the leg and cast right now. b. Document the sleep patterns and information in the patient's chart. c. Explain that a more thorough assessment will be needed next shift. d. Ask the patient about usual sleep patterns and the onset of having difficulty resting.

Ask the patient about usual sleep patterns and the onset of having difficulty resting.

While completing an admission database, the nurse is interviewing a patient who states "I am allergic to latex." Which action will the nurse take first? a. Immediately place the patient in isolation. b. Ask the patient to describe the type of reaction. c. Proceed to the termination phase of the interview. d. Document the latex allergy on the medication administration record.

Ask the patient to describe the type of reaction.

A nurse is pulled from the surgical unit to work on the oncology unit. Which action by the nurse displays humility and responsibility? a. Refusing the assignment b. Asking for an orientation to the unit c. Admitting lack of knowledge and going home d. Assuming that patient care will be the same as on the other units

Asking for an orientation to the unit.

Which patient scenario of a surgical patient in pain is most indicative of critical thinking? a. Administering pain-relief medication according to what was given last shift. b. Offering pain-relief medication based on the health care provider's orders. c. Asking the patient what pain-relief methods, pharmacological and nonpharmacological, have worked in the past. d. Explaining to the patient that self-reporting of severe pain is not consistent with the minor procedure that was performed.

Asking the patient what pain-relief methods, pharmacological and nonpharmacological, have worked in the past.

The patient is being admitted to the neurological unit with a diagnosis of stroke. When will the nurse begin discharge planning? a. At the time of admission b. The day before the patient is to be discharged c. When outpatient therapy will no longer be needed d. As soon as the patient's discharge destination is known

At the time of admission

You notice a respiratory change in your immobilized postoperative patient. The change you note is most consistent with: A. Atelectasis. B. Hypertension. C. Orthistatic hypotension D. Coagulation of blood.

Atelectasis

The nurse is assessing the patient for respiratory complications of immobility. Which action will the nurse take when assessing the respiratory system? a. Inspect chest wall movements primarily during the expiratory cycle. b. Auscultate the entire lung region to assess lung sounds. c. Focus auscultation on the upper lung fields. d. Assess the patient at least every 4 hours.

Auscultate the entire lung region to assess lung sounds.

A nurse is caring for a group of patients. Which patient will the nurse see first? a. A 17-year-old male who has just returned from outside "for a smoke" who needs a temperature taken b. A 20-year-old male postoperative patient whose blood pressure went from 128/70 to 100/60 c. A 27-year-old male patient reporting pain whose blood pressure went from 124/70 to 130/74 d. An 87-year-old male suspected of hypothermia whose temperature is below normal

B

A patient with diabetes mellitus is starting an exercise program. Which types of exercises will the nurse suggest? a. Low intensity b. Low to moderate intensity c. Moderate to high intensity d. High intensity

B

The nurse is caring for a patient who is being discharged from the hospital after being treated for hypertension. The patient is instructed to take blood pressure 3 times a day and to keep a record of the readings. The nurse recommends that the patient purchase a portable electronic blood pressure device. Which other information will the nurse share with the patient? a. You can apply the cuff in any manner. b. You will need to recalibrate the machine. c. You can move your arm during the reading. d. You will need to use a stethoscope properly.

B

The nurse is caring for a patient who reports feeling light-headed and "woozy." The nurse checks the patient's pulse and finds that it is irregular. The patient's blood pressure is 100/72. It was 113/80 an hour earlier. What should the nurse do? a. Apply more pressure to the radial artery to feel pulse. b. Perform an apical/radial pulse assessment. c. Call the health care provider immediately. d. Obtain arterial blood gases.

B

The patient has new-onset restlessness and confusion. Pulse rate is elevated, as is respiratory rate. Oxygen saturation is 94%. The nurse ignores the pulse oximeter reading and calls the health care provider for orders because the pulse oximetry reading is inaccurate. Which factors can cause inaccurate pulse oximetry readings? (Select all that apply.) a. O2 saturations (SaO2) > 70% b. Carbon monoxide inhalation c. Hypothermic fingers d. Intravascular dyes e. Nail polish f. Jaundice

B, C, D, E, F.

The circulating nurse in the operating room is observing the surgical technologist while applying a sterile gown and gloves to care for a patient having an appendectomy. Which behaviors indicate to the nurse that the procedure by the surgical technologist is correct? (Select all that apply.) a. Ties the back of own gown b. Touches only the inside of gown c. Slips arms into arm holes simultaneously d. Extended fingers fully into both of the gloves e. Uses hands covered by sleeves to open gloves f. Applies surgical cap and face mask in the operating suite

B, C, D, E.

A nurse is working in the intensive care unit and must obtain core temperatures on patients. Which sites can be used to obtain a core temperature? (Select all that apply.) a. Rectal b. Tympanic c. Esophagus d. Temporal artery e. Pulmonary artery

B, C, E.

The nurse is preparing to insert a urinary catheter. The nurse is using open gloving to apply the sterile gloves. Which steps will the nurse take? (Select all that apply.) a. While putting on the first glove, touch only the outside surface of the glove. b. With gloved dominant hand, slip fingers underneath second glove cuff. c. Remove outer glove package by tearing the package open. d. Lay glove package on clean flat surface above waistline. e. Glove the dominant hand of the nurse first. f. After second glove is on, interlock hands.

B, D, E, F.

The nurse is caring for a patient in protective environment. Which actions will the nurse take? (Select all that apply.) a. Wear an N95 respirator when entering the patient's room. b. Maintain airflow rate greater than 12 air exchanges/hr. c. Place in special room with negative-pressure airflow. d. Open drapes during the daytime. e. Listen to the patient's interests. f. Place dried flowers in a plastic vase.

B, D, E.

A nurse is preparing to move a patient who is able to assist. Which principles will the nurse consider when planning for safe patient handling? (Select all that apply.) a. Keep the body's center of gravity high. b. Face the direction of the movement. c. Keep the base of support narrow. d. Use the under-axilla technique. e. Use proper body mechanics. f. Use arms and legs.

B, E, F.

The patient's blood pressure is 140/60. Which value will the nurse record for the pulse pressure? a. 60 b. 80 c. 140 d. 200

B. (140 − 60 = 80)

The nurse should do which of the following when placing a bedpan under an immobilized patient? A. Lift the patient's hips off the bed and slide the bedpan under the patient B. After positioning the patient on the bedpan, elevate the head of the bed to a 45-degree angle C. Adjust the head of the bed so it is lower than the feet and use gentle but firm pressure to push the bedpan under the patient D. Have the patient stand beside the bed and then have him or her sit on the bedpan on the edge of the be

B. After positioning the patient on the bedpan, elevate the head of the bed to a 45-degree angle Elevating the head of the bed allows the patient the most normal and comfortable position for defecation on a bedpan.

Which nursing interventions should the nurse implement when removing an indwelling urinary catheter in an adult patient? A. Attach a 3 mL syringe to the inflation port B. Allow the balloon to drain into the syringe by gravity. C. Initiate a voiding record/bladder diary D. Pull catheter quickly E. Clamp the catheter prior to removal.

B. Allow the balloon to drain into the syringe by gravity. C. Initiate a voiding record/bladder diary By allowing the balloon to drain by gravity the development of creases or ridges in the balloon may be avoided and thus minimize trauma to the urethra during withdrawal. All patients who have a catheter removed should have their voiding monitored. The best way to do this is with a voiding record or bladder diary. The size syringe used to deflate the balloon is dictated by the size of the balloon. In the adult patient balloon sizes are either 10 mLs or 30 mLs. Catheters should be pulled out slowly and smoothly. There is no evidence to support clamping catheters prior to removal.

The NAP reports to the nurse that a patient's catheter drainage bag has been empty for 4 hours. What is a priority nursing intervention? A. Implement the "as needed" order to irrigate the catheter. B. Assess the catheter and drainage tubing for obvious occlusion. C. Notify the health care provider immediately. D. Assess the vital signs and intake and output record.

B. Assess the catheter and drainage tubing for obvious occlusion. The priority nursing intervention is to ensure that there is not an occlusion in the catheter or drainage tubing.

A post-operative patient with a three-way indwelling urinary catheter and continuous bladder irrigation (CBI) complains of lower abdominal pain and distention. What should be the nurse's initial intervention? A. Increase the rate of the CBI B. Assess the intake and output C. Decrease the rate of the CBI D. Assess vital signs

B. Assess the intake and output An appropriate first action would be to assess the patency of the drainage system. Urine output in the drainage bag should be more than the volume of the irritant solution infused. If the system is not draining urine and irritant, the irritant should be stopped immediately, the catheter may be occluded and the bladder distended.

A patient is receiving total parenteral nutrition (TPN). What is the primary intervention the nurse should follow to prevent a central line infection? A. Institute isolation precautions B. Clean the central line port through which the TPN is infusing with alcohol C. Change the TPN tubing every 24 hours D.Monitor glucose levels to watch and assess for glucose intolerance

B. Clean the central line port through which the TPN is infusing with alcohol Use either alcohol or an alcoholic solution of chlorhexidine gluconate to clean the injection port or catheter hub 15 seconds before and after each time it is used to reduce the risk of a central line infection.

Which of the following may cause Clostridium difficile infection? A. Chronic laxative use B. Contact with C. difficile bacteria C. Overuse of antibiotics D. Frequent episodes of diarrhea caused by food intolerance E. Inflammation of the bowel

B. Contact with C. difficile bacteria C. Overuse of antibiotics These are the two main causes of C. difficile infection.

An elderly patient comes to the hospital with a complaint of severe weakness and diarrhea for several days. Of the following problems, which is the most important to assess initially? A. Malnutrition B. Dehydration C. Skin breakdown D. Incontinence

B. Dehydration Dehydration caused by fluid loss from the intestinal tract is an immediate and possibly dangerous consequence of diarrhea.

The nurse sees the nursing assistive personnel (NAP) perform the following intervention for a patient receiving continuous enteral feedings. Which action would require immediate attention? A. Fastening tube to the gown with new tape B. Placing patient supine while giving a bath C. Hanging a new container of enteral feeding D. Ambulating patient with enteral feedings still infusing

B. Placing patient supine while giving a bath A patient receiving continuous enteral feedings should never be placed supine because it increases the risk for pulmonary aspiration. If the nurse needs to lay the patient in the supine position, the feedings should be stopped and restarted when the head of the bed is at 45 degrees.

Which of the following is an indication for a binder to be placed around a surgical patient with a new abdominal wound? A. Collection of wound drainage B. Provides support to abdominal tissues when coughing or walking C. Reduction of abdominal swelling D. Reduction of stress on the abdominal incision E. Stimulation of peristalsis (return of bowel function) from direct pressure

B. Provides support to abdominal tissues when coughing or walking D. Reduction of stress on the abdominal incision A binder placed over the abdomen can provide protection to the abdominal incision by offering support and decreasing stress from coughing and movement.

What does the Braden Scale evaluate? A. Skin integrity at bony prominences, including any wounds B. Risk factors that place the patient at risk for skin breakdown C. The amount of repositioning that the patient can tolerate D. The factors that place the patient at risk for poor healing

B. Risk factors that place the patient at risk for skin breakdown The Braden Scale measures factors in six subscales that can predict the risk of pressure ulcer development. It does not assess skin or wounds.

An ambulatory elderly woman with dementia is incontinent of urine. She has poor short term memory and has not been seen toileting independently. What is the best nursing intervention for this patient? A. Recommend she be evaluated for an OAB medication. B. Start a scheduled toileting program. C. Recommend she be evaluated for an indwelling catheter. D. Start a bladder retraining program

B. Start a scheduled toileting program. The first nursing intervention for any patient with incontinence, who is able to toilet, is to assist them with toilet access. This patient is not cognitively intact so a bladder retraining program is not appropriate for her. It is not clear in this case that she has OAB and a catheter is never a good solution for incontinence.

Which patients are at high risk for nutritional deficits? Select all that apply A. The divorced computer programmer who eats precooked food from the local restaurant B. The middle-age female with celiac disease who does not follow her gluten-free diet C. The 45-year-old patient with type II diabetes who monitors her carbohydrate intake and exercises regularly Patients suffering from celiac disease or Crohn's disease need to take vitamin and iron supplements regularly because they have a deficit resulting from malabsorption. D. The 25-year-old patient with Crohn's disease who follows a strict diet but does not take vitamins or iron supplements E. The 65-year-old patient with gallbladder disease whose electrolyte, albumin, and protein levels are normal

B. The middle-age female with celiac disease who does not follow her gluten-free diet D. The 25-year-old patient with Crohn's disease who follows a strict diet but does not take vitamins or iron supplements Patients suffering from celiac disease or Crohn's disease need to take vitamin and iron supplements regularly because they have a deficit resulting from malabsorption.

The nurse would delegate which of the following to nursing assistive personnel (NAP)? Select all that apply A. Repositioning and retaping a patient's nasogastric tube B.Performing glucose monitoring every 6 hours on a patient C.Documenting PO intake on a patient who is on a calorie count for 72 hours D.Administering enteral feeding bolus after tube placement has been verified E.Hanging a new bag of enteral feeding

B.Performing glucose monitoring every 6 hours on a patient C.Documenting PO intake on a patient who is on a calorie count for 72 hours The skills of measuring blood glucose level after skin puncture (capillary puncture) and writing down the amount the patient ate can be delegated to NAP. The nurse needs to administer enteral feeding because of the risk of aspiration. The nasogastric tube should never be repositioned by the NAP for risk of causing injury to the patient.

The nurse is assessing the patient's respirations. Which action by the nurse is most appropriate? a. Inform the patient that she is counting respirations. b. Do not touch the patient until completed. c. Obtain without the patient knowing. d. Estimate respirations.

C

A nurse is preparing to assess a patient for orthostatic hypotension. Which piece of equipment will the nurse obtain to assess for this condition? a. Thermometer b. Elastic stockings c. Blood pressure cuff d. Sequential compression devices

Blood pressure cuff

The nurse is evaluating the body alignment of a patient in the sitting position. Which observation by the nurse will indicate a normal finding? a. The edge of the seat is in contact with the popliteal space. b. Both feet are supported on the floor with ankles flexed. c. The body weight is directly on the buttocks only. d. The arms hang comfortably at the sides.

Both feet are supported on the floor with ankles flexed.

A nurse is preparing medications for a patient. The nurse cheks the name of the medication on the label with the name of the medication on the doctor's order. At the bedside the nurse checks the patient's name against the medication order as well. The nurse is following which critical thinking attitude? A. Responsible. B. Complete. C. Accurate. D. Broad.

Broad

. A nurse is caring for a group of patients. Which patient will the nurse see first? a. A crying infant with P-165 and R-54 b. A sleeping toddler with P-88 and R-23 c. A calm adolescent with P-95 and R-26 d. An exercising adult with P-108 and R-24

C

A nurse is assisting the patient to perform isometric exercises. Which action will the nurse take? a. Encourage wearing tight shoes. b. Set the pace for the exercise session. c. Stop the exercise if pain is experienced. d. Force muscles or joints to go just beyond resistance.

C

A nurse is caring for a patient who smokes and drinks caffeine. Which point is important for the nurse to understand before assessing the patient's blood pressure (BP)? a. Smoking increases BP for up to 3 hours. b. Caffeine increases BP for up to 15 minutes. c. Smoking result in vasoconstriction, falsely elevating BP. d. Caffeine intake should not have occurred 30 to 40 minutes before BP measurement.

C

A nurse reviews blood pressures of several patients. Which finding will the nurse report as prehypertension? a. 98/50 in a 7-year-old child b. 115/70 in an infant c. 120/80 in a middle-aged adult d. 146/90 in an older adult

C

A postoperative patient is breathing rapidly. You should immediately; A. Call the physician. B. Count the respirations. C. Assess the oxygen satuation. D. Ask the patient if he feels uncomfortable.

C

A nurse is completing an assessment. Which findings will the nurse report as subjective data? (Select all that apply.) a. Patient's temperature b. Patient's wound appearance c. Patient describing excitement about discharge d. Patient pacing the floor while awaiting test results e. Patient's expression of fear regarding upcoming surgery

C, E.

What is a critical step when inserting an indwelling catheter into a male patient? A. Slowly inflate the catheter balloon with sterile saline. B. Secure the catheter drainage tubing to the bed sheets C. Advance the catheter to the bifurcation of the drainage and balloon ports. D. Advance the catheter until urine flows, then insert ¼ inch more.

C. Advance the catheter to the bifurcation of the drainage and balloon ports. Advancing the catheter to the bifurcation avoids inflating the catheter balloon in the prostatic urethra causing trauma and pain. Catheter balloons are never inflated with saline. Securing the catheter drainage tubing to the bed sheets increases the risk for accidental pulling or tension on the catheter. The advancement of the catheter until flows and then inserting ¼ inch more is not unique to the male patient.

The patient's blood glucose level is 330 mg/dL. What is the priority nursing intervention? A. Recheck by performing another blood glucose test. B. Call the primary health care provider. C. Check the medical record to see if there is a medication order for abnormal glucose levels. D. Monitor and recheck in 2 hours.

C. Check the medical record to see if there is a medication order for abnormal glucose levels. Check the medical record to see if there is a medication order for deviations in glucose level; if not, notify the health care provider. As the nurse you want to get the patient's blood sugar as close to normal as possible.

A patient is admitted with severe lobar pneumonia. Which of the following assessment findings would indicate that the patient needs airway suctioning? A. Coughing up thick sputum only occasionally B. Coughing up thin, watery sputum after nebulization C. Decreased ability to clear airway through couching D. Lung sounds clear only after coughing

C. Decreased ability to clear airway through couching Impaired ability to cough up mucous due to weakness or very thick secretions indicates a need for suctioning.

The nurse is taking a health history of a newly admitted patient with a diagnosis of possible fecal impaction. Which of the following is the priority question to ask the patient or caregiver? A. Have you eaten more high-fiber foods lately? B. Are your bowel movements soft and formed? C. Have you experienced frequent, small liquid stools recently? D. Have you taken antibiotics recently?

C. Have you experienced frequent, small liquid stools recently? Frequent or continuous oozing of liquid stools occurs when liquid fecal matter above the impacted stool seeps around the fecal impaction.

When a patient has fecal incontinence as a result of cognitive impairment, it may be helpful to teach caregivers to do which of the following interventions? A. Cleanse the skin with antibacterial soap and apply talcum powder to the buttocks B. Use diapers and heavy padding on the bed C. Initiate bowel or habit training program to promote continence D. Help the patient to toilet once every hour

C. Initiate bowel or habit training program to promote continence A cognitively impaired patient may have forgotten how to respond to the urge to defecate and benefit from a structured program of bowel retraining.

When assessing a patient's first voided urine of the day, which finding should be reported to the health care provider? A. Pale yellow urine B. Slightly cloudy urine C. Light pink urine D. Dark amber urine

C. Light pink urine Light pink urine indicates the presence of blood in the urine, which is never a normal finding. First voided urine can normally be slightly cloudy and darker in color. Pale yellow urine indicates normal finding.

What instructions should the nurse give the NAP concerning a patient who has had an indwelling urinary catheter removed that day? A. Limit oral fluid intake to avoid possible urinary incontinence. B. Expect patient complaints of suprapubic fullness and discomfort. C. Report the time and amount of first voiding. D. Instruct patient to stay in bed and use a urinal or bedpan.

C. Report the time and amount of first voiding. In order to adequately assess bladder function after a catheter is removed; voiding frequency and amount should be monitored. Unless contraindicated, fluids should be encouraged. To promote normal micturition, patients should be placed in as normal a posture for voiding as possible. Suprapubic tenderness and pain are possible indicators of urinary retention and/or a UTI.

The nurse is caring for a patient with an ileostomy. Which intervention is most important? A. Cleansing the stoma with hot water B. Inserting a deodorant tablet in the stoma bag C. Selecting or cutting a pouch with an appropriate-size stoma opening D. Wearing sterile gloves while caring for the stoma

C. Selecting or cutting a pouch with an appropriate-size stoma opening A properly fitting pouch that does not leave skin exposed prevents peristomal skin breakdown.

On assessing your patient's sacral pressure ulcer, you note that the tissue over the sacrum is dark, hard, and adherent to the wound edge. What is the correct category/stage for this patient's pressure ulcer? A. Category/Stage II B. Category/Stage IV C. Unstageable D. Suspected deep tissue damage

C. Unstageable To determine the category/stage of a pressure ulcer you examine the depth of the tissue involvement. Since the pressure ulcer assessed was covered with necrotic tissue, the depth could not be determined. Thus this pressure ulcer cannot be staged.

Fareinheit to Celsius

C= (F-32) x 5/9

The nurse is caring for a patient who exhibits labored breathing, using accessory muscles, and is coughing up pink frothy sputum. The patient has bilateral lung bases and diminished breath sounds. What are the priority nursing assessments for the nurse to perform prior to notifying the patient's health care provider? A. SpO2 levels B. Amount, color and consistency of sputum production C. Fluid status D. Change in respiratory rate and pattern E. Pain in lower leg

CORRECT A. SpO2 levels B. Amount, color and consistency of sputum production D. Change in respiratory rate and pattern These are key respiratory assessments that provide data on patient's worsening respiratory status. While fluid status does impact respiratory status, it is not a priority assessment at this time. Pain in lower leg is assessed later.

Clinical Decision Making

Careful reasoning so that the best options are chosen for the best outcomes.

A nurse is preparing to reposition a patient. Which task can the nurse delegate to the nursing assistive personnel? a. Determining the level of comfort b. Changing the patient's position c. Identifying immobility hazards d. Assessing circulation

Changing the patient's position

All of the following measures ae used to assess for deep vein thrombosis except: 1. Checking for a positive Homan's sign. 2. Asking the patient about the presence of calf pain. 3. Observing the dorsal aspect of lower extremeties for redness, warmth, and tenderness. 4. Measuring the circumference of each leg daily, placing the tape measure at the midpoint of the knee.

Checking for a positive Homan's sign.

Reason for Seeking Health Care

Chief concerns or problem.

The nurse observes a patient walking down the hall with a shuffling gait. When the patient returns to bed, the nurse checks the strength in both of the patient's legs. The nurse applies the information gained to suspect that the patient has a mobility problem. This conclusion is an example of: 1. Cue. 2. Reflection. 3. Clinical inference. 4. Probing.

Clinical inference.

Hypothermia

Cold that overwhelms the body's ability to produce heat.

Atelectasis

Collapse of the alveoli

A patient presents with heatstroke. The nurse uses cool packs, cooling blanket, and a fan. Which technique is the nurse using when the fan produces heat loss? a. Radiation b. Conduction c. Convection d. Evaporation

Convection

Effective communication skills

Courtesy. Comfort. Connection. Confirmation.

The nurse is caring for a patient whose calcium intake must increase because of high risk factors for osteoporosis. Which of the following menus should the nurse recommend? 1. Cream of broccoli soup with whole wheat crackers, cheese, and tapioca for dessert 2. Hot dog on whole wheat bun with a side salad and an apple for dessert 3. Low-fat turkey chili with sour cream with a side salad and fresh pears for dessert 4. Turkey salad on toast with tomato and lettuce and honey bun for dessert

Cream of broccoli soup with whole wheat crackers, cheese, and tapioca for dessert

A patient is having trouble reaching the water fountain while holding on to crutches. The nurse suggests that the patient place the crutches against the wall while stabilizing him or herself with two hands on the water fountain. Which critical thinking attitude did the nurse use in this situation? a. Humility b. Creativity c. Risk taking d. Confidence

Creativity

The nursing process organizes your approach while delivering nursing care. To provide the best professional care to patients, nurses need to incorporate nursing process and: A. Decision making. B. Problem solving. C. Intellectual standards. D. Critical thinking skills.

Critical thinking skills

A patient is admitted with a stroke. The outcome of this disorder is uncertain, but the patient is unable to move the right arm and leg. The nurse starts passive range-of-motion (ROM) exercises. Which finding indicates successful goal achievement? a. Heart rate decreased. b. Contractures developed. c. Muscle strength improved. d. Joint mobility maintained.

D

The nurse is caring for a patient who has a pulse rate of 48. His blood pressure is within normal limits. Which finding will help the nurse determine the cause of the patient's low heart rate? a. The patient has a fever. b. The patient has possible hemorrhage or bleeding. c. The patient has chronic obstructive pulmonary disease (COPD). d. The patient has calcium channel blockers or digitalis medication prescriptions.

D

A nurse is reviewing capnography results for adult patients. Which value will cause the nurse to follow up? a. 35 mm Hg b. 40 mm Hg c. 45 mm Hg d. 50 mm Hg

D. Normal capnography results are 35 to 45 mm Hg.

Match the following steps for administering a prepackaged enema with the correct order in which they occur. 1. Insert enema tip gently in the rectum. 2. Help patient to bathroom when he or she feels urge to defecate. 3. Position patient on side. 4. Perform hand hygiene and apply clean gloves. 5. Squeeze contents of container into rectum. 6. Explain procedure to the patient. A. 6, 3, 4, 1, 5, 2 B. 6, 4, 1, 3, 2, 5 C. 4, 6, 3, 1, 2, 5 D. 6, 4, 3, 1, 5, 2

D. 6, 4, 3, 1, 5, 2 These steps are used to administer a prepackaged enema.

A patient has been newly diagnosed with chronic lung disease. In discussing his condition with the nurse, which of his statements would indicate a need for further education? A. "I'll make sure that I rest between activities so I don't get so short of breath." B. "I'll practice the pursed-lip breathing technique to improve my exercise tolerance." C. "If I have trouble breathing at night, I'll use two to three pillows to prop up." D. "If I get short of breath, I'll turn up my oxygen level to 6 L/min."

D. "If I get short of breath, I'll turn up my oxygen level to 6 L/min." Hypoxia is the drive to breathe in a COPD patient who has become used to acidic pH and elevated CO2 levels. Turning up to 6 L/min will increase oxygen level which turns off the drive to breathe.

What is the correct order for an ostomy pouch change? 1. Close the end of the pouch. 2. Measure the stoma. 3. Cut the hole in the wafer. 4. Press the pouch in place over the stoma. 5. Remove the old pouch. 6. Trace the correct measurement onto the back of the wafer. 7. Assess the stoma and the skin around it. 8. Cleanse and dry the peristomal skin. A. 5, 8, 2, 7, 3, 6, 4, 1 B. 8, 5, 6, 2, 7, 3, 4, 1 C. 8, 5, 7, 6, 2, 3, 4, 1 D. 5, 8, 7, 2, 6, 3, 4, 1

D. 5, 8, 7, 2, 6, 3, 4, 1 This order of tasks describes the correct way to change an ostomy pouch.

Negative outcomes of immobility: Infants, toddlers, and preschoolers.

Delays in gross motor skills, intellectual development, or musculoskeletal development.

Which of the following describes a hydrocolloid dressing? A. A seaweed derivative that is highly absorptive B. Premoistened gauze placed over a granulating wound C. A debriding enzyme that is used to remove necrotic tissue D. A dressing that forms a gel that interacts with the wound surface

D. A dressing that forms a gel that interacts with the wound surface A hydrocolloid dressing is made of materials that are adhesive and can form a gel over the open area of the wound. Since moisture enhances wound healing, the gel that forms places the wound in the proper environment for healing.

The nurse is reviewing the results of the patient's diagnostic testing. Of the following results, the finding that falls within expected or normal limits is: A. Palpable, elevated hardened area around a tuberculosis skin testing site. B. Sputum for culture and sensitivity identifies mycobacterium tuberculosis C. Presence of acid fast bacilli in sputum D. Arterial oxygen tension (PaO2) of 95 mmHg

D. Arterial oxygen tension (PaO2) of 95 mmHg A palpable, elevated, hardened area surrounding a tuberculosis skin testing site is indicative of an antigen-antibody reaction and is considered a positive skin test. Sputum for culture and sensitivity noted the presence of an organism and acid fast bacilli. Normal arterial oxygen tension (PaO2) ranges between 95-100 mmHg.

When repositioning an immobile patient, the nurse notices redness over the hip bone. What is indicated when a reddened area blanches on fingertip touch? A. A local skin infection requiring antibiotics B. Sensitive skin that requires special bed linen C. A stage III pressure ulcer needing the appropriate dressing D. Blanching hyperemia, indicating the attempt by the body to overcome the ischemic episode.

D. Blanching hyperemia, indicating the attempt by the body to overcome the ischemic episode. When repositioning an immobile patient, it is important to assess all bony prominences for the presence of redness, which can be the first sign of impaired skin integrity. Pressing over the area compresses the blood vessels in the area; and, if the integrity of the vessels is good, the area turns lighter in color and then returns to the red color. However, if the area does not blanch when pressure is applied, tissue damage is likely.

Which of the following diagnosis is a patient who started smoking in adolescence and continues to smoke for 40 years at this risk for? A. Alcoholism and hypertension B. Obesity and diabetes C. Stress-related illnesses D. Cardiopulmonary disease and lung cancer

D. Cardiopulmonary disease and lung cancer Effects of nicotine on blood vessels and lung tissue have increasing pathological effects on the cardiovascular and pulmonary systems.

When obtaining a wound culture to determine the presence of a wound infection, from where should the specimen be taken? A. Necrotic tissue B. Wound drainage C. Wound circumference D. Cleansed wound

D. Cleansed wound Drainage that has been present on the wound surface can contain bacteria from the skin, and the culture may not contain the true causative organisms of a wound infection. By cleaning the area before obtaining the culture, the skin flora is removed.

A patient has been diagnosed with severe iron deficiency anemia. During physical assessment, which of the following symptoms are associated with decreased oxygenation? A. Increased breathlessness but increased activity tolerance B. Decreased breathlessness and decreased activity tolerance C. Increased activity tolerance and decreased breathlessness D. Decreased activity tolerance and increased breathlessness

D. Decreased activity tolerance and increased breathlessness Hypoxia occurs due to decreased circulating blood volume which leads to decreased oxygen to muscles causing fatigue and decreased activity tolerance as well as a feeling of shortness of breath.

A patient's gastric residual volume was 250 mL at 0800 and 350 mL at 1200. What is the appropriate nursing action? A. Assess bowel sounds B. Raise the head of the bed to at least 45 degrees C.Position the patient on his or her right side to promote stomach emptying D. Do not reinstall aspirate and hold the feeding until you talk to the primary care provider

D. Do not reinstall aspirate and hold the feeding until you talk to the primary care provider Do not administer feeding when a single gastric residual volume exceeds 500 mL or when two consecutive measurements (taken 1 hour apart) each exceed 250 mL because of the potential for aspiration.

Which statement made by a patient of a 2-month-old infant requires further education? A. I'll continue to use formula for the baby until he is a least a year old. B. I'll make sure that I purchase iron-fortified formula. C. I'll start feeding the baby cereal at 4 months. D. I'm going to alternate formula with whole milk starting next month.

D. I'm going to alternate formula with whole milk starting next month. Infants should not have regular cow's milk during the first year of life. It is too concentrated for the infant's kidneys to manage. There is also an increased risk for developing milk-product allergies.

A patient is admitted with the diagnosis of severe left-sided heart failure. What adventitious lung sounds are expected on auscultation? A. Sonorous wheezes in the left lower lung B. Rhonchi mid sternum C. Crackles only in apex of lungs D. Inspiratory crackles in lung bases

D. Inspiratory crackles in lung bases Decreased effective contraction of left side of heart leads to back up of fluid in the lungs increasing hydrostatic pressure and causing pulmonary edema.

During the nursing assessment a patient reveals that he has diarrhea and cramping every time he has ice cream. He attributes this to the cold nature of the food. However, the nurse begins to suspect that these symptoms are associated with what problem? A. Food allergy B. Irritable bowel C. Increased peristalsis D. Lactose intolerance

D. Lactose intolerance These symptoms are consistent with lactose intolerance, and they occur with ingestion of dairy products.

Body mechanics

Describes the coordinated efforts of the musculoskeletal and nervous system

The nurse is caring for a patient with dysphagia and is feeding her a pureed chicken diet when she begins to choke. What is the priority nursing intervention? A. Suction her mouth and throat B. Turn her on their side C. Put on oxygen at 2-L nasal cannula D. Stop feeding her and place on NPO

D. Stop feeding her and place on NPO Stop feeding and place patient on NPO. If choking persists, suction airway. Notify health care provider

A patient is receiving both parenteral (PN) and enteral nutrition (EN). When would the nurse collaborate with the health care provider and request discontinuing parenteral nutrition? A.When 25% of the patient's nutritional needs are met by the tube feedings B. When bowel sounds return C. When central line has been in for 10 days D. When 75% of the patient's nutritional needs are met by the tube feedings

D. When 75% of the patient's nutritional needs are met by the tube feedings When meeting 75% of nutritional needs by enteral feedings or reliable dietary intake, it is usually safe to discontinue PN therapy.

The patient has been in bed for several days and needs to be ambulated. Which action will the nurse take first? a. Maintain a narrow base of support. b. Dangle the patient at the bedside. c. Encourage isometric exercises. d. Suggest a high-calcium diet.

Dangle the patient at the bedside.

A patient is admitted to the hospital with shortness of breath. As the nurse assesses this patient the nurse is using the process of: A. Evaluation. B. Data Collection. C. Problem Identification. D. Testing a Hypothesis.

Data Collection

Orthostatic hypotension

Decrease in blood pressure related to positional or postural changes from lying to sitting or standing positions

Variation in pulse Older Adult

Decreased heart rate at rest. Once elevated pulse rate takes longer to return to normal.

Hypoxemia

Decreased level of oxygen in the blood

Which of the following are physiological outcomes of immobility? 1. Increased metabolism 2. Reduced cardiac workload 3. Decreased lung expansion 4. Decreased oxygen demand

Decreased lung expansion

A patient has been on bed rest for over 4 days. On assessment, the nurse identifies the following as a sign associated with immobility: 1. Decreased peristalsis 2. Decreased heart rate 3. Increased blood pressure 4. Increased urinary output

Decreased peristalsis

Body alignment

Determines normal physiological changes, identifies deviations, learning needs, trauma and risk.

Diagnostic Reasoning

Determining a patient's health status after you have assigned meaning to the behaviors and symptoms presented.

A nurse enters a 72-year-old patient's home and begins to observe and examine her condition. The nurse learns that the patient lives alone and notices bruising on the patients leg. When watching the patient walk, the nurse notes that she has an unsteady gait and leans to one side. The patient admits to having fallen in the past. The nurse identifies the patient as having the nursing diagnosis of risk for falls. This scenario is an example of? 1. Inference. 2. Basic critical thinking. 3. Evaluation. 4. Diagnostic reasoning.

Diagnostic reasoning.

Pulse defecit

Difference between the apical and radial pulse rates

While obtaining a health history, the nurse ask Mr. Jones if he has noticed any change in his activity tolerance. This is an example of which interview technique? 1. Direct question. 2. Problem solving. 3. Problem seeking. 4. Open-ended question.

Direct question

By using known criteria in conducting an assessment such as reviewing with a patient the typical characteristics of pain, a nurse is demonstrating which critical thinking attitude? 1. Curiosity 2. Adequacy 3. Discipline 4. Thinking independently

Discipline

Integrity

Do not compromise nursing standards or honesty in delivering nursing care.

A diabetic patient presents to the clinic for a dressing change. The wound is located on the right foot and has purulent yellow drainage. Which action will the nurse take to prevent the spread of infection? a. Position the patient comfortably on the stretcher. b. Explain the procedure for dressing change to the patient. c. Review the medication list that the patient brought from home. d. Don gloves and other appropriate personal protective equipment.

Don gloves and other appropriate personal protective equipment.

Airborne mode of transmission

Droplets that suspend in air

A nurse is providing passive range of motion (ROM) for a patient with impaired mobility. Which technique will the nurse use for each movement? a. Each movement is repeated 5 times by the patient. b. Each movement is performed until the patient experiences pain. c. Each movement is completed quickly and smoothly by the nurse. d. Each movement is moved just to the point of resistance by the nurse.

Each movement is moved just to the point of resistance by the nurse.

The nurse is caring for a school-aged child who has injured the right leg after a bicycle accident. Which signs and symptoms will the nurse assess for to determine if the child is experiencing a localized inflammatory response? a. Malaise, anorexia, enlarged lymph nodes, and increased white blood cells b. Chest pain, shortness of breath, and nausea and vomiting c. Dizziness and disorientation to time, date, and place d. Edema, redness, tenderness, and loss of function

Edema, redness, tenderness, and loss of function

Which of the following is an appropirate intervention to maintain the respiratory system of the immobilized patient? 1. Turn the patient every 4 hours. 2. Maintain a maximum fluid intake of 1500 mL/day. 3. Apply an abdominal binder continuously while the patient is in bed. 4. Encourage the patient to deep breathe and cough every 1 to 2 hours.

Encourage the patient to deep breathe and cough every 1 to 2 hours.

The nurse is caring for an older-adult patient who has been diagnosed with a stroke. Which intervention will the nurse add to the care plan? a. Encourage the patient to perform as many self-care activities as possible. b. Provide a complete bed bath to promote patient comfort. c. Coordinate with occupational therapy for gait training. d. Place the patient on bed rest to prevent fatigue.

Encourage the patient to perform as many self-care activities as possible.

Probing

Encourages a full description without trying to control the direction of the story.

A nurse is caring for an older adult who has had a fractured hip repaired. In the first few postoperative days, which of the following nursing measures will best facilitate the resumption of activities of daily living for this patient? 1. Encouraging use of an overhead trapeze for positioning and transfer 2. Frequent family visits 3. Assisting the patient to a wheelchair once per day 4. Ensuring that there is an order for physical therapy

Encouraging use of an overhead trapeze for positioning and transfer

Curiosity

Explore and learn more about a patient to make appropriate clinical judgements.

Present illness/health concerns

Essential and relevant data about the nature and onset of symptoms

A nurse is developing an individualized plan of care for a patient. Which action is important for the nurse to take? a. Establish goals that are measurable and realistic. b. Set goals that are a little beyond the capabilities of the patient. c. Use the nurse's own judgment and not be swayed by family desires. d. Explain that without taking alignment risks, there can be no progress.

Establish goals that are measurable and realistic.

The infection control nurse is reviewing data for the medical-surgical unit. The nurse notices an increase in postoperative infections from Aspergillus. Which type of health care-associated infection will the nurse report? a. Vector b. Exogenous c. Endogenous d. Suprainfection

Exogenous

Biographical information

Factual demographic data about the patient

Pyrexia

Fever

Nursing Process

Five-step clinical decision-making approach

A nurse is using the problem-oriented approach to data collection. Which action will the nurse take first? a. Complete the questions in chronological order. b. Focus on the patient's presenting situation. c. Make accurate interpretations of the data. d. Conduct an observational overview.

Focus on the patient's presentin situation.

Decision Making

Focuses on problem resolution

Shear

Force exerted against the skin while the skin remains stationary and the bony structures move

Friction

Force that occurs in a direction to oppose movement

Which of the following is not an element in the development or chain of infection. 1. Means of transmission. 2. Infectious agent or pathogen. 3. Formation of immunoglobulin. 4. Reservoir for pathogen growth.

Formation of immunoglobulin

The nurse is admitting a patient who has been diagnosed as having had a stroke. The health care provider writes orders for "ROM as needed." What should the nurse do next? a. Restrict patient's mobility as much as possible. b. Realize the patient is unable to move extremities. c. Move all the patient's extremities. d. Further assess the patient.

Further assess the patient.

The nurse is observing the way a patient walks. Which aspect is the nurse assessing? a. Activity tolerance b. Body alignment c. Range of motion d. Gait

Gait

Contact precautions require:

Gown and gloves.

The nurse is changing linens for a postoperative patient and feels a prick in the left hand. A nonactivated safe needle is noted in the linens. For which condition is the nurse most at risk? a. Diphtheria b. Hepatitis B c. Clostridium difficile d. Methicillin-resistant Staphylococcus aureus

Hepatitis B

Malignant hyperthermai

Hereditary condition of uncontrolled heat production.

The use of diagnostic reasoning involves a rigorous approach to clinical practice and demonstrates that critical thinking cannot be done: A. Logically. B. Haphazardly. C. Independently. D. Systematically.

Haphazardly

Immunocompromised

Having an impaired immune system

Fowlers position

Head of bed elevated 45-60 degrees and the knees slighly elevated.

Acute inflammation: Tissue repair

Healing involves the defensive, reconstructive, and maturative stages.

Health history

Health care experiences and current health habits and lifestyle patterns.

A nurse assesses a patient who comes to the pulmonary clinic. "I see that it's been over 6 months since you've been here, but your appointment was for every 2 months. Tell me about that. Also I see from your last visit that the doctor recommended routine exercise. Can you tell me how successful you've been in following his plan?" The nurse's assessment covers which of Gordon's functional health patterns? 1. Value-belief pattern. 2. Cognitive-perceptual pattern. 3. Coping-stress tolerance pattern. 4. Health perception-health management pattern.

Health perception- health management pattern.

The nurse is assessing a patient who she suspects has the nursing diagnosis hyperthermia related to vigarous exercise in hot weather. In reviewing the data, the nurse knows that the most important sign of heatstroke is: 1. Confusion. 2. Excess thirst. 3. Hot, dry skin. 4. Muscle cramps.

Hot dry skin. Victims of heat stroke do not sweat.

The nurse is caring for a patient who has an elevated temperature. Which principle will the nurse consider when planning care for this patient? a. Hyperthermia and fever are the same thing. b. Hyperthermia is an upward shift in the set point. c. Hyperthermia occurs when the body cannot reduce heat production. d. Hyperthermia results from a reduction in thermoregulatory mechanisms.

Hyperthermia occurs when the body cannot reduce heat production.

The nurse is caring for a patient who has a bloodborne pathogen. The nurse splashes blood above the glove to intact skin while discontinuing an intravenous (IV) infusion. Which step(s) will the nurse take next? a. Obtain an alcohol swab, remove the blood with an alcohol swab, and continue care. b. Immediately wash the site with soap and running water, and seek guidance from the manager. c. Do nothing; accidentally getting splashed with blood happens frequently and is part of the job. d. Delay washing of the site until the nurse is finished providing care to the patient.

Immediately wash the site with soap and running water, and seek guidance from the manager.

A nurse is assessing pressure points in a patient placed in the Sims' position. Which areas will the nurse observe? a. Chin, elbow, hips b. Ileum, clavicle, knees c. Shoulder, anterior iliac spine, ankles d. Occipital region of the head, coccyx, heels

Ileum, clavicle, knees

A patient has damage to the cerebellum. Which disorder is most important for the nurse to assess? a. Imbalance b. Hemiplegia c. Muscle sprain d. Lower extremity paralysis

Imbalance

The severity of a patients illness depends on all of the following except: 1. Incubation period. 2. Extent of infection. 3. Susceptibility of the host. 4. Pathogenicity of the microorganism.

Incubation period

Susceptibility

Individual's degree of resistance to pathogens

Reflection

Involved purposeful thinking back or recalling a situation to discover its purpose.

The nurse is assessing body alignment for a patient who is immobilized. Which patient position will the nurse use? a. Supine position b. Lateral position c. Lateral position with positioning supports d. Supine position with no pillow under the patient's head

Lateral position

An older-adult patient has been bedridden for 2 weeks. Which of the following complaints by the patient indicates to the nurse that he or she is developing a complication of immobility? 1. Loss of appetite 2. Gum soreness 3. Difficulty swallowing 4. Left ankle joint stiffness

Left ankle joint stiffness

Close-ended questions

Limit the patient's answers to one or two words.

Fairness

Listen to both sides in any discussion

Creativity

Look for different approaches if interventions are not working.

A nurse is preparing a care plan for a patient who is immobile. Which psychosocial aspect will the nurse consider? a. Loss of bone mass b. Loss of strength c. Loss of weight d. Loss of hope

Loss of hope

The patient is immobilized after undergoing hip replacement surgery. Which finding will alert the nurse to monitor for hemorrhage in this patient? a. Thick, tenacious pulmonary secretions b. Low-molecular-weight heparin doses c. SCDs wrapped around the legs d. Elastic stockings (TED hose)

Low-molecular-weight heparin doses

Variation in temp. Older Adult

Lower end of normal temperature. 96.8-98.3 F orally. 98-99 F rectally.

Prone

Lying face or chest down

A charge nurse is supervising the care of a new nurse. Which action by a new nurse indicates the charge nurse needs to intervene? a. Making an ethical clinical decision. b. Making an informed clinical decision. c. Making a clinical decision in the patient's best interest. d. Making a clinical decision based on previous shift assessments.

Making a clinical decision based on previous shift assessments.

The nurse is assessing an immobile patient for deep vein thromboses (DVTs). Which action will the nurse take? a. Remove elastic stockings every 4 hours. b. Measure the calf circumference of both legs. c. Lightly rub the lower leg for redness and tenderness. d. Dorsiflex the foot while assessing for patient discomfort.

Measure the calf circumference of both legs.

A nurse is caring for a patient with osteoporosis and lactose intolerance. What will the nurse do? a. Encourage dairy products. b. Monitor intake of vitamin D. c. Increase intake of caffeinated drinks. d. Try to do as much as possible for the patient.

Monitor intake of vitamin D.

Normal Flora

Microorganisms that live in the body that maintain a sensitive balance with other microorganisms to prevent infection. -Skin, saliva, oral mucosa, intestinal walls.

Ventilation

Movement of gases into and out of the lungs

Diffusion

Movement of oxygen and carbon dioxide between alveoli and red blood cells.

A nurse is providing range of motion to the shoulder and must perform external rotation. Which action will the nurse take? a. Moves patient's arm in a full circle b. Moves patient's arm cross the body as far as possible c. Moves patient's arm behind body, keeping elbow straight d. Moves patient's arm until thumb is upward and lateral to head with elbow flexed

Moves patient's arm until thumb is upward and lateral to head with elbow flexed

When the patient is immobile the body excretes more oxygen resulting in:

Negative nitrogen balance.

You are caring for a patient who underwent surgery 48 hours ago On physical assessment, you notice that the wound looks red and swollen. The patient's WBC's are elevated. You should: A. Start antibiotics. B. Notify the provider. C. Document the findings and reassess in 2 hours. D. Place the patient on isolation precautions.

Notify the provider.

Cartilage

Nonvascular, firm connective tissue

Observation

Nonverbal communication

Colonization

Organism that multiplies within a host but does not cause an infection

A nurse is reviewing care plans. Which finding, if identified in a plan of care, should the registered nurse revise? a. Patient's outcomes for learning. b. Nurse's assumptions about hospital discharge. c. Identification of several actual health problems. d. Documentation of patient's ability to meet the goal.

Nurse's assumptions about hospital discharge.

A patient presents with pneumonia. Which priority intervention should be included in the plan of care for this patient? a. Observe the patient for decreased activity tolerance. b. Assume the patient is in pain and treat accordingly. c. Provide the patient ice chips as requested. d. Maintain the room temperature at 65° F.

Observe the patient for decreased activity tolerance.

Problem Solving

Obtaining information, then use it plus what you already know to find a solution.

Endogenous infection

Occurs when part of the normal flora becomes altered and overgrowth results.

Frostbite

Occurs when the body is exposed to subnormal temperatures

Variation in Respirations Older Adult

Older adults depend more on accessory abdnominal muscles during respirations on weaker thoracic muscles.

Which type of interview question does the nurse first use when assessing the reason for patient seeking health care? 1. Probing. 2. Open-ended. 3. Problem-oriented. 4. Confirmation.

Open ended

The interview technique that is most effective in strengthening the nurse-patient relationship by demonstrating the nurse's willingness to hear the patient's thoughts is: 1. Direct question. 2. Problem solving. 3. Problem seeking. 4. Open-ended question

Open ended question: Prompts the patient to describe a situation in more than one or two words.

Be tolerant of the patient's views and beliefs.

Open-mindedness

SaO2

Oxygen saturation. Normal range is 95% to 100%

A nurse is conducting a nursing health history. Which component will the nurse address? a. Nurse's concerns b. Patient expectations c. Current treatment orders d. Nurse's goals for the patient

Patient expectations

A nurse delegates a position change to a nursing assistive personnel. The nurse instructs the NAP to place the patient in the lateral position. Which finding by the nurse indicates a correct outcome? a. Patient is lying on side. b. Patient is lying on back. c. Patient is lying semiprone. d. Patient is lying on abdomen.

Patient is lying on side.

Side-lying position

Patient rests on the side with the major portion of body weight on the dependent hip and shoulder.

Environmental History

Patient's home and work, focusing on determining the patient's safety.

A nurse is checking a patient's intravenous line and while doing so, notices how the patient bathes himself and then sits on the side of the bed independently to put on a new gown. This observation is an example of assessing: 1. Patient's level of function. 2. Patient's willingness to perform self-care. 3. Patient's level of consciousness. 4. Patient's health management values.

Patient's level of function

Patient Expectations

Patient's understanding of why he or she is seeking health care

A nurse is gathering information about a patient's habits and lifestyle patterns. Which method of data collection will the nurse use that will best obtain this information? a. Carefully review lab results. b. Conduct the physical assessment. c. Perform a thorough nursing health history. d. Prolong the termination phase of the interview.

Perform a thorough nursing health history.

Which method of data collection will the nurse use to establish a patient's database? a. Reviewing the current literature to determine evidence-based nursing actions b. Checking orders for diagnostic and laboratory tests c. Performing a physical examination d. Ordering medications

Performing a physical examination

. The home health nurse is teaching a patient and family about hand hygiene in the home. Which situation will cause the nurse to emphasize washing hands before and after? a. Shaking hands b. Performing treatments c. Opening the refrigerator d. Working on a computer

Performing treatments

Incubation period

Period between exposure to an infection and the appearance of the first symptoms.

Direct mode of transmisson

Person-to-person pr physical source and susceptible host.

Indirect mode of transmission

Personal contact of susceptible host with contaminated inanimate object

Exercise and activity tolerance

Physical activity for conditioning the body, improving health, and maintaining fitness.

A nurse is focusing on temperature regulation of newborns and infants. Which action will the nurse take? a. Apply just a diaper. b. Double the clothing. c. Place a cap on their heads. d. Increase room temperature to 90 degrees.

Place a cap on their heads.

The nurse is caring for a patient who needs to be placed in the prone position. Which action will the nurse take? a. Place pillow under the patient's abdomen after turning. b. Turn head toward one side with large, soft pillow. c. Position legs flat against bed. d. Raise head of bed to 45 degrees.

Place pillow under the patient's abdomen after turning.

When a patient of respiratory isolation must be transported to another part of the hospital, the nurse: 1. Places a mask on the patient before leaving the room. 2. Obtains a health care providers order to prohibit the patient from being transported. 3. Instructs the patient to cover his or her mouth or nose with a tissue when coughing or sneezing. 4. Advises other healthteam members to wear mask and gowns when comin in contact with the patient.

Places a mask on the patient before leaving the room.

Which of the following is a potential hazard that you should assess when the patient is in the prone position? 1. Plantar flexion. 2. Increased cervical flexion. 3. Internal rotation of the shoulder. 4. Unprotected pressure points at the sacrum and heels.

Plantar flexion

Motivational Interviewing

Process that addresses a patient's ambivilence to medically indicated behavior change and supports patients in making health care decisions.

A patient on prolonged bed rest is at an increased risk to develop this common complication of immobility if preventive measures are not taken: 1. Myoclonus 2. Pathological fractures 3. Pressure ulcers 4. Pruritus

Pressure ulcers

Complications of immobility in relation to the integumentary system

Pressure ulcers

A nurse is performing passive range of motion (ROM) and splinting on an at-risk patient. Which finding will indicate goal achievement for the nurse's action? a. Prevention of atelectasis b. Prevention of renal calculi c. Prevention of pressure ulcers d. Prevention of joint contractures

Prevention of joint contractures

Bacteriostasis

Prevention of the growth and reproduction of bacteria by cold temperatures

Trochanter roll

Prevents external rotation of the hips when pt is in a supine position

Mr. Davis tells the nurse that he has been experiencing more frequent episodes of indigestion. The nurse asks if the indigestion is associated with meals or a reclining position and asks what relieves the indigestion. This is an example of which interview technique? 1. Direct question 2. Problem solving. 3. Problem seeking. 4. Open-ended question.

Problem seeking

The nurse begins a shift assessment by examining a surgical dressing that is saturated with serosanguineous drainage on a patient who had open abdominal surgery yesterday (or 1 day ago). Which type of assessment approach is the nurse using? a. Gordon's Functional Health Patterns b. Activity-exercise pattern assessment c. General to specific assessment d. Problem-oriented assessment

Problem-oriented assessment

Inference

Process of drawing conclusions from related pieces of evidence.

Open ended questions

Prompts patients to describe a situation in more than one or two words.

Negative outcomes of immobility: Adults

Psysiological systems are at risk (loss of job)

Acute inflammation: Vascular and cellular response

Rapid vasodilation that causes redness at the site and localized warmth.

Hyperventilation

Rate and depth of respirations increase

Tachypnea

Rate of breathing is regular but abnormally rapid (greater than 20 breaths/min)

Bradypnea

Rate of breathing is regular but abnormally slow (less than 12 breaths/min).

Thinking Independently

Reads the Nursing Literature, considering a wide range of ideas before making own conclusion.

The nurse is working the night shift on a surgical unit and is making 4:00 AM rounds. The nurse notices that the patient's temperature is 96.8° F (36° C), whereas at 4:00 PM the preceding day, it was 98.6° F (37° C). What should the nurse do? a. Call the health care provider immediately to report a possible infection. b. Administer medication to lower the temperature further. c. Provide another blanket to conserve body temperature. d. Realize that this is a normal temperature variation.

Realize that this is a normal temperature variation.

Humility

Recognize when you need more information to make a decision.

Critical Thinking Involves

Recognizing that an issue exists, analyzing information, evaluating information, and making conclusions.

Responsibility

Refer to policy and procedure manual to review steps of a skill

A nurse just started working at a well-baby clinic. One of her recent experiences was to help a mother learn the steps of breast feeding. During the first clinic visit the mother had difficulty positioning the baby during feeding. After the visit the nurse considers what affected the inability of the mother to breastfeed, including the mother's obesity and inexperience. The nurses review of the situation is called? A. Reflection. B. Perserverance. C. Intuition. D. Problem solving.

Reflection

The nurse is caring for a patient in the endoscopy area. The nurse observes the technician performing these tasks. Which observation will require the nurse to intervene? a. Washing hands after removing gloves b. Disinfecting endoscopes in the workroom c. Removing gloves to transfer the endoscope d. Placing the endoscope in a container for transfer

Removing gloves to transfer the endoscopeThe nurse is caring for a patient who is at risk for infection.

A nurse reviews an immobilized patient's laboratory results and discovers hypercalcemia. Which condition will the nurse monitor for most closely in this patient? a. Hypostatic pneumonia b. Renal calculi c. Pressure ulcers d. Thrombus formation

Renal calculi

The nurse is performing hand hygiene before assisting a health care provider with insertion of a chest tube. While washing hands, the nurse touches the sink. Which action will the nurse take next? a. Inform the health care provider and recruit another nurse to assist. b. Rinse and dry hands, and begin assisting the health care provider. c. Extend the handwashing procedure to 5 minutes. d. Repeat handwashing using antiseptic soap.

Repeat handwashing using antiseptic soap.

Spiritual History

Represents the totality of one's being.

The nurse is gathering data on a patient. Which data will the nurse report as objective data? a. States "doesn't feel good" b. Reports a headache c. Respirations 16 d. Nauseated

Respirations 16

Hyperpnea

Respirations are labored, increased in depth, and increased in rate (greater than 20 breaths/min) (occurs normally during exercise).

Apnea

Respirations cease for several seconds

A patient who visits the surgery clinic 4 weeks after a traumatic amputation of his right lef tells the nurse practitioner that is worried about his ability to continue to support his family. He tells the nurse he feels that he has let his family down after having an auto accident that led to the loss of his left leg. The nurse listens and then ask the patient "How do you see yourself now?" One the basis of Gordons functional health pattern, which pattern does the nurse assess? 1. Health perception-health management. 2. Value-beliefs pattern. 3. Cognitive-perceptual pattern. 4. Self-perception-self-concept pattern.

Self-perception-self-concept-pattern.

Cheyne-Stokes

Respiratory rate and depth are irregular; alternating periods of apnea and hyperventilation.

Hypoventilation

Respiratory rate is abnormally low, and depth of ventilation is depressed. Hypercarbia sometimes occurs.

Supine

Rest on their backs

Which interventions utilized by the nurse will indicate the ability to recognize a localized inflammatory response? a. Vigorous range-of-motion exercises b. Turn, cough, and deep breathe c. Orient to date, time, and place d. Rest, ice, and elevation

Rest, ice, and elevation

Psychosocial History

Reveals the patient's support systems and coping mechanisms.

A nurse is using the critical thinking skill of evaluation. Which action will the nurse take? a. Examine the meaning of data. b. Support findings and conclusions. c. Review the effectiveness of nursing actions. d. Search for links between the data and the nurse's assumptions.

Review the effectiveness of nursing actions.

Leading questions

Risky, limits information.

The nurse is providing an education session to an adult community group about the effects of smoking on infection. Which information is most important for the nurse to include in the educational session? a. Smoke from tobacco products clings to your clothing and hair. b. Smoking affects the cilia lining the upper airways in the lungs. c. Smoking can affect the color of the patient's fingernails. d. Smoking tobacco products can be very expensive.

Smoking affects the cilia lining the upper airways in the lungs.

Negative outcomes of immobility: Adolescents

Social isolation

Body alignment

The individual's center of gravity is stable.

Confidence

Speak with conviction and always be prepared to perform care safely.

A nurse is using a critical thinking model to provide care. Which component is first that helps a nurse make clinical decisions? a. Attitude b. Experience c. Nursing process d. Specific knowledge base

Specific knowledge base.

An aspect of clinical decision making is knowing the patient. Which of the following is the most critical aspect of developing the ability to know the patient? 1. Working in multiple health care settings 2. Learning good communication skills 3. Spending time establishing relationships with patients 4. Relying on evidence in practice

Spending time establishing relationships with patients.

The nurse is caring for a patient who has just delivered a neonate. The nurse is checking the patient for excessive vaginal drainage. Which precaution will the nurse use? a. Contact b. Droplet c. Standard d. Protective environment

Standard

The nurse sets up nonbarrier sterile field on the patients overbed table. In which of the following instances is the field contaminated? 1. Sterile saline solution is spilled on the field. 2. The nurse, who has a cold, wears a double mask. 3. Sterile objects are kept within a 1-inch border of the field. 4. The nurse keeps the top of the table above his or her waist.

Sterile saline is spilled in on the field.

The nurse is caring for an adult patient in the clinic who has been evacuated and is a victim of flooding. The nurse teaches the patient about rest, exercise, and eating properly and how to utilize deep breathing and visualization. What is the primary rationale for the nurse's actions related to the teaching? a. Topics taught are standard information taught during health care visits. b. The patient requested this information to teach the extended family members. c. Stress for long periods of time can lead to exhaustion and decreased resistance to infection. d. These techniques will help the patient manage the pain and loss of personal belongings.

Stress for long periods of time can lead to exhaustion and decreased resistance to infection.

Scientific Method

Systematic, ordered approach to gathering data and solving problems.

Organize assessment on the basis of patient priorities.

Systematicity

Prehypertension range

Systolic: 120-139 Diastolic: 80-89

Stage 1 Hypertension range

Systolic: 140-159 Diastolic: 90-99

Stage 2 Hypertension range

Systolic: >160 Diastolic: >100

Discipline

Take time to be thorough, and manage your time effectively.

Which action indicates a registered nurse is being responsible for making clinical decisions? a. Applies clear textbook solutions to patients' problems. b. Takes immediate action when a patient's condition worsens. c. Uses only traditional methods of providing care to patients. d. Formulates standardized care plans solely for groups of patients.

Takes immediate action when a patient's condition worsens

The nurse is caring for a patient who is susceptible to infection. Which instruction will the nurse include in an educational session to decrease the risk of infection? a. Teaching the patient about fall prevention b. Teaching the patient to take a temperature c. Teaching the patient to select nutritious foods d. Teaching the patient about the effects of alcohol

Teaching the patient to select nutritious foods

During a visit to the clinic, a patient tells the nurse that he has been having headaches on and off for a week. The headaches sometimes make him feel nauseated. Which of the following responses by the nurse is an example of probing? 1. So you've had headaches periodically in the last week and sometimes they cause you to feel nauseated—correct? 2. Have you taken anything for your headaches? 3. Tell me what makes your headaches begin. 4. Uh huh, tell me more.

Tell me what makes your headaches begin

A patient has a head injury and damages the hypothalamus. Which vital sign will the nurse monitor most closely? a. Pulse b. Respirations c. Temperature d. Blood pressure

Temperature

Which process will be required after exposure of a nurse to blood by a cut from a used scalpel in the operative area? a. Placing the scalpel in a needle safe container b. Testing the patient and offering treatment to the nurse c. Removing sterile gloves and disposing of in kick bucket d. Providing a medical evaluation of the nurse to the manager

Testing the patient and offering treatment to the nurse

Acute inflammation: Inflammatory exudate

The accumulation of fluid and dead tissue cells. WBC's form at the site.

Hyperthermia

The body's inability to promote heat loss or reduce heat production

Perfusion

The distribution of red blood cells to and from the pulmonary capillaries.

The nurse is observing a family member changing a dressing for a patient in the home health environment. Which observation indicates the family member has a correct understanding of how to manage contaminated dressings? a. The family member places the used dressings in a plastic bag. b. The family member saves part of the dressing because it is clean. c. The family member removes gloves and gathers items for disposal. d. The family member wraps the used dressing in toilet tissue before placing in trash.

The family member places the used dressings in a plastic bag.

Immobility

The inability to move about freely

Which scenario best illustrates the nurse using data validation when making a nursing clinical decision for a patient? a. The nurse determines to remove a wound dressing when the patient reveals the time of the last dressing change and notices old and new drainage. b. The nurse administers pain medicine due at 1700 at 1600 because the patient reports increased pain and the family wants something done. c. The nurse immediately asks the health care provider for an order of potassium when a patient reports leg cramps. d. The nurse elevates a leg cast when the patient reports decreased mobility.

The nurse determines to remove a wound dressing when the patient reveals the time of the last dressing change and notices old and new drainage.

The nurse is caring for a patient in the hospital. The nurse observes the nursing assistive personnel (NAP) turning off the handle faucet with bare hands. Which professional practice principle supports the need for follow-up with the NAP? a. The nurse is responsible for providing a safe environment for the patient. b. Different scopes of practice allow modification of procedures. c. Allowing the water to run is a waste of resources and money. d. This is a key step in the procedure for washing hands.

The nurse is responsible for providing a safe environment for the patient.

A new nurse is completing an assessment on an 80-year-old patient who is alert and oriented. The patient's daughter is present in the room. Which action by the nurse will require follow-up by the charge nurse? a. The nurse makes eye contact with the patient. b. The nurse speaks only to the patient's daughter. c. The nurse leans forward while talking with the patient. d. The nurse nods periodically while the patient is speaking.

The nurse speaks only to the patient's daughter.

A nurse is assessing body alignment. What is the nurse monitoring? a. The relationship of one body part to another while in different positions b. The coordinated efforts of the musculoskeletal and nervous systems c. The force that occurs in a direction to oppose movement d. The inability to move about freely

The relationship of one body part to another while in different positions

A patient is pyrexic. Which piece of equipment will the nurse obtain to monitor this condition? a. Stethoscope b. Thermometer c. Blood pressure cuff d. Sphygmomanometer

Thermometer

The nurse is caring for an older-adult patient with a diagnosis of urinary tract infection (UTI). Upon assessment the nurse finds the patient confused and agitated. How will the nurse interpret these assessment findings? a. These are normal signs of aging. b. These are early signs of dementia. c. These are purely psychological in origin. d. These are common manifestation with UTIs.

These are common manifestation with UTIs.

The nurse is dressed and is preparing to care for a patient in the perioperative area. The nurse has scrubbed hands and has donned a sterile gown and gloves. Which action will indicate a break in sterile technique? a. Touching clean protective eyewear b. Standing with hands above waist area c. Accepting sterile supplies from the surgeon d. Staying with the sterile table once it is open

Touching clean protective eyewear

A nurse is assessing the skin of an immobilized patient. What will the nurse do? a. Assess the skin every 4 hours. b. Limit the amount of fluid intake. c. Use a standardized tool such as the Braden Scale. d. Have special times for inspection so as to not interrupt routine care.

Use a standardized tool such as the Braden Scale.

The patient has been diagnosed with a spinal cord injury and needs to be repositioned using the logrolling technique. Which technique will the nurse use for logrolling? a. Use at least three people. b. Have the patient reach for the opposite side rail when turning. c. Move the top part of the patient's torso and then the bottom part. d. Do not use pillows after turning.

Use at least three people.

Be objective in asking questions of a patient.

Truth seeking

Major route of transmission example

Unwashed hands of a health care worker

The nurse is caring for a patient in an intensive care unit who needs a bath. Which priorityaction will the nurse take to decrease the potential for a health care-associated infection? a. Use local anesthetic on reddened areas. b. Use nonallergenic tape on dressings. c. Use a chlorhexidine wash. d. Use filtered water.

Use a chlorhexidine wash.

The nurse is caring for a patient on contact precautions. Which action will be most appropriate to prevent the spread of disease? a. Place the patient in a room with negative airflow. b. Wear a gown, gloves, face mask, and goggles for interactions with the patient. c. Transport the patient safely and quickly when going to the radiology department. d. Use a dedicated blood pressure cuff that stays in the room and is used for that patient only.

Use a dedicated blood pressure cuff that stays in the room and is used for that patient only.

The patient is unable to move self and needs to be pulled up in bed. What will the nurse do to make this procedure safe? a. Place the pillow under the patient's head and shoulders. b. Do by self if the bed is in the flat position. c. Place the side rails in the up position. d. Use a friction-reducing device.

Use a friction-reducing device.

The nurse is caring for a patient who has a temperature reading of 100.4° F (38° C). The patient's last two temperature readings were 98.6° F (37° C) and 96.8° F (36° C). Which action will the nurse take? a. Wait 30 minutes and recheck the patient's temperature. b. Assume that the patient has an infection and order blood cultures. c. Encourage the patient to move around to increase muscular activity. d. Be aware that temperatures this high are harmful and affect patient safety.

Wait 30 minutes and recheck the patient's temperature.

The nurse is caring for a patient who becomes nauseated and vomits without warning. The nurse has contaminated hands. Which action is best for the nurse to take next? a. Wash hands with an antimicrobial soap and water. b. Clean hands with wipes from the bedside table. c. Use an alcohol-based waterless hand gel. d. Wipe hands with a dry paper towel.

Wash hands with an antimicrobial soap and water.

The nurse is providing an educational session for a group of preschool workers. The nurse reminds the group about the most important thing to do to prevent the spread of infection. Which information did the nurse share with the preschool workers? a. Encourage preschool children to eat a nutritious diet. b. Suggest that parents provide a multivitamin to the children. c. Clean the toys every afternoon before putting them away. d. Wash their hands between each interaction with children.

Wash their hands between each interaction with children.

Negative outcomes of immobility: Older Adults

Weaker bones, increased risk of falls, increased physical dependence on others.

The nurse is caring for a patient who is at risk for infection. Which action by the nurse indicates correct understanding about standard precautions? a. Teaches the patient about good nutrition b. Dons gloves when wearing artificial nails c. Disposes an uncapped needle in the designated container d. Wears eyewear when emptying the urinary drainage bag

Wears eyewear when emptying the urinary drainage bag

The nurse ask the patient, "Describe for me a typical nights sleep. What do you do to fall asleep? Do you have difficulty falling asleep or staying asleep?" This series of questions would likely occur during which phase of a patient-centered interiew? 1. Orientation. 2. Working phase. 3. Data validation. 4. Termination.

Working Phase

A nurse is assigned to a 42-year-old mother of 4 who weighs 136.2 kg (300 lbs), has diabetes, and works part time in the kitchen of a restaurant. The patient is facing surgery for gallbladder disease. Which of the following approaches demonstrates the nurse's cultural competences in assessing the patients health care problem? 1. "I can tell that your eating habits have led to your diabetes. Is that right?" 2. "It's been difficult for people to find jobs. Is that why you work part time?" 3. "You have four children, do you have any concerns about going home and caring for them?" 4. "I wish patients understood how overeating affects their health."

You have four children, do you have any concerns about going home and caring for them?

Inference (ch16)

Your judgement or interpretation of cues.

Range of motion

the maximum amount of movement available at a joint in one of the three planes of the body: sagittal, transverse, or frontal

Objective Data

what the health professional observes by inspecting, palpating, percussing, and auscultating during the physical examination

Subjective Data

what the patient says during history taking.


Set pelajaran terkait

Homework 12: Photosynthesis and Carbon Fixation

View Set

8 dědičnost multifaktorialnich znaků a chorob

View Set

VCU Basic Human Anatomy- Module II

View Set

Inventors In the Industrial Revolution

View Set

Test One Intro to Clinical Test Answers

View Set